You are on page 1of 292

Table of Contents

PREFACE ............................................................................................................................................. 6 IMPORTANT CHAPTER GUIDELINES ........................................................................................... 7 CHAPTER: SIMPLIFICATION & APPROXIMAITON ................................................................... 8
Solved Examples (Simplification & Approximation) ............................................................................................. 12 Practice Set-1 (Simplification & Approximation) .................................................................................................. 22 Practice Set-2 (Simplification & Approximaiton) .................................................................................................. 25 Simplification & Approximaiton Averages Practice Set-1 (Answers) .................................................................... 27 Simplification & Approximaiton Averages Practice Set-2 (Answers) .................................................................... 28

CHAPTER: NUMBER SERIES......................................................................................................... 29


Solved Examples (Number Series)........................................................................................................................ 33 Practice Set (Number Series)................................................................................................................................ 38 Number Series Practice Set (Answers) ................................................................................................................. 41

CHAPTER: NUMBER SYSTEM....................................................................................................... 42


Solved Examples (Number System)...................................................................................................................... 48 Practice Set-1 (Number System) .......................................................................................................................... 53 Practice Set-2 (Number System) .......................................................................................................................... 55 Number System Practice Set-1 (Answers) ............................................................................................................ 58 Number System Practice Set-2 (Answers) ............................................................................................................ 59

CHAPTER: RATIO, PROPORTION & ALLIGATION .................................................................. 60


Solved Examples (Ratio, Proportion & Alligation) ................................................................................................ 66 Practice Set-1 (Ratio, Proportion & Alligation) ..................................................................................................... 70 Practice Set-2 (Ratio, Proportion & Alligation) ..................................................................................................... 74

IBPS BANK PO EXAM 2013 : Quantitative Aptitude

Ratio, Proportion & Alligation Practice Set-1 (Answers)....................................................................................... 77 Ratio, Proportion & Alligation Practice Set-2 (Answers)....................................................................................... 78

CHAPTER: AVERAGES ................................................................................................................... 79


Solved Examples (Averages) ................................................................................................................................ 80 Practice Set (Averages) ........................................................................................................................................ 86 Averages Practice Set (Answers) .......................................................................................................................... 90

CHAPTER: PERCENTAGES, PARTNERSHIP AND SHARE ....................................................... 91


Solved Examples (Percentages, Partnership and Share) ....................................................................................... 95 Practice Set-1 (Percentages, Partnership and Share).......................................................................................... 101 Practice Set-2 (Percentages, Partnership and Share).......................................................................................... 104 Percentages, Partnership and Share Practice Set-1 (Answers) ........................................................................... 106 Percentages, Partnership and Share Practice Set-2 (Answers) ........................................................................... 107

CHAPTER: PROFIT & LOSS ........................................................................................................ 108


Solved Examples (Profit & Loss) ......................................................................................................................... 112 Practice Set (Profit & Loss) ................................................................................................................................. 117 Profit & Loss Practice Set (Answers) .................................................................................................................. 121

CHAPTER: SIMPLE INTEREST & COMPOUND INTEREST .................................................. 122


Solved Examples (Simple Interest & Compound Interest) .................................................................................. 126 Practice Set (Simple Interest & Compound Interest) .......................................................................................... 132 Simple Interest & Compound Interest Practice Set (Answers) ............................................................................ 136

CHAPTER: TIME AND WORK .................................................................................................... 137


Solved Examples (Time and Work) ..................................................................................................................... 140 Practice Set (Time and Work)............................................................................................................................. 146 Averages Practice Set (Time and Work) ............................................................................................................. 151

IBPS BANK PO EXAM 2013 : Quantitative Aptitude

CHAPTER: SPEED DISTANCE & TIME ..................................................................................... 152


Solved Examples (Speed Distance & Time)......................................................................................................... 156 Practice Set (Speed Distance & Time) ................................................................................................................ 162 Speed Distance & Time Practice Set (Answers) .................................................................................................. 166

CHAPTER: MENSURATION ........................................................................................................ 167


Solved Examples (Mensuration) ........................................................................................................................ 174 Practice Set (Mensuration) ................................................................................................................................ 178 Mensuration Practice Set (Answers) .................................................................................................................. 181

CHAPTER: PERMUTATIONS, COMBINATIONS & PROBABILITY ..................................... 182


Solved Examples (Permutations, Combinations & Probability) .......................................................................... 188 Practice Set-1 (Permutations, Combinations & Probability) ............................................................................... 195 Practice Set-2 (Permutations, Combinations & Probability) ............................................................................... 197 Permutations, Combinations & Probability Practice Set-1 (Answers)................................................................. 199 Permutations, Combinations & Probability Practice Set-2 (Answers)................................................................. 200

CHAPTER: DATA INTERPRETATION ...................................................................................... 201 CHAPTER: DATA INTERPRETATION-TABLE CHART ......................................................... 204
Solved Examples (Data Interpretation-Table Chart) ........................................................................................... 206 Practice Set (Data Interpretation-Table Chart)................................................................................................... 213 Data Interpretation-Table Chart Practice Set (Answers) .................................................................................... 221

CHAPTER: DATA INTERPRETATION-LINE GRAPHS ........................................................... 222


Solved Examples (Data Interpretation-Line Graphs) .......................................................................................... 225 Practice Set (Data Interpretation-Line Graphs) .................................................................................................. 229 Data Interpretation-Line Graphs Practice Set (Answers) .................................................................................... 233

CHAPTER: DATA INTERPRETATION-BAR GRAPHS............................................................ 234 4 IBPS BANK PO EXAM 2013 : Quantitative Aptitude

Solved Examples (Data Interpretation-Bar Graphs)............................................................................................ 237 Practice Set (Data Interpretation-Bar Graphs) ................................................................................................... 243 Data Interpretation-Bar Graphs Practice Set (Answers) ..................................................................................... 247

CHAPTER: DATA INTERPRETATION-PIE DIAGRAM .......................................................... 248


Solved Examples (Data Interpretation-Pie Diagram) .......................................................................................... 250 Practice Set (Data Interpretation-Pie Diagram) .................................................................................................. 256 Data Interpretation- Pie Diagram Practice Set (Answers)................................................................................... 262

CHAPTER: DATA INTERPRETATION-CASE LETS ................................................................ 263


Solved Examples (Data Interpretation-Caselets) ................................................................................................ 266 PracticeSet-(Data Interpretation- Caselets)........................................................................................................ 270 Data Interpretation-Caselets Practice Set (Answers).......................................................................................... 274 Solved Examples (Data Interpretation-Miscellaneous) ...................................................................................... 275 Practice Set (Data Interpretation-Miscellaneous) .............................................................................................. 285 Data Interpretation-Miscellaneous Practice Set (Answers) ................................................................................ 291

FEEDBACK ..................................................................................................................................... 292

IBPS BANK PO EXAM 2013 : Quantitative Aptitude

Preface
Jagranjoshs IBPS PO Exam 2013: Quantitative Aptitude e-Book is a one-stop solution for aspirants who endeavor to leave no stone unturned to score considerably in IBPS PO Written Examination 2013. The eBook is highly useful for all officers level Banking exams - IBPS Specialist officers Exam and SBI PO Exam. IBPS PO Exam 2013: Quantitative Aptitude e-Book is prepared by Jagranjoshs team of subject matter experts who worked up the best to come up with this all-inclusive preparation package for Quantitative Aptitude section of Officers level Banking Exams. This eBook is a perfect blend of chapter-wise basic concepts and questions regarding all the units included in the syllabus for Quantitative Aptitude Section of Officers level Banking Exams. The chapter-wise compilation of this e-Book makes the concept of Quantitative aptitudes easy to understand for students. It further includes previous year questions and model practice sets along with the importance factor of each and every chapter included out here. Our IBPS PO Exam 2013: Quantitative Aptitude e-Book will let students to practice for the QA section within the standard time limit set by the IBPS examination board. This will help them hone time management skills. The IBPS PO Exam 2013: Quantitative Aptitude e-Book includes: Chapter wise Basic Concepts & Questions IBPS PO Previous Year Questions Chapter wise Model Practice Set Important Chapter Guidelines Jagranjoshs IBPS PO Exam 2013: Quantitative Aptitude e-Book is a one-stop solution edition to help preparing for the QA Section of Officers level Banking Exams. All the chapters of this e-book are readerfriendly and easy to understand. Just prepare with it to score more. Our team at Jagranjosh.com wishes all the very best to the aspirants of Banking Exams. All the Best!
Copyright Jagranjosh.com All rights reserved. No part or the whole of this eBook may be copied, reproduced, stored in retrieval system or transmitted and/or cited anywhere in any form or by any means (electronic, mechanical, photocopying, recording or otherwise), without the written permission of the copyright owner. If any misconduct comes in knowledge or brought in notice, strict action will be taken. Disclaimer Readers are requested to verify/cross-check up to their satisfaction themselves about the advertisements, advertorials, and external contents. If any miss-happening, ill result, mass depletion or any similar incident occurs due to any information cited or referenced in this e-book, Editor, Director/s, employees of Jagranjosh.com cant be held liable/responsible in any matter whatsoever. No responsibilities lie as well in case of the advertisements, advertorials, and external contents.

IBPS BANK PO EXAM 2013 : Quantitative Aptitude

Important Chapter Guidelines


Important Chapter Guidelines for IBPS PO 2013 : Quantitative Aptitude Chapter Number System Number Series Percentage Partnership and Share Simplification and Approximation Average Ratio ,Proportion & Alligation Time & Work Profit, Loss & Discount Simple Interest and Compound Interest Time, Distance and Speed Permutation and Combination & Probability Data Interpretation-Table Charts Data Interpretation-Bar Graph Data Interpretation-Line Graph Data Interpretation-Pie Charts Data Interpretation-Caselets Data Interpretation-Miscelleneous Graphs Mensuration Importance Very Important Very Important Very Important Very Important Important Important Important Very Important Important Important Very Important Very Important Very Important Very Important Very Important Very Important Very Important Important

Note : Prepared on the basis of questions asked in previous year papers

IBPS BANK PO EXAM 2013 : Quantitative Aptitude

Chapter: Simplification & Approximaiton


Some Important Concepts BODMAS Rule: This BODMAS Rule shows the correct sequence of all the operations that are to be executed to find out the value of a given exp ression. In this rule B Stands for Bracket, O stands for of, D for Division, M for Multiplication, A for Addition and S for Subtraction. Therefore, the correct order to simplify an expression is: (a) (b) (c) (d) (e) (f) (g) (h) () {} [] of Division Multiplication Addition Subtraction

Modulus of a Real Number: If the real number is r, then |r|= .

Example: What will be the value of x in the following equation? 5 + + x +2 = 9 . Solution: Simplifying the above equation x=9 - 5 2 x= -

x= x=
=1 .

IBPS BANK PO EXAM 2013 : Quantitative Aptitude

Example: If

= , then find the value of

Solution: Given expression:

( x/y =3/2)

Example: Arjun spends of his salary on house rent, of his salary on food and

of his salary

on conveyance. If he has Rs.2400 left with him, then find his expenditure on conveyance. Solution: Suppose Arjuns monthly salary is Rs. x Then, remaining part of his salary = x- ( + + Now, = 2400 x =Rs. 4500. )x=x( )x= = .

Expenditure on Conveyance = Rs.(

4500 ) = Rs.450.

IBPS BANK PO EXAM 2013 : Quantitative Aptitude

Approximation In these types of questions there is no need to find out the exact values. The candidate is required to calculate the approximate value in the following manner: Step I: Round off the numbers given in the question. Step II: Simplify the value Step III: Round off the final simplified value Rounding off numbers: Rounding off the numbers given in the question can be done in the following manner: (a) Rounding off to the nearest 10: Example: Rounded off value of 56 is 60, because the digit at unit place is greater than 5, therefore, we will add 1 to the digit at tens place and replace the units digit by 0. Example: Rounded off value of 54 is 50, because the digit at unit place is less than 5, therefore, the value at tens place will remains the same and units digit will be replaced by 0. Example: Rounded off value of 35 is 40, because the digit at unit place is equal to 5, therefore, we will add 1 to the digit at tens place and replace the units digit by 0. (b) Rounding off to the nearest 100: Example: Rounded off value of 386 is 400, because the digit at tens place is greater than 5, therefore, we will add 1 to the digit at hundreds place and replace the digit at unit and tens places by 00. Example: Rounded off value of 741 is 700, because the digit at tens place is less than 5, therefore, the value at hundreds place will remains the same. Unit and tens digit will be replaced by 00. (c) Rounding off to the nearest 1000 Example: Rounded off value of 1963 is 2000, because the digit at hundreds place is greater than 5, therefore, we will add 1 to the digit at thousand place and replace the ones, tens and hundreds by 000. Although the numbers can be rounded off by the above procedures but it depends on the other numbers involved in the simplification. Rounding off a number to a decimal place:

10

IBPS BANK PO EXAM 2013 : Quantitative Aptitude

The candidate is required to follow the following steps to round off a number to the nth decimal place: Step I: check the digit immediately, next right to the nth place. Step II: Add 1 to the digit in the nth place, if the next right digit is 5 or more, otherwise the digit will remains the same. Step III: Remove all the digits in places to the right of the nth place. Example: Rounded off value of 4.693 to the second place is 4.69, because next right digit to the second place is 3, therefore, the value will remain the same and the digit in place to the right of the second place will be removed.

11

IBPS BANK PO EXAM 2013 : Quantitative Aptitude

Solved Examples (Simplification & Approximation)


Directions (1-5) What will come in place of the question mark (?) in the following question? (IBPS PO/MT Exam 2012) 1. 4003 77 - 21015=? 116 (1) 2477 (2) 2478 (3) 2467 (4) 2476 (5) None of these Solution: ? 116 = 4003 77 21015 or, ? 116 = 308231 21015 = 287216 or, ? 116 = 287216 ?= Ans: (4) 2. (1) 143 (2) (3) 134 (4) (5) None of these Solution:
=

= 2476

- 361 - 361

= = 504 -361 = 143 Ans: (1)

12

IBPS BANK PO EXAM 2013 : Quantitative Aptitude

3. (444440) + (64525) + (399126) =? (1) 280.4 (2) 290.4 (3) 295.4 (4) 285.4 (5) None of these Solution: (444440) + (64525) + (399126) = + +

= 111.1 + 25.8 + 153.5 = 290.4 Ans: (2) 4. - (83)2 = (?)2 + (37)2

(1) 37 (2) 33 (3) 34 (4) 38 (5) None of these Solution: (?)2 + (37)2 = 2 2 2 or, (?) + (37) = 18251 - (83) or, (?)2 + 1369 = 9282-6889 =2393 or, (?)2 = 2393 - 1369 = 1024 ?= Ans: (5) 5. 5 4 11 + 2 =? = 32 - (83)2

(1) 303.75 (2) 305.75 (3) (4) (5) None of these 13 IBPS BANK PO EXAM 2013 : Quantitative Aptitude

Solution: ? = 5 = =

11 + 2 +

+ = 303 + =

= 101 3 + = Ans: (2)

= 305.75

Direction (6-10): What approximate value should come in place of the question mark (?) in the following question? (Note: You are not excepted to calculate the exact value.) (IBPS PO/MT Exam 2012) 6. 8787 343 (1) 250 (2) 140 (3) 180 (4) 100 (5) 280 Solution: ? = 8787 343 = 25.61 7.07 = 181.09 Ans: (3) 7. (1) 48 (2) 38 (3) 28 (4) 18 (5) 58 Solution: 14 (3038) = (?)2 IBPS BANK PO EXAM 2013 : Quantitative Aptitude (3038) = (?)2 =?

or, 38 37.8 = (?)2 (37.8 ~ 38) or, 38 38 = (?)2

Ans: (2) 8. of 4011.33 + of 3411.22 =?

(1) 4810 (2) 4980 (3) 4890 (4) 4930 (5) 4850 Solution: ? = = + 4011.33 + 3411.22

= 2507.08 + 2387.854 = 2507 + 2387 = 4894 Ans: (3) 9. 23% of 6783 + 57% of 8431=? (1) 6460 (2) 6420 (3) 6320 (4) 6630 (5) 6360 Solution: ?= 23% of 6783 + 57% of 8431 6783 + = 23 = 1560.09 + 4805.67 = 6365.76 Ans: (5) 15 IBPS BANK PO EXAM 2013 : Quantitative Aptitude 8431

10. 335.01 244.9955=? (1) 1490 (2) 1550 (3) 1420 (4) 1590 (5) 1400 Solution: ? = 335.01 244.99 = 335 245 55 = 335 Ans: (1) Directions (Q.11-15): What will come in place of the question mark (?) in the following questions? (IBPS PO/MT Exam 2011) 11. 3463 295 - 18611 =? + 5883 (1) 997091 (2) 997071 (3) 997090 (4) 999070 (5) None of these Solution: 3463 x 295 -18611 =? + 5883 ? = 1021585 - 18611 - 5883 = 997091 Ans: (1) = = 1422.27 1490 55

16

IBPS BANK PO EXAM 2013 : Quantitative Aptitude

12. (23.1)2 + (48.6)2 - (39.8)2 =? + 1147.69 (1) (13.6)2 (2)

(3) 163.84 (4) 12.8 (5) None of these Solution: 533.61 + 2361.96 - 1584.04 =? + 1147.69 or, ? = 1311.53 - 1147.69 = 163.84 Ans: (3) 13. + =?

1)

2) 0.75 3) 1

4) None of these Solution: Ans: (4) + = = = =

17

IBPS BANK PO EXAM 2013 : Quantitative Aptitude

14. [(3 (1) 2 (2) 8 (3) 382 (4) 386

) X (8

+7

)]- 98 =?

(5) None of these Solution: [ (3 +1) (8 + 7) ]- 98

= [4 = [60 Ans: (3) 15. (1) 3844 (2) 3721 (3) 3481 (4) 3638

15

] - 98

8] - 98 = 480 - 98 = 382

- (54)2 =

+ (74)2

(5) None of these Solution: - (54)2 - (74)2 =

or ,

- 2916 - 5476

= 8453 - 2916 - 5476 = 61 18 IBPS BANK PO EXAM 2013 : Quantitative Aptitude

or , ? = (61)2 = 3721 Ans: (2) Directions ( 16-20): What approximate value should come in place of question mark (?) in the following questions? (Note: You are not expected to calculate the exact value] (IBPS PO/MT Exam 2011) 16. 39.897% of 4331 + 58.779% of 5003 =? (1) 4300 (2) 4500 (3) 4700 (4) 4900 (5) 5100 Solution: 40 + 59 = 1732 + 2950 = 4682 4700

Ans: (3) 17. 43931.03 / 2111.02 x 401.04 =? 1) 8800 2) 7600 3) 7400 4) 9000 5) 8300 Solution: 43931 or, ? = 44000 2111 401 =? 400

19

IBPS BANK PO EXAM 2013 : Quantitative Aptitude

or, ? =

400 = 8800

Ans: (5) 18. 1) 3000 2) 2800 3) 2500 4) 3300 5) 2600 Solution: 34.993 = 80 x 35 = 2800 34.993 =?

Ans: (2) 19. 1) 7600 2) 7650 3) 7860 4) 7560 5) 7680 Solution: 17 + 349 =? or, 366 21 = ? 7680. 21 +349 =? 21.003

or, ? = 7686 Ans: (5) 20

IBPS BANK PO EXAM 2013 : Quantitative Aptitude

20. 59.88 1) 10 2) 50 3) 30 4) 70 5) 90

12.21

6.35 =?

Solution: 60 Ans: (3)

12

6 = 30

21

IBPS BANK PO EXAM 2013 : Quantitative Aptitude

Practice Set-1 (Simplification & Approximation)


Directions (Q. 1-5): What will come in place of question mark (?) in the following questions? (IBPS RRB Grade A Officer Exam 2012) 1. (1) (2) (3) (4) (5) (2) (3) (4) (5) 5. (1) (2) (3) (4) (5) 6 2 4 0 None of these 64 8 7 9

2 16 8 None of these

2. 55% of (1) (2) (3) (4) (5) 3. (1) (2) (3) (4) 18 (5) 32 4. (1) 81 22 126.5 126.6 124.6 125.4 None of these

Directions (Q. 6-10). What approximate value will come in place of question mark (?) in the following questions? (You are not expected to calculate the exact value.) 6. 68% of 1288 + 26% of 734 215 =? (1) (2) (3) (4) (5) 7. (1) (2) (3) (4) (5) 670 530 420 780 960 620 930 540 850 710

8. 6578 67 15 =? 6 IBPS BANK PO EXAM 2013 : Quantitative Aptitude

(1) (2) (3) (4) (5) 9. (1) (2) (3) (4) (5) 10. (1) (2) (3) (4) (5)

200 250 150 100 300

(5) None of these 13. (0.0729 0.1)3 (0.081 10)5 (0.3 3)5 = (0.9)? + 3 (1) (2) (3) (4) (5) 14. (1) (2) (3) (4) (5) 1 2 4 7 None of these of 5) = 149.8 112

540 760 800 1260 1040

840 910 1320 1120 1550

18 324 24 None of these

15. (27)2 6 9 + (7)3 + 71 = (?)3 431 (1) (2) (3) (4) (5) 11 (13)3 13 (11)2 None of these

Directions (Q.11 to 15): What will come in place of question mark (?) in the following questions? (RBI Grade B Officers Exam 2011) 11. [(3024 189)1/2 + (684 19)2] = (?)2 + 459 (1) (2) (3) (4) (5) -27 -29 31 841 1089 of 30% of 216 =?

Directions (Q. 16-20): What will come in place of question mark (?) in the following questions? (Corporation Bank PO Exam 2011) 16. 5907 1296 144 = ? 8 (1) (2) (3) (4) (5) 17. 726.75 767.25 737.25 676.75 None of these

12. 4.4 times (1) (2) (3) (4) 81.9 83.7 87.3 89.1 23

IBPS BANK PO EXAM 2013 : Quantitative Aptitude

(1) (2) (3) 3 (4) 9 (5) -3 18. (1) (2) (3) (4) (5) 19. 11 5 7 9 None of these

(1) (2) 200 (3) (4) 100 (5) 10 20. (1) (2) (3) (4) (5) 169 13 14 196 None of these

24

IBPS BANK PO EXAM 2013 : Quantitative Aptitude

Practice Set-2 (Simplification & Approximaiton)


Directions (Q. 1-5): What approximate value will come in place of question mark (?) in the following questions? (Corporation Bank PO Exam 2011) (You are not expected to calculate the exact value.) 1. 3237 31 15 = ? 17 (1) (2) (3) (4) (5) 2. (1) (2) (3) (4) (5) 1760 1880 1950 1720 1650 (1) (2) (3) (4) (5) 90 100 110 120 80 (5) 160 5. 61% of 981 150.17 = ? 65% of 676 (1) (2) (3) (4) (5) 760 780 830 860 890

Directions-(Q. 6-10): What will come in place of question-mark (?) in the following questions? (Allahabad Bank Probationary Officers Exam 2011) 6. of 30% of 3420 = (?)2 x 2 (81)2 7 9 81 49

7. 1898 73 x 72 = (?)2 x 13 (1) (2) (3) (4) (5) 8. - 256 256 12 144 -16 =? 42 1024 1764 (1024)2 32

3. (34.34)2 + (5.96) 2 (23.09) 2 =? (1) (2) (3) (4) (5) 4. (1) (2) (3) (4) 200 180 120 140 25 510 540 620 680 650

(1) (2) (3) (4) (5)

IBPS PO EXAM 2013 : Quantitative Aptitude

9. (081)2+ (0729)3 x (09)2 = (0.9)?-3 (1) (2) (3) (4) (5) 6 2 4 0 None of these x 5 =? + 154

12. 89988% of 6999 + 50002% of 99999 170015 =? (1) (2) (3) (4) (5) 13. (1) (2) (3) (4) (5) 760 800 690 870 780 990 900 920 960 860 =?

10. 65% of (1) (2) (3) (4) (5) 56 28 35 32 None of these

Directions-(Q. 11-15) What approximate value will come in place of question-mark (?) in the following questions? (Allahabad Bank Probationary Officers Exam 2011) (You are not expected to calculate the exact value.) 11. (1) (2) (3) (4) (5) 620 670 770 750 700 =? 8

14. 6999 70005 x 94998 =? x 19999 (1) (2) (3) (4) (5) 475 420 320 540 525

15. (4999)2 (89)2 (159)2 =? (1) (2) (3) (4) (5) 2165 2000 1965 1920 1885

26

IBPS PO EXAM 2013 : Quantitative Aptitude

Simplification & Approximaiton Averages Practice Set -1 (Answers)


1) 2) 3) 4) 5) 6) 7) 8) 9) 10) 5 1 1 4 3 4 2 2 5 3 11) 12) 13) 14) 15) 16) 17) 18) 19) 20) 2 4 1 5 1 3 2 4 5 2

27

IBPS PO EXAM 2013 : Quantitative Aptitude

Simplification & Approximaiton Averages Practice Set -2 (Answers)

1) 2) 3) 4) 5) 6) 7) 8)

1 1 5 3 5 3 3 5

9) 10) 11) 12) 13) 14) 15)

4 2 2 4 4 1 1

28

IBPS PO EXAM 2013 : Quantitative Aptitude

Chapter: Number Series


In Number Series, questions are asked on the basis of relation between numbers given in a series. The questions asked can be divided into different types: Type I: In this type of questions, a series of numbers is given with one number missing represented by a question mark. Candidate has to select from the options choices to correct option in place of the question mark. The given sequences of numbers will be such that each number follows its predecessor in the same way, i.e., according to a particular pattern. Candidates are required to find out the correct ways in which the sequence is formed and there after find out the number to complete the series. 1. 30, 34, 43, 59, 84, 120,? (1) 169 (2) 148 (3) 153 (4) 176 (5) None of these Solution: (1) The given pattern is: +22, 32, +42, + 62, +72 So, missing term is 169=120 +72 2. 40, 54, 82,?, 180,250 (1) 142 (2) 124 (3) 136 (4) 163 (5) None of these Solution: (2) The pattern is: +14, + 28, + 42, + 52, + 70 29 IBPS PO EXAM 2013 : Quantitative Aptitude

So, missing term is 82 + 42=124 Type II: Here, we are given a sequence of number. Whole sequence, except the odd number follow a certain rule. You have to find that number which does not follow the rule. 1. 0, 1,3,8,18,35,264 (1) 62 (2) 35 (3) 18 (4) 8 (5) None of these Solution: (1) The pattern is +(02+1), +(12+1), + (22+1) ,+ (32+1), + (42+1), + (52+1) So, 62 is wrong and must be replaced by 35 + (52+1) = 62 2. 1, 9, 125, 49, 729, 121, 2147 (1) 2147 (2) 729 (3) 125 (4) 1 (5) None of these Solution: (1) Type III: In this type of questions, a number series is given. After the series is over, in the next line, a number is followed by (A), (B), (C), (D) and (E). The candidates have to complete the series starting with the number given following the sequence of the given series. 1. 2, 3 4, (A) 9, (B) 20, (C) 43, (D) 90 (E)

Which number will come in place of (D)? (1) 58 (2) 99 30 IBPS PO EXAM 2013 : Quantitative Aptitude

(3) 48 (4) 59 (5) None of these Solution: (4)

Similarly,

Hence, the number 59 will come in place of D.

2. 3, 3

4, (A)

10, (B)

33, (C)

136, (D) (E)

Which number will come in place of (E)? (1) 1035 (2) 1165 (3) 1039 (4) 891 (5) None of these Solution: (2)

31

IBPS PO EXAM 2013 : Quantitative Aptitude

Similarly,

Hence, the number 1165 will come in place of E.

32

IBPS PO EXAM 2013 : Quantitative Aptitude

Solved Examples (Number Series)


Directions: (1-5) In each of these questions a number series is given. In each series only one number is wrong. Find out the wrong number. (IBPS CWE PO MT 2012) 1. 5531 5506 5425 5304 5135 4910 4621 (1) 5531 (2) 5425 (3) 4621 (4) 5135 (5) 5506 Solution: The number should be 5555 in place of 5531. -72, -92, -112, -132, -152, -172... Ans: (1) 2. 6 7 9 13 26 37 69 (1) 7 (2) 26 (3) 69 (4) 37 (5) 9 Solution: The number should be 21 in place of 26. +1, +2, +4, +8, +16, +32 Ans: (2) 3. 1 3 10 36 152 760 4632 (1) 3 (2) 36 (3) 4632 (4) 760 33 IBPS PO EXAM 2013 : Quantitative Aptitude

(5) 152 Solution: The number should be 770 In place of 760. 1 +2, 2 +4, 3 +6, 4 + 8, 5 +10, 6 + 12, ... Ans: (4) 4. 4 3 9 34 96 219 435 (1) 4 (2) 9 (3) 34 (4) 435 (5) 219 Solution: The series is 02+ 4, 12+2, 32+0, 62-2, 102-4, 152- 6,212 - 8... Hence, 435 should be replaced with 433 Ans: (1) 5. 157.5 (1) 1 (2) 2 (3) 6 (4) 157.5 (5) 45 Solution: The number should be 2 in place of 1. 3.5, 3, 2.5, 2, 1.5, 1, ... Ans: (1) Directions (6-10): In the following number series only one number is wrong. Find out the wrong number. 6. 7 12 (1) 7 (2) 12 (3) 40 34 IBPS PO EXAM 2013 : Quantitative Aptitude 40 222 1742 17390 208608 45 15 6 3 2 1

(4) 1742 (5) 208608 Solution: The pattern of number series is as follows: 7 12 40 222 1744 2 - 2 = 12 4 - (2 + 6) = 48 - 8 = 40 6 - (8 + 10) = 240 - 18 = 222 8 - (18 + 14) = 1776 - 32 = 1744 1742

10 - (32 + 18) = 17440 - 50 = 17390

Ans: (4) 7. 6 (1) 91 (2) 70558 (3) 584 (4) 2935 (5) 35277 Solution: The pattern of number series is as follows: 6 91 582 2935 7 + 72 = 42 + 49 = 91 6 + 62 = 546 + 36 = 582 5+52 =2910 + 25=2935 4 + 42 = 11740 + 16 = 11756 584 91 584 2935 11756 35277 70558

11756 x 3 + 32 = 35268 + 9 = 35277 Ans: (3) 35 IBPS PO EXAM 2013 : Quantitative Aptitude

8. 9050 (1) 3478 (2) 1418 (3) 5675 (4) 2147 (5) 1077

5675

3478 2147 1418 1077 950

Solution: The pattern of number series is as follows: 9050 I53 = 9050 - 3375 = 5675 5675 - 133 = 5675 - 2197 = 3478 3478 - 113 = 3478 - 1331 = 2147 2147 - 93 = 2147 - 729 = 1418 1418 - 73 = 1418 - 343 = 1075 Ans: (5) 9. 1 4 25 256 (1)3125 (2) 823543 (3) 46656 (4) 25 (5) 256 Solution: The pattern of number series is as follows: 11 = 1;-22 = 4; 33 = 27 77 = 823543 Ans: (4) 25; 44 = 256; 55 = 3125; 66 = 46656; 3125 46656 823543 1077

36

IBPS PO EXAM 2013 : Quantitative Aptitude

10. 8424 (1) 131.625 (2) 1051 (3) 4212 (4) 8424 (5) 263.25

4212 2106 1051 526.5

263.25 131.625

Solution: The pattern of number series is as follows: 8424 4212 2106 1053 2 = 4212 2 = 2106 2 = 1053 2 = 526.5 1051

526.5 2 = 263.25 263.25 2 = 13 1.625 Ans: (2)

37

IBPS PO EXAM 2013 : Quantitative Aptitude

Practice Set (Number Series)


Directions (Q. 1-3): What will come in place of question mark (?) in the following number series? (IBPS RRB Grade Officer Exam 2012) 1. 987 587 331 187 123 (?) (1) 104 (2) 113 (3) 107 (4) 114 (5) None of these 2. 125 171 263 401 585 (?) (1) 835 (2) 815 (3) 792 (4) 788 (5) None of these 3. 121 132 167 226 309 (?) (1) 424 (2) 413 (3) 427 (4) 416 (5) None of these Directions (Q. 4-5): In the following number series, only one is wrong. Find out the wrong number. 4. 454 327 648 524 842 713 1036 (1) 327 (2) 648 (3) 521 (4) 842 (5) 713 5. 72.5 86 113 168 275 491 923 (1) 86 (2)113 (3)168 (4)275 (5)491 Directions (Q. 6-10): What will come in place of question mark (?) in the following number series? (RBI GradeB Officers Exam 2011) 6. 17 19 (1) (2) (3) (4) (5) 33 (?) 129 227

64 73 67 72 None of these 451 620 763 (?)

7. 35 256 38

IBPS PO EXAM 2013 : Quantitative Aptitude

(1) (2) (3) (4) (5)

680 893 633 880 None of these 868 917 (?) 1051

(1) (2) (3) (4) (5)

253 239 246 253 None of these

8. 18 139 (1) (2) (3) (4) (5)

13. 342 337.5 328.5 315 297 (?) 265.5 274.5 270

1042 1036 942 996 None of these (?) 1162 874 730

9. 2890 658 (1) (2) (3) (4) (5)

260 None of these 14. 161 164 179 242 497 (?) (1) (2) (3) (4) (5) 1540 1480 1520 1440 None of these

1684 1738 1784 1672 None of these

10. 14 1004 1202 1251.5 1268 (?) (1) (2) (3) (4) (5) 1267.5 1276.25 1324.5 1367.25 None of these

15. 239 254 284 344 464 (?) (1) (2) (3) (4) (5) 726 716 724 714 None of these

Directions (Q. 11-15): What will come in place of question mark (?) in the following number series? (Corporation Bank PO 2011) 11. 8 11 20 47 128 (?) (1) (2) (3) (4) (5) 483 488 397 371 None of these

Directions-(Q. 16-20) What will come in place of question-mark (?) in the following number series? (Allahabad Bank Probationary Officers Exam 2011) 16. 958 833 733 658 608, (?) (1) (2) (3) (4) (5) 577 583 567 573 None of these

12. 71 78 99 134 183 (?) 39

IBPS PO EXAM 2013 : Quantitative Aptitude

17. 11 10 18 51 200, (?) (1) (2) (3) (4) (5) 885 1025 865 995 None of these

19. 14 24 43 71 108 (?) (1) (2) (3) (4) (5) 194 154 3) 145 4) 155 5) None of these

18. 25 48 94 186 370 (?) (1) (2) (3) (4) (5) 738 744 746 724 None of these

20. 144 173 140 169 136 (?) (1) (2) (3) (4) (5) 157 148 164 132 None of these

40

IBPS PO EXAM 2013 : Quantitative Aptitude

Number Series Practice Set (Answers)


1) 2) 3) 4) 5) 6) 7) 8) 9) 10) 3 2 4 5 3 3 4 1 2 2 11) 12) 13) 14) 15) 16) 17) 18) 19) 20) 4 3 2 3 5 2 4 1 2 5

41

IBPS PO EXAM 2013 : Quantitative Aptitude

Chapter: Number System


Introduction: Number is a symbol which represents quantity. There are three types of numbers: 1. Real Numbers: Real numbers are those numbers which can be easily indentify and quantify. For example: -10, -7.33, -1, 0, 1, 2, 5.77 etc. 2. Imaginary Numbers: Imaginary numbers are those numbers which we can just imagine but cannot physically perceive. For example: , etc, numbers are imaginary. is represented by i . Square root of all negative

3. Complex Number: Combination of Real and Imaginary number is called complex numbers. For example: (2+5i) , (1+3i) etc . Here we will only discuss about the real numbers . Types of Real Numbers: there are two types of real numbers 1. Rational numbers: - All that numbers which can be expressed in the form of where p & q are integers and q 0 are called rational numbers. =

For example: - -1, 2, , 0, 1 , 2.7 etc. , here -1 = Again Rational numbers are classified as:

(a) Integers: All rational numbers which do not have decimal or fractional parts are called integers. For example: -3, -1, 0 , 1 , 2 etc . Integers are of two type whole numbers and natural numbers. All the non negative integers are whole numbers , for example 0 , 1 , 2 , 3 etc and all the whole numbers except 0 are natural numbers , for example 1, 2 , 3 etc

42

IBPS PO EXAM 2013 : Quantitative Aptitude

(b) Fractions: All rational numbers which are in the form of q 0 and p is not a multiple of q are called fractions. For example: - 1.2, , , 1.7, .3 etc. Fractions are of three types: Proper Improper Mixed fractions

where p & q are integers and

A proper fraction is a fraction whose numerator is smaller than denominator, for example , etc. An improper fraction is a fraction whose numerator is equal to or greater than its , etc. and a mixed fraction is an integer plus a fraction, for

denominator for example example , etc.

2. Irrational Numbers:- All that numbers which cannot be expressed in the form of are called irrational numbers. They have non-terminating and non-recurring decimal parts. For example: On the basis of origin: 1. Prime Numbers: All the natural numbers greater than 1 which are only divisible by 1 and the number itself are called prime numbers. For example: 2, 3, 5, 7, 11, 13, 17 etc. 2. Composite Numbers: All the natural numbers greater than 1 which are divisible by at least one more number other than 1 and the number itself are called composite numbers. For example: 4, 6, 8, 9, 10, 12 etc. Note: 1 is neither a prime number nor a composite. On basis of divisor: 1. Even Numbers: All the natural numbers which are multiple of 2 are called even numbers. For example: 2, 4, 6, 8, 10 etc. , , etc.

43

IBPS PO EXAM 2013 : Quantitative Aptitude

Odd Numbers: All the natural numbers which are not a multiple of 2 are called odd numbers. They are denoted as 2k For example: 3, 5, 7, 9, 11 etc. To find whether a number is prime or not. Step 1: Find the approximate square root of a number . Step 2: Check if any prime number from 2 to that square root divides that number or not. Step 3: If none of those prime number divides the number than the number must be prime number. Example: Take 631 , the approx square root of 631 is 25 , now from 2 to 25 there are 2 , 3 ,5 , 7 , 11 , 13 , 17 , 19 and 23 prime number . Since none of these divides 631, so 631 must be a prime number. Conversion of recurring decimal into fraction. The form of purely recurring number = 1, where k is a natural number.

Let x = 0.77777.

10x = 7.77777

If x = 0.27272727 .

100 x = 27.272727

The form of purely recurring number =

Let x= 0.143333333..

100x = 14.333333.

1000x = 143.3333. = 129

44

IBPS PO EXAM 2013 : Quantitative Aptitude

Quotient and remainder: Dividend = (Divisor Quotient) + remainder.

For example: If dividend = 15968, Quotient = 89 and remainder = 37 then Divisor is? Divisor = =

45

IBPS PO EXAM 2013 : Quantitative Aptitude

Divisibility Rules: For Number 2 Divisibility Rule Example Note

If the last digit of a number is 0,2,4,6,8 , then the number is divisible by 2 If the sum of all the digits of a number is divisible by 3 , then the number is divisible by 3

742 is divisible by 2 but 743 is not.

1458 (sum of digits = 18) is 766 (sum of digits divisible by 3, but 766 (sum of =19) the remainder digits =19) is not divisible by 3. when 19 is divided by 3 i.e. 1 will also be the remainder when 766 is divided by3 6732 is divisible by 4 as 32 is divisible by 4, but 2142 is not divisible by 4. Similarly the remainder when 42 is divided by 4 i.e 2 will also be the remainder when 2142 is divided by 4.

If the last two digits of a number are divisible by 4, then the number is also divisible by 4

If the last digits of a number 1465, 1320 are divisible by 5 are 0 and 5, then the as their last digit is 5 and 0 number is divisible by 5. respectively. If the number is divisible by 2 and 3 both, then it is also divisible by 6. 1452 is divisible by both 2 and 3 so it is divisible by 6 also, but 3362 is not divisible by 6 as it is not divisible by 3. If the number is divisible by 4 and 6 both, then it is not necessary that it is divisible by 24 (6 4).

If the last three digit of a number are divisible by 8 or are 000, then the number is divisible by 8 .

43102 and 13000 are divisible by 8 since 102 is divisible by 8 and 13000 have 000 as last three digits, but 2148 is not as

The remainder when 148 is divided by 8 i.e. 4 will also be the remainder of 2148

46

IBPS PO EXAM 2013 : Quantitative Aptitude

148 is not divisible by 8 9 If the sum of all digits of a number is divisible by 9 , then the number is also divisible by 9. 25344 (sum of digits = 18) is divisible by 9 , 764 (sum of digits =17) is not.

when divided by 8. The remainder when 17 is divided by 9 i.e 8 will also be the remainder when 764 is divided by 9.

11

If the difference between the sum of the digits in the even places and the sum of the digits in the odd places is either 0 or is divisible by 11 , then the number is also divisible by 11.

9415956 is divisible by 11 as the difference of 9+1+9+6 =25 and 4+5+5 = 14 is 11, but 31872 is not as the sum of even places = 13 and sum of odd is 8 their difference is neither 0 nor 11.

47

IBPS PO EXAM 2013 : Quantitative Aptitude

Solved Examples (Number System)


1. When X is subtracted from the numbers 9, 15 and 27, the remainders are in continued proportion. What is the value of X? (IBPS CWE PO MT 2012) (1) 8 (2) 6 (3) 4 (4) 5 (5) None of these Solution: Let be subtracted from the numbers 9, 15 and 27 we get continue proportion. Now, 9 : 15 : 27 b2 = ac (15 )2 = (9 ) (27 ) or, 225 30 + 2 = 243 + 2 36 or, 6 = 243 225 = 18 X=3 Hence number become 9 - x = 9-3=6 15 x = 15 -3 = 12 And 27-x = 27 -3 = 24 6 : 12 : 24 = 1: 2: 4 Thus 1: 2: 4 is continued proportion Ans: (5) 2. Sum of three consecutive numbers is 2262. What is 41% of the highest number? (IBPS CWE PO MT 2012) (1) 301.5.1 (2) 303.14 (3) 308.73 (4) 306.35 (5) 309.55 Solution: Let the three consecutive number be x, x + 1 and x + 2. Then, x + x + 1 + x + 2 =2262 or, 3x = 2262 3 = 2259 x= = 753 48 IBPS PO EXAM 2013 : Quantitative Aptitude

The Numbers are 753, 754, 755. The highest number is 755 41% of 755 = 755 = 41 7.55 = 309.55 Ans: (5) 3. Rachita enters a shop to buy ice-creams, cookies and pastries. She has to buy at least 9 units of each. She buys more cookies than ice-creams and more pastries than cookies. She picks up a total of 32 items. How many cookies does she buy? (IBPS CWE PO MT 2012) (1) Either 12 or 13 (2) Either 11 or 12 (3) Either 10 or 11 (4) Either 9 or 11 (5) Either 9 or 10 Solution: Total number of items = 32 Maximum number of ice creams = 9 pastries> cookies> ice cream So, 13 10 9 12 11 9 Hence number of cookies is either 10 or 11. Number of pastries is either 13 or 12. Ans: (3) 4. The fare of a bus is Rs. x for the first five kilometres and Rs. 13/- per kilometre thereafter. If a passenger pays Rs. 2402/- for a journey of 187 kilometres, what is the value of X? (IBPS CWE PO MT 2012) (1) Rs. 29 (2) Rs. 39 (3) Rs. 36 (4) Rs. 31 (5) None of these Solution: Let the fare of first five kilometres be Rs. x. Total distance = 187 km Remaining distance = 187 - 5 = 182 km Now, x + 182 13 = 2402 49 IBPS PO EXAM 2013 : Quantitative Aptitude

x = 2402 - 2366= Rs.36 Ans: (3) 5. The product of three consecutive even numbers is 4032. The product of the first and the third number is 252. What is five times the second number? (1) 80 (2) 100 (3) 60 (4) 70 (5) 90 Solution: Let the three consecutive even numbers be 2x, 2x + 2 and 2x+4. Then, (2x) (2x + 2) (2x + 4) = 4032 ... (I) Again, product of first and third number 2x (2x + 4) = 252 ... (II) Putting the values of the product of first and third number in eqn (I), we have (2x + 2) 252 = 4032 or, 2x + 2 = = 16 x=7 Hence, first number = 7 8 = 14 Second number = 7 x 2 + 2 = 16 And third number = 7 x 2 + 4 = 18 Five times of second number = 5 x 16 = 80 Ans: (1) 6. Rubina could get equal number of Rs. 55, Rs. 85 and Rs. 105 tickets for a movie. She spends Rs. 2,940 for all the tickets. How many of each did she buy? (1) 12 (2) 14 (3) 16 (4) Cannot be determined (5) None of these Solution: Let she buy x tickets. Then total money spent = 55x + 85x + 105x or, 245x = 2940 or, x = 12 Ans: (1) 50 IBPS PO EXAM 2013 : Quantitative Aptitude

7. Seema bought 20 pens, 8 packets of wax colours, 6 calculators and 7 pencil boxes. The price of one pen is Rs. 7, one packet of wax colour is for Rs. 22, one calculator is for Rs. 175 and one pencil box costs Rs. 14 more than the combined price of one pen and one packet of wax colours. How much amount did Seema pay to the shopkeeper? (1) Rs. 1,491 (2) Rs. 1,725 (3) Rs. 1,667 (4) Rs. 1,527 (5) None of these Solution: Price of one pencil box = 14 + (Price of one pen + Price of one packet of wax colours)= 14+ (7+22) = Rs. 43 Total amount paid by Seema = {(20 7) + (8 22) + (6 175) + (7 43)} = Rs 1667 Ans: (3) 8. In a cricket match, Sachin and Viru scored a century each (more than 100 runs) and Yuvi and Gauti scored a half century each. Gauti scored 76 runs and Yuvi scored 12 runs less than Gauti. Viru scored 102 runs. The sum of the scores of all the four players is 442. How many runs did Sachin score? (Corporation Bank PO 2011) (1) (2) (3) (4) (5) 200 210 180 160 None of these

Solution: Sachin's score = 442 76 (76 12) 102 = 200 Ans: (1) 9. The sum of seven consecutive even numbers of Set A is 378. What is the sum of a different set of four consecutive numbers whose lowest number is 23 less than the mean of Set A? (1) (2) (3) (4) (5) 132 146 156 124 None of these 51 IBPS PO EXAM 2013 : Quantitative Aptitude

Solution: Mean of Set A Lowest number of Set B = (54 23) = 31 Sum of the four numbers of Set B = 31 + 32 + 33 + 34 = 130 Ans: (5) 10. The sum of six consecutive even numbers of Set-A is 402. What is the sum of another SetB of four consecutive numbers whose lowest number is 15 less than double the lowest number of set- A? (Allahabad Bank Probationary Officers Exam 2011) (1) (2) (3) (4) (5) 444 442 440 446 None of these = 67 - 1 = 66

Solution: Third even number = smallest even number = 62

smallest number of set B = 2 x 62 - 15 = 109 required sum = 109 + 110 + 111+ 112 = 442 Ans: (2)

52

IBPS PO EXAM 2013 : Quantitative Aptitude

Practice Set-1 (Number System)


1. A person on tour has Rs 360 for his daily expense. He decides to extend his tour programme by 4 days which leads to cutting down daily expense by Rs 3 a day. The number of days of his tour programme is (1) (2) (3) (4) 15 20 18 16

2. Two times a two-digit number is 9 times the number obtained by reversing the digits and sum of the digits is 9. The number is (1) (2) (3) (4) 72 54 63 81

3. If 5 students utilize 18 pencils in 9 days, how long at the same rate will 66 pencils last for 15 students? (1) (2) (3) (4) 10 days 12 days 11 days None of these

4. A man has certain number of small boxes to pack into parcels. If he packs 3, 4, 5 or 6 in a parcel, he is left with one over; if he packs 7 in a parcel, none is left over. What is the number of boxes he may have to pack? (1) (2) (3) (4) 106 301 309 400

5. Out of a group of swans, 7/2 times the square foot of the number are playing on the shore of the pond. The two remaining are inside the pond. What is the total number of swans? (1) 10 (2) 14 (3) 12 53 IBPS PO EXAM 2013 : Quantitative Aptitude

(4) 16 6. In a family, each daughter has the same number of brothers as she has sisters and each son has twice as many sisters as he has brothers. How many sons are there in the family? (1) (2) (3) (4) 4 3 2 5

7. A yearly payment to a servant is Rs. 90 plus one turban. The servant leaves the job after 9 months and received 65 and a turban. Then find the price of the turban. (1) (2) (3) (4) Rs 10 Rs 15 Rs 7.50 Cannot be determined

8. Mr. Mukherjee is 5 yr older to his wife and they have one son and one daughter. If the daughter is 23 yr younger to his mother, what is the difference in the ages of the brother and the sister? (1) (2) (3) (4) 7 yr 12 yr 4 yr 2 yr

9. The expenses of a hotel consist of two parts. The first one varies with the number of inmates, while the second one is fixed. When the numbers of inmates are 275 and 350, the expenses are Rs 1600 and Rs 1900 respectively. The expenses when the number of inmates is 315, are (1) (2) (3) (4) Rs 1760 Rs 1680 Rs 1780 Rs 1660

10. If the numerator of a fraction is doubled and the denominator is increased by 3, the new fraction is 3/5 what is the original fraction, if its denominator is more than twice the numerator by 1? (1) (2) (3) (4) 3/7 6/13 1/3 5/11 54 IBPS PO EXAM 2013 : Quantitative Aptitude

Practice Set-2 (Number System)


1. The sum and product of two numbers are 5 and 6, respectively. The sum of reciprocals of their squares is (1) (2) (3) (4) 2. The sum of the squares of 3 consecutive positive numbers is 365. The sum of the numbers is (1) (2) (3) (4) 30 33 36 45

3. The sum of a natural number and its square equals the product of the first three prime numbers. The number is (1) (2) (3) (4) 2 3 5 6

4. 47 is added to the product of 71 and an unknown number. The new number is divisible by 7, giving the quotient 98. The unknown number is a multiple of (1) (2) (3) (4) 2 7 5 3

55

IBPS PO EXAM 2013 : Quantitative Aptitude

5. Two natural numbers are in the ratio 3: 5 and their product is 2160. The smaller of the number is (1) (2) (3) (4) 36 24 18 12

6. The sum of two numbers is 10. Their product is 20. Find the sum of the reciprocals of the two numbers. (1) 1 (2) (3) (4) 7. The sum of the digits of a two digit numbers is 10. The number formed by reversing the digits is 18 less than the original number. Find the original number. (1) (2) (3) (4) 81 46 64 60

8. The least number to be subtracted from 36798 to get a number which is exactly divisible by 78 is (1) (2) (3) (4) 18 60 38 68

9. The sum of first sixty number from one to sixty is divisible by (1) (2) (3) (4) 13 59 60 61

56

IBPS PO EXAM 2013 : Quantitative Aptitude

10. Eight consecutive number are given. If the average of the two numbers that appear in the middle is 6, then the sum of the eight given numbers is (1) (2) (3) (4) 36 48 54 64

57

IBPS PO EXAM 2013 : Quantitative Aptitude

Number System Practice Set-1 (Answers)


1 2 3 4 5 6 7 8 9 10 (2) (4) (3) (2) (4) (2) (1) (4) (1) (1)

58

IBPS PO EXAM 2013 : Quantitative Aptitude

Number System Practice Set-2 (Answers)


1 (1)

(2)

(3)

(4)

(1)

(3)

(3)

(2)

(4)

10

(2)

59

IBPS PO EXAM 2013 : Quantitative Aptitude

Chapter: Ratio, Proportion & Alligation


Introduction:Ratio is the relation which one quantity bears to another of the same kind. The ratio of two quantities a and b is the fraction and we write it as a: b. In the ratio a: b, we call a as the first term or antecedent and b, the second term or consequent. Note: The multiplication or division of each term of a ratio by the same non- zero number does not affect the ratio. Compound Ratio: - It is obtained by multiplying together the numerators for new numerator and denominators for new denominator. Example 1. If the ratios are 4:3, 15:20, 2:6 and 3:5 find the compound ratio? Sol 1. Required ratio = Duplicate ratio of a: b = Triplicate ratio of a: b = etc.

Example2. If we divide 4185 into two parts such that they are in ratio 7:2, then find the values of both the parts? Sol 2. Let the actual variable be 7x and 2x. Then 7x+2x = 4185 So, the 1st part = 7 The 2nd part = 2 Note: 60 IBPS PO EXAM 2013 : Quantitative Aptitude

The ratio of first , second and third quantities is given by ac : bc : bd If the ratio between first and second quantity is a:b and third and fourth is c:d. a: b c: d ac : bc : bd Similarly, the ratio of first, second, third and fourth quantities is given by ace : bce : bde : bdf If the ratio between first and second quantity is a: b and third and fourth is c:d .

a : b c : d e: f ace : bce : bde : bdf Example 3. If Savita has Rs 1880. How much money does Ravina have if the ratio of money with savita and Rita is 15: 7 and that with Rita and Ravina 16: 7? Solution3: Savita : Rita : Ravina 15 : 7 16 : 7 15 240 : 112 : 14 The ratio of money with Savita , Rita and Ravina is 240 : 112 : 14. We see that 240 x = 1880 Hence 14 61 IBPS PO EXAM 2013 : Quantitative Aptitude

Proportion
Introduction:Four quantities are said to be proportional if the two ratios are equal i.e. the A, B, C and

D are proportion. It is denoted by :: it is written as A : B : C : D where A and D are extremes and B and C are called means . Product of the extreme = Product of the means Direct proportion: - The two given quantities are so related that if one quantity increases (or decreases) then the other quantity also increases (or decreases). Example 1. If 5 pens cost Rs 10 then 15 pen cost? Sol 1. It is seen that if number of pens increases then cost also increases. So, 5 pens: 15 pens:: Rs 10 : required cost Required cost = Inverse proportion: - The two given quantities are so related that if one quantity increases (or decreases) then the other quantity also decreases (or increases). Example 2.If 10 men can do a work in 20 days then in how many days 20 men can do that work? Sol 2. Here if men increase then days should decrease, so this is a case of inverse proportion, so 10 men: 20 men :: required days : 20 days Required days =

Rule of three: It Is the method of finding 4th term of a proportion if all the other three are given, if ratio is a:b :: c:d then , d= Compound proportion: - Lets take an example to explain this. 62 IBPS PO EXAM 2013 : Quantitative Aptitude

Example3. If 9 men can do a piece of work in 40 days of 10 hours each, how many men will it take to do 12 times the amount of work if they work for 30 days of 9 hours? Solution 3: Step 1. Days: . : . Hours: . : .. Work: . : .. Step 2. 1. Compare days with men : to do the work in less days we will need more men , so it is the case of inverse proportion hence , 30 : 40 :: 9 : required no. of men 2. Compare hours with men : to do the work in less hours we will need more men , so it is the case of inverse proportion hence , 9 : 10 :: 9 : required no. of men 3. Compare work with men : to do the more work we will need more men , so it is the case of direct proportion hence , 1: 12:: 9: required no. of men Put all the values in step 1, 30: 40 9: 10 1: 12 Now, :: 9: required no. of men :: 9: required no. of men

63

IBPS PO EXAM 2013 : Quantitative Aptitude

ALLIGATION Introduction:The word allegation means linking. It is used to find: 1. The proportion in which the ingredients of given price are mixed to produce a new mixture at a given price. 2. The mean or average value of mixture when the price of the two or more ingredients and the proportion in which they are mixed are given. Mathematical Formula: For two ingredient:-

CP of unit quantity of cheaper (c)

CP of unit quantity of dearer (d)

Mean Price

(d-m) Then (cheaper quantity): (dearer quantity) = (d-m): (m-c)

(m-c)

Example 1: If the rice at Rs 3.20 per kg and the rice at Rs 3.50 per kg be mixed then what should be their proportion so that the new mixture be worth Rs 3.35 per kg ? Sol 1: CP of 1 kg of cheaper rice 320 paisa CP of 1 kg of dearer rice 350 paisa

Mean Price (m) 335 paisa 15 64 15 IBPS PO EXAM 2013 : Quantitative Aptitude

By allegation: Hence they must be mixed in equal proportion i.e. 1:1 For three ingredient:1. The price of the ingredient should be reduced to one denomination and then place them in ascending order under one another. 2. After that place the mean prices to the left of all the price 3. Then pair the price so that price less than and greater than the mean prices go together. 4. Then find out the difference between mean price and each price and place it opposite to the price with which it is linked. 5. These difference will help to find out the given answer and similarly it will work for four ingredients . Example 2: Find out the ratio of new mixture so that it will cost Rs 1.40 per kg from the given three kinds of rice costing Rs 1.20, Rs 1.45 and Rs 1.74? Sol 2: 1st rice cost = 120, 2nd rice cost = 145 and 3rd rice cost = 174 paisa. From the above rule: we have, 120 140 145 174 5+34 [(145-140) + (174-140)] 20 20 (140-120) (140-120)

Therefore, three rice must be mixed in 39: 20: 20 ratios to have a new mixture of rice.

65

IBPS PO EXAM 2013 : Quantitative Aptitude

Solved Examples (Ratio, Proportion & Alligation)


1. A certain amount was to be distributed among A, B and C in the ratio 2: 3: 4 respectively, but was erroneously distributed in the ratio 7: 2: 5 respectively. As a result of this, B got Rs 40 less. What is the amount? (IBPS CWE PO MT 2012) (1) Rs. 210 (2) Rs. 270 (3) Rs. 230 (4) Rs. 280 (5) None of these Solution: Let the amount be x. B's share = Due to error B's share = Difference in B's share due to error = 40 or, or, 24x = 40 x= Ans: (1) 2. Rs.73,689/- are divided between A and B in the ratio 4: 7. What is the difference between thrice the share of A and twice the share of B? (IBPS CWE PO MT 2012) (1) Rs. 36,699 (2) Rs. 46,893 (3) Rs. 20,097 (4) Rs. 26,796 (5) Rs. 13,398 Solution: Let As share be 4x and Bs share be 7x. 4x +7x = 73689 or, 11x = 73689 66 IBPS PO EXAM 2013 : Quantitative Aptitude = 40 = 40 126 = Rs. 210

x=

= 6699

As share = 6699 4 = 26796 Bs share = 6699 7 = 44893 Thrice the share of A = 26796 3 = 80388 Twice the share of B = 46893 2= 93786 Difference = 93786 - 80388 = Rs. 13398 Ans: (5) 3. The ratio of the present age of Manisha and Deepali is 5:X. Manisha is 9 years younger than Parineeta. Parineetas age after 9 years will be 33 years. The difference between Deepali's and Manisha's age is the same as the present age of Parineeta. What should come in place of X? (1) 23 (2) 39 (3) 15 (4) Cannot be determined (5) None of these Solution: Parineetas present age = (33 - 9 =) 24 yrs. Manisha's present age = (24 - 9 =) 15 yrs. Deepalis present age = IS + 24 = 39 yrs. Ratio of the present age of Manisha and Deepali = 15 : 39 = 5 : 13 X = 13 Ans: (5) 4. The ratio between the three angles of a quadrilateral is 3 : 5 : 9. The value of the fourth angle of the quadrilateral is 71. What is the difference between the largest and the smallest angles of the quadrilateral? (IBPS RRB Group A Officers Exam 2012) (1) 82 (2) 106 (3) 102 (4) 92 (5) None of these 67 IBPS PO EXAM 2013 : Quantitative Aptitude

Solution. Let the quadrilateral angles be 3x, 5x, 9x and 71. Total sum of angles = 3x + 5x + 9x + 71 = 360 or, 17x = 360 71 = 289 x = 17 Hence angles are 51, 85, 153, and 71. Difference = 153 51 = 102. Ans: (3) 5. The second largest and the smallest angles of a triangle are in the ratio of 6 : 5. The difference between the second largest angle and the smallest angle 'of the triangle is equal to 9. What is the difference between the smallest and the largest angles of the triangle? (IBPS RRB Group A Officers Exam 2012) (1) 36 (2) 24 (3) 12 (4) 18 (4) None of these Solution. Let the second largest angle of the triangle be 6x and the smallest angle 5x. Now, 6x - 5x = 9 or, x = 9 Second largest angle = 54 Smallest angle = 45 Sum of angles of a triangle = 180 largest angle = 180 - 99 = 810 Difference = 81 - 45 = 36. Ans: 1 6. The ratio between the speed of a bus and train is 15 : 27 respectively. Also, a car covered a distance of 720 km in 9 hours. The speed of the bus is three- fourth the speed of the car. How much distance will the train cover in 7 hours? (Allahabad Bank Probationary Officers Exam 2011) (1) 760 km (2) 756 km (3) 740 km 68 IBPS PO EXAM 2013 : Quantitative Aptitude

(4) Cannot be determined (5) None of these Solution: Speed of the car = Speed of the bus = Speed of the train = 60km/hr 108km/hr = = 80hm/hr

Distance covered by train in 7 hours = (7 x 108 =) 756 km Ans: (2)

69

IBPS PO EXAM 2013 : Quantitative Aptitude

Practice Set-1 (Ratio, Proportion & Alligation)


1. A container has 80 L of milk. From this container 8 L of milk was taken out and replaced by water. The process was further repeated twice. The volume of milk in the container after that is [SSC Quantitative Aptitude (Arihant)] a) 58.23 L b) 85.23 L c) 58.32 L d) 85.32 L 2. A can contains a mixture of two liquids A and B in the ratio 7 : 5. When 9 L of mixture is drawn off and the can is filled with B, the ratio of A and B becomes 7 : 9. Litres of liquid A contained by the can initially was a) 10 b) 20 c) 21 d) 25 3. What number should be added to or subtracted from each term of the ratio 17 : 24 so that it becomes equal to 1 : 2? a) 5 is subtracted b) 10 is added c) 7 is added d) 10 is subtracted 4. The ratio of weekly incomes of A and B is 9 : 7 and the ratio of their 70 expenditures is 4 : 3. If each saves Rs. 200 per week, then the sum of their weekly incomes is a) Rs. 3200 b) Rs. 4200 c) Rs. 4800 d) Rs. 5600 5. The ratio of alcohol and water in 40 L of mixture is 5 : 3.8 L of the mixture is removed and replaced with water, Now, the ratio of the alcohol and water in the resultant mixture is a) 1 : 2 b) 1 : 1 c) 2 : 1 d) 1 : 3 6. Rama's expenditure and savings are in the ratio 3 : 2. His income increases by 10%. His expenditure also increases by 12%. His saving increases by a) 7% b) 10% c) 9% d) 13% 7. Three numbers are in the ratio 3 : 4 : 5. The sum of the largest and the smallest equals the sum of the second and 52. The smallest number is a) 20

IBPS PO EXAM 2013 : Quantitative Aptitude

b) 27 c) 39 d) 52 8. The ratio of the ages of Ram and Rahim 10 yr ago was 1 : 3. The ratio of their ages 5 yr hence will be 2 : 3. Then, the ratio of their present ages is a) 1 : 2 b) 3 : 5 c) 3 : 4 d) 2 : 5 9. The ratio of milk and water in mixtures of four container are 5 : 3, 2 : 1, 3 : 2 and 7 : 4, respectively. In which container is the quantity of milk, relative to water, minimum? a) First b) Second c) Third d) Fourth 10. In a mixture of 25 L, the ratio of acid to water is 4 : 1. Another 3 L of water is added to the mixture. The ratio of acid to water in the new mixture is a) 5 : 2 b) 2 : 5 c) 3 : 5 d) 5 : 3 11. A shopkeeper buys two varieties of tea, the price of the first being twice the second. He sells the mixture at Rs 36 per kilogram and makes a profit of 20%. If the ratio of quantities of the first and second 71

variety in these mixture is 3 : 4, then what is the cost price of each variety of tea ? [Mission MBA MAT (Arihant)] a) Rs 21, 42 b) Rs 15, 30 c) Rs 16.5, 33 d) Rs 17, 34 12. Two liquids are mixed in the ratio 3 : 5 and the mixture is sold at Rs 120 with a profit of 20%. If the first liquid is costlier than the second by Rs 2 per litre, find the cost of the costlier liquid per litre. a) Rs 92.30 b) Rs 74.10 c) Rs 101.25 d) Rs 99.25 13. A grocer buys two kinds of rice at Rs 1.80 and Rs 1.20 per kg respectively. In what proportion should these be mixed, so that by selling the mixture at Rs 1.75 per kg, 25% may be gained? a) 2 : 1 b) 3 : 2 c) 3 : 4 d) 1 : 2 14. A jar full of whisky contains 40% of alcohol. A part of this whisky is replaced by another containing 19% alcohol and now the percentage of alcohol was found to be 26. The quantity of whisky replaced is a)

IBPS PO EXAM 2013 : Quantitative Aptitude

b) c) d) 15. A container contains 240 L of wine. 80 L is taken out of the container everyday and an equal quantity of water is put into it. Find the quantity of the wine that remains in the container at the end of the fourth day. a) 39.2 L b) 32 L c) 42.5 L d) 47.40 L 16. A tea trader mixed two varieties of tea, one costing Rs 3.50 per kg and the other costing Rs 4 per kg and sells 40 kg of the mixture to a vendor at Rs 4.50 per kg and makes a profit of 20%. How much of each variety did the vendor mix? a) 30 kg, 10 kg b) 20 kg, 20 kg c) 10 kg, 30 kg d) None of these 17. A vessel contains 50 L milk. The milkman delivers 10 L to the first house and adds an equal quantity of water. He does exactly the same at the second and third house. What is the ratio of milk and water when he has finished delivering at the third house? a) 61 : 64 72

b) 27 : 37 c) 16 : 19 d) None of these 18. Prabhu purchased 30 kg of rice at the rate of Rs 17.50 per kg and another 30 kg rice at a certain rate. He mixed the two rice and sold the entire quantity at the rate of Rs 18.60 per kg and made 20% over all profit. At what price per kg did he purchase the lot of another 30 kg rice? a) Rs 14.50 b) Rs 12.50 c) Rs 15.50 d) Rs 13.50 19. A person has a chemical of Rs 50 per litre. In what ratio should water be mixed in that chemical so that after selling the mixture at Rs 40 per litre he may get a profit of 50%. a) 8 : 7 b) 9 : 8 c) 10 : 7 d) 4 : 3 20. A trader has 50 kg of pulses, part of which he sells at 8% profit and the rest at 18% profit. He gains 14% on the whole. What is the quantity sold at 18% profit? a) 30 kg b) 35 kg c) 40 kg d) None of these

IBPS PO EXAM 2013 : Quantitative Aptitude

21. A container of capacity 120 L is filled with milk and water. 80% of milk and 40% of water is taken out of vessel. It is found that the vessel is vacated by 65%. What is the ratio of milk to water? a) 5 : 3 b) 6 : 5 c) 3 : 5 d) 4 : 3

22. How much water should be added to 60 L of milk at L for Rs 10 so

as to have a mixture worth Rs per litre? a) 16 L b) 15 L c) 18 L d) 20 L

73

IBPS PO EXAM 2013 : Quantitative Aptitude

Practice Set-2 (Ratio, Proportion & Alligation)


1. An employer reduces the number of employees in the ratio 8 : 5 and increases their wages in the ratio 7 : 9. As a result, the overall wages bill is a) increased in the ratio 56 : 69 b) decreased in the ratio 56 : 45 c) increased in the ratio 13 : 17 d) decreased in the ratio 17 : 13 2. A can contains a mixture of two liquids A and B in the ratio 7 : 5. When 9 L of mixture is drawn off and the can is filled with B, the ratio of A and B becomes 7 : 9. Litres of liquid A contained by the can initially was e) 10 f) 20 g) 21 h) 25 3. What number should be added to or subtracted from each term of the ratio 17 : 24 so that it becomes equal to 1 : 2? e) 5 is subtracted f) 10 is added 74 g) 7 is added h) 10 is subtracted 4. The ratio of weekly incomes of A and B is 9 : 7 and the ratio of their expenditures is 4 : 3. If each saves Rs. 200 per week, then the sum of their weekly incomes is e) Rs. 3200 f) Rs. 4200 g) Rs. 4800 h) Rs. 5600 5. Rama's expenditure and savings are in the ratio 3 : 2. His income increases by 10%. His expenditure also increases by 12%. His saving increases by e) 7% f) 10% g) 9% h) 13% 6. If A : B is 2 : 3, B : C is 6 : 11, then A : B : C is a) 2 : 3 : 11 b) 4 : 6 : 22 IBPS PO EXAM 2013 : Quantitative Aptitude

c) 4 : 6 : 11 d) 2 : 6 : 11 7. The ratio of the ages of Ram and Rahim 10 yr ago was 1 : 3. The ratio of their ages 5 yr hence will be 2 : 3. Then, the ratio of their present ages is e) 1 : 2 f) 3 : 5 g) 3 : 4 h) 2 : 5 8. If the annual income of A, B and C are in the ratio 1 : 3 : 7 and the total annual income of A and C is Rs. 800000, then the monthly salary of B is a) Rs. 20000 b) Rs. 25000 c) Rs. 30000 d) Rs. 15000 9. Two numbers are such that the ratio between them is 4 : 7. If each is increased by 4 the ratio becomes 3 : 5. The larger number is a) 36 b) 48 c) 56 75

d) 64 10. Acid and water are mixed in a vessel A in the ratio of 5 : 2 and in the vessel B in the ratio of 8 : 5. In what proportion should quantities be taken out from the two vessels, so as to form a mixture in which the acid and water will be in the ratio of 9 : 4? a) 7 : 2 b) 2 : 7 c) 7 : 4 d) 2 : 3 11. In an alloy, Zinc and Copper are in the ratio 1 : 2. In the second alloy, the same elements are in the ratio 2 : 3. If these two alloys be mixed to form a new alloy in which two elements are in the ratio 5 : 8, the ratio of these two alloys in the new alloy is a) 3 : 10 b) 3 : 7 c) 10 : 3 d) 7 : 3 12. A boy has a few coins of denominations 50 paise, 25 paise and 10 paise in the ratio 1 : 2 : 3. If the total amount of the coins is Rs. IBPS PO EXAM 2013 : Quantitative Aptitude

6.50, the number of 10 paise coins is a) 5 b) 10 c) 15 d) 20 13. The sum of the ages of a father and his son is 100 yr now. 5 yr ago their ages were in the ratio of 2 : 1. The ratio of the ages of father and son after 10 yr will be a) 5 : 3 b) 4 : 3 c) 10 : 7 d) 3 : 5 14. In a school having roll strength 286, the ratio of boys and girls is 8 : 5. If 22 more girls get admitted into the

school, the ratio of boys and girls becomes a) 12 : 7 b) 10 : 7 c) 8 : 7 d) 4 : 3 15. A box contains Rs. 1,50 paise and 25 paise coins in the ratio 8 : 5 : 3. If the total amount of money in the box is Rs. 112.50, the number of 50 paise coins is a) 80 b) 50 c) 30 d) 42

76

IBPS PO EXAM 2013 : Quantitative Aptitude

Ratio, Proportion & Alligation Practice Set-1 (Answers)

1) 2) 3) 4) 5) 6) 7) 8) 9) 10) 11)

c c d a b a c b c a a

12) 13) 14) 15) 16) 17) 18) 19) 20) 21) 22)

c d c d b a d a a a b

77

IBPS PO EXAM 2013 : Quantitative Aptitude

Ratio, Proportion & Alligation Practice Set-2 (Answers)

1) 2) 3) 4) 5) 6) 7) 8)

b c d a a c b b

9) 10) 11) 12) 13) 14) 15)

c a a c a d b

78

IBPS PO EXAM 2013 : Quantitative Aptitude

Chapter: Averages
The term Average refers to the sum of all observations divided by the total number of observations. Average is used quite regular in our day to day life. For example to calculate the average marks of the students, Average height of a particular group etc. The term average is also referred to as Mean. Basic formula to calculate the average is as follows: Average = ( Example. What is the average of First 10 Prime numbers? Solution: First 10 Prime number are 2,3,5,7,11,13,17,19,23,29. Hence, Average = {2+3+5+7+11+13+17+19+23+29} / 10 = 129 / 10 = 12.90 )

So, Average of First 10 Prime numbers is 12.90.

Example. The total number of sales visits made by a Salesman in the month of June is 90. What is the Average visit he makes per day? Solution: Number of days in the month of June are 30 Hence, Average Visit per day = Number of total visits / Number of total days = 90 / 30 =3 So, the salesman makes 3 visits per day.

79

IBPS PO EXAM 2013 : Quantitative Aptitude

Solved Examples (Averages)


1. Ramola's monthly income is three times Ravina's monthly income. Ravina's monthly income is fifteen per cent more than Ruchira's monthly income. Ruchira's monthly income is Rs. 32,000. What is Ramola's annual income? (1) Rs. 1,10.400 (2) Rs. 13,24,800 (3) Rs. 36,800 (4) Rs. 52,200 (5) None of these Solution: Ravina's monthly income = 32000 = Rs. 36800

Ramola's monthly income = 3

36800 = Rs. 110400

Ramola's annual income = 12

110400 = Rs. 1324800

Ans: (2) 2. The average marks in English of a class of 24 students is 56. If the marks of three students were misread as 44, 45 and 61 in lieu of the actual marks 48, 59 and 67 respectively, then what would be the correct average? (1) 56.5 (2) 59 (3) 57.5 (4) 58 (5) None of these Solution: Total marks = 24 56 = 1344 Total of actual marks = 1344 - (44 + 45 + 61) + (48 + 59 + 67) = 1368 Actual Average = = 57

80

IBPS PO EXAM 2013 : Quantitative Aptitude

Ans: (5) 3. In a test, a candidate secured 468 marks out of maximum marks' A:. Had the maximum marks' A' converted to 700, he would have secured 336 marks. What was the maximum marks of the test? (1) 775 (2) 875 (3) 975 (4) 1075 (5) None of these Solution: Converted maximum marks = 700

Converted marks = 336 % marks = x 100= 48%

468 is 48 % of maximum marks 'A' A=


Ans : (3)

100 =975

4. The ratio between the speed of a truck, car and train is 3: 8: 12. The car moved uniformly and covered a distance of 1040 km in 13 hours. What is the average speed of the truck and the train together? (IBPS RRB Group A Officers Exam 2012)
(1) 75km/hr (2) 60 km/hr (3) 48 km/hr (4) Cannot be determined (5) None of these

Solution: Speed of car Ratio of speed of truck, car and train = 3 : 8 : 9 Now 8x = 80 81 IBPS PO EXAM 2013 : Quantitative Aptitude

x = 10 Hence truck = 30 kmph. Train = 90 kmph. Average speed of truck and train together

Ans: 2 5. The circumference of a circle is twice the perimeter of a rectangle. The area of the circle is 5544 sq cm. What is the area of the rectangle if the length of the rectangle is 40cm? (IBPS RRB Group A Officers Exam 2012) (1) 1120 sq cm (2) 1020 sq cm (3) 1140 sq cm (4) 1040 sq cm (5) None of these Solution: Area of circle

r = 42 Circumference of circle = 2 perimeter of rectangle Or, Or, perimeter of rectangle = 132 cm Or, 2(l + b) = 132 l + b = 66 b = 66 40 = 26 82 IBPS PO EXAM 2013 : Quantitative Aptitude

Area of rectangle = 40 26 = 1040 cm2 = 1040 sq.cm. Ans: 4 6. Among three numbers the first is twice the second thrice the third. If the average of the three numbers is 49.5, then the difference between the first and the third number is (1) 54 (2) 28 (3) 39.5 (4) 41.5 Solution: Let first number = x Then, second number = and third number = According to given condition, X + + = 3 49.5

= 148.5

11x = 6 148.5

x =

= 81

First number = 81 Third Number = = 27

Required difference = 81-27=54 Ans: (1)

83

IBPS PO EXAM 2013 : Quantitative Aptitude

7. 10 yrs ago, the average age of P and Q was 20 yrs. Average age of P, Q is 30 yrs now. After 10 yr, the age of R will be (1) (2) (3) (4) 35 yr 40 yr 30 yr 45 yr

Solution: 10 yr ago, total age P and q = 20 2 = 40 yr Present total age of P and Q = 40+210= 60 yr Present total age of P and Q and R=303= 90 yr Rs age = 90-60 = 30 yr After 10 yr Rs age = (30+10) yr = 40 yr Ans: (2) 8. In an examination, the average of marks was found to be 50. For 100 students, marks was computed wrongly as 90 instead of 60.For deducting marks for computational errors, the average of marks came down to 45. The total number of candidate, who appeared at the examination, was (1) (2) (3) (4) 600 300 200 150

Solution: Let the total number of candidate be x. By given condition. 50x- 90x100+ 60x100= 45x

50x 9000+6000 = 45x

50x 45x = 3000


x= Ans: (1) 84 IBPS PO EXAM 2013 : Quantitative Aptitude = 600.

9. The average of 25 observations 13. It was later found that an observation 73 was wrongly entered as 48. The new average is (1) (2) (3) (4) 12.6 14 15 13.8 Required new average = = Ans: (2) 10. A tabulator while calculating the average marks of 100 students of an examination, by mistake enters 68, instead of 86 and obtained the average marks of those students are (1) (2) (3) (4) 58.8 57.82 58.81 57.28 = = 14

Solution:

Solution: Actual total number of 100 students = 5800 + (86 - 68) = 5818 Required actual average = Ans: (1) = 58.18

85

IBPS PO EXAM 2013 : Quantitative Aptitude

Practice Set (Averages)


1. The average age of a jury of 5 is 40. If a member aged 35 resigns and a managed 25 becomes a member, then the average age of the new jury is a) 30 yr b) 38 yr c) 40 yr d) 42 yr 2. The average of the runs made by Raju, Shyam and Hari is 7 less than that made by Shyam, Hari and Kishore. If the number of Kishore's run is 35, what is Raju's run? a) 21 b) 35 c) 7 d) 14 3. The mean of 50 numbers is 30. Later it was discovered that two entries were wrongly entered as 82 and 13 instead of 28 and 31. Find the correct mean. a) 36.12 b) 30.66 86 IBPS PO EXAM 2013 : Quantitative Aptitude c) 29.28 d) 38.21 4. One third of a certain journey is covered at the rate of 25 km/h, one-fourth at the rate of 30 km/h and the rest at 50 km/h. The average speed for the whole journey is a) 35 km/h b) c) 30 km/h d) 5. The average of 5 numbers is 140. If one number is excluded, the average of the remaining 4 numbers is 130. The excluded number is a) 135 b) 134 c) 180 d) 150

6. The average weight of 5 persons sitting in a boat is 38 kg. The average weight of the boat and the persons sitting in the boat is 52 kg. What is the weight of the boat? a) 228 kg b) 122 kg c) 232 kg d) 242 kg 7. There are 50 students in a class. Their average weight is 45 kg. When one student leaves the class the average weight reduces by 100 g. What is the weight of the student who left the class? a) 45 kg

9. In a class, the average score of girls in an examination is 73 and that of boys is 71. The average score for the whole class is 71.8. Find the percentage of girls a) 40% b) 50% c) 55% d) 60% 10. The average of the first 100 positive integers is a) 100 b) 51 c) 50.5 d) 49.5

b) 47.9 kg c) 49.9 kg d) 50.1 kg 8. Out of 4 numbers, whose average is 60, the first one is one fourth of the sum of the last three. The first number is a) 15 b) 45 c) 48 d) 60 11. Out of the three numbers, the first number is twice of the second and the second is thrice of the third number. If the average of these 3 numbers is 20, then the sum of the largest and smallest numbers is a) 24 b) 42 c) 54 d) 60

87

IBPS PO EXAM 2013 : Quantitative Aptitude

12. The average age of 40 students of a class is 18 yr. When 20 new students are admitted to the same class, the average age of the students of the class is increased by 6 months. The average age of newly admitted students is a) 19 yr b) 19 yr 6 months c) 20 yr d) 20 yr 6 months 13. The average age of 40 students of a class is 15 yr. When 10 new students are admitted, the average age is increased by 0.2 yr. The average age of new students is a) 15.2 yr b) 16 yr c) 16.2.yr d) 16.4 yr 14. The average of 6 observations is 45.5. If one new observation is added to the previous observation, then the new average becomes 47. The new observation is a) 58 b) 56 c) 50 88

d) 46 15. The average of marks scored by the students of a class is 68. The average of marks of the girls in the class is 80 and that of boys is 60. What is the percentage of boys in the class? a) 40 b) 60 c) 65 d) 70 16. The average age of 30 boys in a class is 15 yr. One boy aged 20 yr, left the class but two new boys came in his place whose ages differ by 5 yr. If the average age of all the boys now in the class still remains 15 yr, the age of the younger newcomer is a) 20 yr b) 15 yr c) 10 yr d) 8 yr

IBPS PO EXAM 2013 : Quantitative Aptitude

17. Out of 10 teachers of a school, one teacher retires and in his place a new teacher of age 25 yr joins. As a result of it, the average age of the teachers is reduced by 3 yr. The age of the retired teacher is a) 60 yr b) 58 yr c) 56 yr d) 55 yr 18. The mean weight of 34 students of a school is 42 kg. If the weight of the teacher be included, the mean rises by 400 g. Find the weight of the teacher (in kg). a) 66

b) 56 c) 55 d) 57 19. Ram aims to score an average of 80 marks in quarterly and half yearly exams. But his average in quarterly is 3 marks less than his target and that in half yearly is 2 marks more than his aim. The difference between the total marks scored in both the exams is 25. Total marks aimed by Ram is a) 380 b) 400 c) 410 d) 430

89

IBPS PO EXAM 2013 : Quantitative Aptitude

Averages Practice Set (Answers)

1) 2) 3) 4) 5) 6) 7) 8) 9) 10)

b d c b c b c c a c

11) 12) 13) 14) 15) 16) 17) 18) 19)

b b b b b b d b a

90

IBPS PO EXAM 2013 : Quantitative Aptitude

Chapter: Percentages, Partnership and Share


INTRODUCTION The word per cent means per hundred. Thus 19 parts out of 100 parts. This can also be written as . Fraction Equivalents of important Percentages.

PER CENT TO FRACTION To convert a per cent to a fraction, divide it by 100 and delete the % sign. Example: 2% can be converted to a fraction as PER CENT OF A NUMBER Per cent of a number is the product of equivalent fraction (of rate per cent) and the number. Example: To find out 25% of 500 91 IBPS PO EXAM 2013 : Quantitative Aptitude

Solution: Required value = 25% of 500

equivalent fraction for 25% = 125 Example: 9% of what number is 36? Solution: the required number (base number)

= 400 Example: If 30% of a number is 48, then what is 40% of the number? Solution: Here, unitary method can be used to save the time. 30% 48 Hence, the required value is 64 Calculating % EXCESS OR % SHORTNESS 1% 40%

When a number A exceeds the another number B by x %, then % shortness of

It implies that B is less than A by

Similarly, if a number A is short of (or less than) B by x%, then % excess of B

92

IBPS PO EXAM 2013 : Quantitative Aptitude

i.e. B is more than A by Example: If the income of Ram is more than that of Mohan by 25% then by much percentage Mohan's income is than that of Ram?

Solution: Required % shortness (less) income of Mohan = 20% Therefore, income of Mohan is 20% less than that of Ram. Partnership and Shares Meaning: When two or more than two persons run a business jointly, they are called partners in that business and the deal between them is known as partnership. There are two types of partners in the business 1. Working Partner: A person who manages the business is known as working partner. 2. Sleeping Partner: A person who simply invests the money is known as sleeping partner. Some Important Formulae: Suppose two persons P and Q invests Rs. X and Rs. Y respectively for a year in a business, then their share of profit or loss at the end of the year: = Suppose two persons P and Q invests Rs. X for m month and Rs. Y for n months respectively, then = Example1: P, Q and R started a business by investing Rs. 1, 50,000, Rs. 2, 50,000 and 3, 50,000 respectively. At the end of the year, out of an annual profit of Rs.30, 000 find the share of P, Q and R respectively. Solution: Ratio of shares of P, Q and R respectively = Ratio of their investments 93 IBPS PO EXAM 2013 : Quantitative Aptitude

= 1,50,000 : 2,50,000 : 3,50,000 = 3: 5:7

Share of Ps profit = Rs. (30,000 Share of Qs profit = Rs. (30,000 Share of Rs profit = Rs. (30,000

) = Rs. 6,000. ) = Rs. 10,000. ) = Rs. 14,000.

Example2: Ram started a business investing Rs. 50,000.After 4 months, Shyam joined him with a capital of Rs. 30,000 .After another 2 months, Mohan joined them with a capital of Rs. 60,000.At the end of the year, they made a profit of Rs. 24,000.Find the share of profits of Ram, Shyam and Mohan. Solution: According to the given problem, it is clear that Ram invested his capital for 12 months, Shyam invested for 8 months and Mohan invested for 6 months. Then, ratio of their Capitals = (50,000 12) : (30,000 8) : (60,000 6) = 60:24:36 = 5:2:3

Share of Rams profit = (24,000 Share of Shyams profit = (24,000 Share of Mohans profit = (24,000

= Rs.12,000;

) = Rs.4,800; ) = Rs.7,200.

94

IBPS PO EXAM 2013 : Quantitative Aptitude

Solved Examples (Percentages, Partnership and Share)


1. Akash scored 73 marks in subject A. He scored 56% marks in subject B and X marks in subject C. Maximum marks in each subject were 150. The overall percentage marks obtained by Akash in all the three subjects together were 54%. How many marks did he score in subject C? (IBPS CWE PO MT 2012) (1) 84 (2) 86 (3) 79 (4) 73 (5) None of these Solution: Akash scored in subject A = 73 marks Subject B = = 84 marks Total marks Akash got in all the three subjects together = 54 4.5 = 243 marks Let Akash's marks in subject C be X. A + B + C = 243 or, A + B + X = 243 or, X = 243- (84 +73) = 243 -157= 86 marks Ans: (2) 2. An HR Company employs 4800 persons, out of which 45 per cent are males and 60 per cent of the males are either 25 years or older. How many males are employed in that HR Company who are younger than 25 years? (1) 2640 (2) 2160 (3) 1296 (4) 864 (5) None of these Solution: Total number of persons = 4800 95 IBPS PO EXAM 2013 : Quantitative Aptitude

Number of males = 45% of 4800 =

= 2160

Now, according to the question, Number of males who are younger than 25 years = (100-60 =) 40% of 2160 = 864

Ans: (4) 3. Six-elevenths of a number is equal to 22 per cent of the second number. The second number is equal to one- fourth of the third number. The value of the third number is 2400. What is 45% of the first number? (1) 109.8 (2) 111.7 (3) 117.6 (4) 123.4 (5) None of these Solution: According to the question, x First number = 22% of second number Second number = x Third number or, Second number = x 2400 = 600 or, First number = = 242 required answer = 45% of 242 = 96 = 108.9

IBPS PO EXAM 2013 : Quantitative Aptitude

Ans: (5) 4. In an entrance examination, Ritu scored 56 per cent marks, Smita scored 92 per cent marks and Rina scored 634 marks. The maximum marks of the examination is 875. What is the average marks scored by all the three girls together? (1) 1929 (2) 815 (3) 690 (4) 643 (5) None of these Solution: Ritu's marks = 875 x Smita's, marks = 875 x Rina's marks = 634 Total marks = 490 + 80S + 634 = 1929 Average = Ans: (4) 5. If twenty five per cent of three-sevenths of twenty six per cent of a number is 136.5, what is the number? (IBPS RRB Group A Officers Exam 2012) (1) 6300 (2) 5600 (3) 4800 (4) 4900 (5) None of these Solution: Let the number be x. Then, = 643 = 805 = 490

Ans: (4) 97 IBPS PO EXAM 2013 : Quantitative Aptitude

6. Two-thirds of Ranjits monthly salary is equal to Ramans monthly salary. Ramans monthly salary id thirty per cent more than pawans monthly salary. Pawans monthly salary is Rs. 32000. What is Ranjits monthly salary? (IBPS RRB Group A Officers Exam 2012) (1) Rs. 64200 (2) Rs. 62500 (3) Rs. 64500 (4) Rs. 62400 (5) None of these Solution: Pawans monthly salary = Rs. 32,000 Ramans monthly salary Ranjits monthly salary Ans: (4) 7. In a class there are 60 students, out of whom 15 per cent are girls. Each girls monthly fee is Rs. 250 and each boys monthly fee is 34 per cent more than a girl. What is the total monthly fees of girls and boys together? (IBPS RRB Group A Officers Exam 2012) (1) Rs. 19335 (2) Rs. 18435 (3) Rs. 19345 (4) Rs. 19435 (5) None of these Solution: Number of girls Total monthly fee of girls = 250 9 = Rs. 2250 Number of boys = 60 9 = 51 Monthly fee of one boy Total monthly fees of boys = 51 335 = Rs. 17085 Sum = 17085 + 2250 = Rs. 19,235 Ans: (1)

98

IBPS PO EXAM 2013 : Quantitative Aptitude

8. Ravi scored 225 marks in a test and failed by 15 marks. If the passing percentage of the test is 25 per cent, what is the maximum marks of the test? (1) (2) (3) (4) (5) 860 840 920 960 None of these

Solution: Passing marks = 225 + 15 = 240 Maximum marks Ans: (4) 9. Ravi scored 225 marks in a test and failed by 15 marks. If the passing percentage of the test is 25 per cent, what is the maximum marks of the test? (1) (2) (3) (4) (5) 860 840 920 960 None of these

Solution: Passing marks = 225 + 15 = 240 Maximum marks Ans: (4) 10. In a school, there are 800 students out of whom 12 per cent are girls. Each boy's monthly fee is Rs. 220 and each girl's monthly fee is 25 per cent less than that of a boy. What is the total monthly fee of all the girls and boys together? (Corporation Bank PO 2011) (1) (2) (3) (4) (5) Rs. 1,72,020 Rs. 1 ,80,780 Rs. 1,70,720 Rs. 1,80,600 None of these

99

IBPS PO EXAM 2013 : Quantitative Aptitude

Solution: Total number of girls Total number of boys = 800 96 = 704 Each girls monthly fee Total monthly fee of girls and boys together = 96 165 + 704 220 = 15840 + 154880 = Rs. 170720 Ans: (3)

100

IBPS PO EXAM 2013 : Quantitative Aptitude

Practice Set-1 (Percentages, Partnership and Share)


1. The number of seats in an auditorium is increased by 25%. The price of a ticket is also increased by 12%. Then, the increase in revenue collection will be a) 40% b) 35% c) 45% d) 48% 2. Two numbers are 30% and 40% more than the third number, respectively. The first number is x % of the second. Then, x is equal to a) b) c) d) 3. The price of cooking oil has increased by 25%. The percentage of reduction that a family should effect in the use of cooking oil, so as not to increase the expenditure on this account is a) 15% b) 20% c) 25% d) 30% 4. In an examination, a student had to obtain 33% of the maximum marks 101 to pass. He got 125 marks and failed by 40 marks. The maximum marks were a) 500 b) 600 c) 800 d) 1000 5. In an office 40% of the staff is female, 40% of the female and 60% of the male voted for me. The percentage of votes I got was a) 24% b) 42% c) 50% d) 52% 6. If A's income is 50% less than that of B's, then B's income is what per cent more than that of A? a) 125% b) 100% c) 75% d) 50% 7. In an examination, 35% of the candidates failed in Mathematics and 25% in English. If 10% failed in both Mathematics and English, then how much per cent passed in both the subjects? a) 50% b) 55% c) 57% d) 60%

IBPS PO EXAM 2013 : Quantitative Aptitude

8. The price of sugar rise by 25%. If a family wants to keep their expenses on sugar the same as earlier, the family will have to decrease its consumption of sugar by a) 25% b) 20% c) 80% d) 75% 9. If the numerator of a fraction is increased by 20% and the denominator is decreased by 5%, the value of the new fraction becomes . The original fraction is a) b) c) d) 10. If the price of tea is increased by 20%, by how much per cent the consumption of tea be reduced, so that there is no increase in the expenditure on it? a) b) 20% c) d)

11. A number reduced by 25% becomes 225. What per cent should it be increased, so that it becomes 375? a) 25% b) 30% c) 35% d) 75% 12. 25% of the candidates who appeared in an examination failed to qualify and only 450 candidates qualified. The number of candidates, who appeared in the examination was a) 700 b) 600 c) 550 d) 500 13. A worker suffers a 20% cut in his wages. He may regain his original wages by obtaining a rise of a) 27.5% b) 25.0% c) 22.5% d) 20.0% 14. A certain company has 80 engineers. If the engineers constitute 40% of its workers, then the number of people employed in the company is a) 150 b) 800 c) 200 d) 3200

102

IBPS PO EXAM 2013 : Quantitative Aptitude

15. A saves 20% of his monthly salary. If his monthly expenditure is Rs. 6000. Then, his monthly saving is a) Rs. 1200 b) Rs. 4800

c) Rs. 1500 d) Rs. 1800

103

IBPS PO EXAM 2013 : Quantitative Aptitude

Practice Set-2 (Percentages, Partnership and Share)


1. A starts business with Rs. 7000 and after 5 months, B joined as a partner. After a year the profit is divided in the ratios 2: 3. The capital of B is (a) Rs 10000 (b) Rs 6500 (c) Rs 18000 (d) Rs 9000 2. A, Band C started a business by investing Rs 40500, Rs 45000 and Rs 60000, respectively. After 6 months C withdrew Rs 15000 while A invested Rs 4500 more. In annual profit of Rs 56100 the share of C will exceed that of A by (a) Rs 900 (b) Rs 1100 (c) Rs 3000 (d) Rs 3900 3. A, B and C entered into partnership in a business. A got of the profit and B and C 6. A began a business with Rs 10500 and is joined afterwards by B with Rs 18000. After how many months did B join, if the profit at the end of year is divided equally? (a) 5 months (b) 15 months (c) 10 months IBPS PO EXAM 2013 : Quantitative Aptitude (d) Rs 800 4. A began a business with Rs 2250 and was joined afterwards by B with Rs 2700. If the profits at the end of the year were divided in the ratio of 2: 1, after how much time B joined the business? (a) 5 months (b) 6 months (c) 3 months (d) 7 months 5. Amit and Brijesh started a business with initial investments in the ratio of 12: 11 and their annual profits were in the ratio of 4: 1. If Amit invested the money for 11 months, then for what time Brijesh invested the money? (a) 9 months (b) 3 months (c) 5 months (d) 10 months distributed the remaining profit equally. If C got Rs 400 less than A, the total profit was (a) Rs 1600 (b) Rs 1200 (c) Rs 1000

104

(d) 9 months 7. A and B started a business by investing Rs 35000 and Rs 20000, respectively. After 5 months B left the business and C joined the business with a sum of Rs 15000. The profit earned at the end of year is Rs 84125. What is the share of B in profit?

(a) Rs 14133 (b) Rs 15000 (c) Rs 13460 (d) Cannot be determined

105

IBPS PO EXAM 2013 : Quantitative Aptitude

Percentages, Partnership and Share Practice Set-1 (Answers)

1) 2) 3) 4) 5) 6) 7) 8)

a d b a d b a b

9) 10) 11) 12) 13) 14) 15)

c c a b b c c

106

IBPS PO EXAM 2013 : Quantitative Aptitude

Percentages, Partnership and Share Practice Set-2 (Answers)

1) 2) 3) 4) 5) 6) 7)

c d c d b a c

107

IBPS PO EXAM 2013 : Quantitative Aptitude

Chapter: Profit & Loss


Profit and Loss is an extension of the chapter of percentages. It is a very important branch of basic Mathematics. This branch deals with the study of Profit and loss made in any commercial transaction. The entire economy and the concept of capitalism is based on the so called Profit Motive. Some basic terms used in Profit and loss are: Cost price The price, at which an article is purchased, is called Cost price and it is abbreviated by C.P. Selling Price The price, at which an article is sold, is called its selling price and it is abbreviated by S.P. Profit If S.P. > C.P., then seller is said to have a profit. Loss If SP < CP, Then seller is said to have incurred a loss. Formulae Profit or Gain = S.P. C.P. Loss = C.P. S.P. Gain % =

Loss % =

S.P. =

C.P.

S.P. =

C.P.

C.P =

S.P.

108

IBPS PO EXAM 2013 : Quantitative Aptitude

C.P =

S.P.

Example: 100 apples are bought at the rate of Rs. 500 and sold at the rate of Rs. 84 per dozen. What will be the percentage of profit and loss? Solution: We will solve this in steps Step I: Given that C.P. of 100 apples = 500 Then, C.P. of 1 apple = =5 Step II: Also given that per dozen S.P. of apples = 84 Then, S.P. of 1 apple = =7 Step III: Now, we know that Gain % = = = = = 40% Therefore, there is a profit of 40% in the whole selling process. If a person sells two similar items, one at a gain of A%, and the other at a loss of A%, then the seller always incurs a loss. This loss can be calculated by:

Loss % =

109

IBPS PO EXAM 2013 : Quantitative Aptitude

Example : A man sold two plots for Rs. 15, 00,000 each. On one he gains 25% while on the other he loses 25%. How much does he gain or loss in the whole transaction. Solution: In such a case there is always a loss
Loss % =

=6.25%.

If an article sold at two different selling price . On one is made and on the other is made then:

= Profit calculation on the basis of equating the Amount Spent and the Amount Earned: If the person is going through the transaction has got back all the money that he has spent, but has ended up with some amount of goods left over after the transaction. % Profit = 100

Example: A fruit vendor recovers the cost of 15 oranges by selling 10 oranges. Find his percentage profit. Solution: Here the money spent is equal to the money earned the percentage profit is given by % Profit = Discount: Discount is the reduction offered amount on the market price. Therefore, SP = Where, MP = Market Price of the product 110 IBPS PO EXAM 2013 : Quantitative Aptitude MP 100 = 5 100/10 =50%.

d = discount in percentage on the market price Equivalent Single discount for successive discounts a% and b% = (a+b )%

Example: What are the successive discounts of 10 %, 12 % and 15% amount to a single discount? Solution: Suppose the marked price = Rs. 100 The, S.P. = 85% of 88% of 90% of Rs. 100 = Rs. 67.32. Therefore, the single discount = (100 67.32) % = 32.68 %.

111

IBPS PO EXAM 2013 : Quantitative Aptitude

Solved Examples (Profit & Loss)


1. An article was purchased for Rs. 78,350/-. Its price was marked up by 30%. It was sold at a discount of 20% on the marked up price. What was the profit percent on the cost price? (IBPS CWE PO MT 2012) (1) 4 (2) 7 (3) 5 (4) 3 (5) 6 Solution: Cost price = Rs. 78350 Marked price = 78350 Selling price = 101855 = Rs. 101855 = Rs. 81484

Profit = 81484 -78350 = 3134 Reqd % profit = Ans: (1) 2. The cost of 8 kg of almonds is equal to the cost of 50 kg of apples. The cost of 19 kg of mangoes is Rs. 456. The cost of 1 kg of apples is twice the cost of 2 kg of mangoes. What is the total cost of 3 kg of almonds and 4 kg of apples together? (1) Rs. 2,168 (2) Rs. 2,248 (3) Rs. 2,184 (4) Rs. 2,264 (5) None of these Solution: Cost of 1 kg of mangoes Cost of 1 kg of apples = 2 48 = Rs. 96 Cost of 1 kg of almonds 112 IBPS PO EXAM 2013 : Quantitative Aptitude 100 = 4%

Cost of 3 kg of almonds and 4 kg of apples = 3 600 + 4 96 = Rs. 2184 Ans: (3) 3. Meena purchases 1500 ml of milk every day. If the cost of one liter of milk is Rs. 44, how much amount will she pay in 20 days? (Corporation Bank PO 2011) (1) (2) (3) (4) (5) Rs. 1,340 Rs. 1,320 Rs. 1,280 Rs. 1,260 None of these

Solution: Amount paid in 20 days = 1.5 20 44= Rs. 1320 Ans: (2) 4. Kamya purchased an item of Rs. 46,000 and sold it at loss of 12 per cent. With that amount she purchased another item and sold it at a gain of 12 per cent. What was her overall gain/loss? (Allahabad Bank Probationary Officers Exam 2011) (1) (2) (3) (4) (5) Loss of Rs 662.40 Profit of Rs 662.40 Loss of Rs 642.80 Profit Rs 642.80 None of these = Rs 40480 = Rs 45337.6

Solution: First S.P. = Second S.P =

Loss = Rs (46000-45337.6) = RS 662.4 Ans: (1) 5. A grain dealer cheats to the extent of 10% while buying as well as selling by using false weights. His total profit percentage is (1) (2) 21% 23% 113 IBPS PO EXAM 2013 : Quantitative Aptitude

(3) (4)

25% 20%

Ans: (1) Solution: Here, x = 10%, y = 10% Total profit percentage = = 21%

6. When the price of sugar decreases by 10%, a man could buy 1 kg more for Rs 270. Then, the original price of sugar per kg is (1) (2) (3) (4) Rs 25 Rs 30 Rs 27 Rs 32

Ans: (2) Solution: Let original CP of sugar = Rs x per kg New CP of sugar = X x = Rs

According to the give condition,

=1 =1 =1 = Rs 30 per kg

x=

114

IBPS PO EXAM 2013 : Quantitative Aptitude

7. For a certain article, if discount is 25% then profit is 25%. If the discount is 10% then the profit is (1) 50% (2) 40% (3) 30% (4) %

Ans: (1) Solution: Suppose cost price of the article = Rs x Then, X


x =

= Rs 510 = Rs

If discount is 10% then profit = = 150

Profit price of the gift item 50%

115

IBPS PO EXAM 2013 : Quantitative Aptitude

8. A man bought orange at the rate of 8 for Rs 34 and sold them at the rate of 12 for Rs 57. How many orange should be sold to earn a net profit of RS 45? (1) (2) (3) (4) 90 100 135 150

Ans: (1) Solution: Cost price of one orange = and selling price of one orange = Profit of one orange = = = =

On selling one orange, we get Rs On getting profit of Rs 45, we well = 45 2 = 90 orange

116

IBPS PO EXAM 2013 : Quantitative Aptitude

Practice Set (Profit & Loss)


1. A man makes a profit of 20% on the sale by selling 20 articles for Rs 1, the number of articles he bought by Rs 1 is (a) 20 (b) 24 (c) 25 (d) 30 2. A man buys one table and one chair for Rs 500. He sells the table at a loss of 10% and the chair at a gain of 10%. He still gains Rs 10 on the whole. The cost price of the chair is (a) Rs 300 (b) Rs 350 (c) Rs 200 (d) Rs 250 3. What single discount is equivalent to two successive discounts of 20% and 15%? (a) 35% (b) 32% (c) 34% (d) 30% 4. If the selling price of 10 articles is equal to the cost price of 11 articles, then the gain per cent is (a) 10% 117 IBPS PO EXAM 2013 : Quantitative Aptitude (c) loss 8 % (d) gain 8 % 6. A shopkeeper makes a profit of 20% even after giving a discount of 10% on the marked price of an article. If marked price is Rs 500, then the cost price of the article is (a) Rs 350 (b) Rs 375 (c) Rs 425 (d) Rs 475 (b) 11% (c) 15% (d) 25% 5. Krishna purchased a number of articles at Rs 10 for each and the same number for Rs 14 each. He mixed them together and sold them for Rs 13 each. Then, his gain or loss per cent is (a) loss 8 % (b) gain 8 %

7. A fruit seller bought 240 bananas at the rate of Rs 48 per dozen. He sells of them

equals the cost price of the article. The cost price of the article is (a) Rs 90

at the rate of Rs 5 per banana. th of the remaining are 6 found to be rotten. The price per banana at which he has to sell the remaining bananas to get a profit of 25% on his entire investment is (a) Rs 5.5 (b) Rs 6.0 (c) Rs 5.0

(b) Rs 8O (c) Rs 75 (d) Rs 60 11. When the price of cloth was reduced by 25%, the quantity of cloth sold increased by 20%. What was the effect on gross receipt of the shop? (a) 5% increase

(d) Rs 6.5 (b) 5% decrease 8. The difference between the selling price and cost price of an article is Rs 210. If the profit per cent is 25, then the selling price of the article is (a) Rs 950 (b) Rs 1050 (c) Rs 1150 (d) Rs 1250 9. A shopkeeper allows 23% commission on his advertised price and still makes a profit of 10%. If he gains Rs 56 on one item, his advertised price of the item, is (a) Rs 820 (b) Rs 780 (c) Rs 790 (d) Rs 8OO 10. By selling an article for Rs 144, a person gained such that the percentage gain 118 IBPS PO EXAM 2013 : Quantitative Aptitude (c) 10% increase (d) 10% decrease 12. If an article is sold at 200% profit, then the ratio of its cost price to its selling price will be (a) 1:2 (b) 2:1 (c) 1:3 (d) 3:1

13. If an electricity bill is paid before due date one gets a reduction of 4% on the amount of the bill. By paying the bill before due date a person got a reduction of Rs 13. The amount of his electricity bill was (a) Rs 125 (b) Rs 225 (c) Rs 325 (d) Rs 425 14. A shopkeeper marks an article at (. 60 and sells it at a discount of 15%. He also gives a gift worth Rs 3. If he still makes 20% profit, the cost price, (in Rs) is (a) 22 (b) 32 (c) 40 (d) 42 15. A shopkeeper earns a profit of 12% on selling a book at 10% discount on the printed price. The ratio of the cost price and the printed price of the book is (a) 45:56 (b) 45:51 (c) 47: 56 (d) 47: 51 16. A reduction of 10% in the price of sugar enables a housewife to buy 6.2 kg more for Rs 1116. The reduced price per kg is (a) Rs 12 119

(b) Rs 14 (c) Rs 16 (d) Rs 18 17. A man buys a certain number of oranges at 20 for Rs 60 and an equal number at 30 for Rs 60. He mixes them and sells them at 25 for Rs 60. What is gain or loss per cent (a) Gain of 4% (b) Loss of 4% (c) Neither gain nor loss (d) Loss of 5% 18. The marked price of a shirt and trousers are in the ratio 1:2. The shopkeeper gives 40% discount on the shirt. If the total discount on the set of the shirt and trousers is 30%, the discount offered on the trousers is (a) 15% (b) 20% (c) 25% (d) 30%

IBPS PO EXAM 2013 : Quantitative Aptitude

19. The percentage of loss when an article is sold at Rs 50 is the same as that of the profit when it is sold at Rs 70. The above mentioned percentage of profit or loss on the article is (a) 10% (b) 16 % (c) 20% (d) 22 % 20. I purchased 120 exercise books at the rate of Rs 3 each and sold of them at the rate of Rs 4 each, of them at the rate of Rs 5 each and the rest at the cost price. My profit per cent was (a) 44 % (b) 44 % (c) 44 % (d) 45%

120

IBPS PO EXAM 2013 : Quantitative Aptitude

Profit & Loss Practice Set (Answers)

1) 2) 3) 4) 5) 6) 7) 8) 9) 10)

b a b a d b b b d b

11) 12) 13) 14) 15) 16) 17) 18) 19) 20)

d c c c a d b c b b

121

IBPS PO EXAM 2013 : Quantitative Aptitude

Chapter: Simple Interest & Compound Interest


Definition: If a person X borrows some money from another person Y for a certain period, then after that specified period, X (borrower) has to return the borrowed money with some additional money. This additional money that X (borrower) has to pay is called Interest. The actual borrowed money is called Principal or Sum. The Principle and interest together is called amount, and the time for which X the borrower has been used the borrowed money is called the time. The interest that X has to pay for every 100 rupees each year is called rate percent per annum. If the interest on a sum borrowed for a certain period is reckoned uniformly, then it is called Simple Interest and it is denoted by S.I. FORMULAE: Let Principle = P, Rate =R% per annum, and Time = T years. Then S.I. = ( Or P= or R= or T= Now, Simple Interest + Principle = Amount If we denote the amount by A, then ) )

122

IBPS PO EXAM 2013 : Quantitative Aptitude

Simple Interest = A P S.I. = A P = A=P (1+ ) = SI (1+ )

Two different cases can be compared by using the following formula = Example: What will be the simple interest on Rs. 78,000 at 10% per annum for 9 years? Solution: Here, given that Principal (P) =78,000 Rate (R) = 10% Time (T) = 9 years Now, we know that S.I. = ( S.I. = ( S.I. = Rs. 70,200 Therefore, the simple interest on Rs. 78,000 at 10% per annum for 9 years will be Rs. 70, 200.

123

IBPS PO EXAM 2013 : Quantitative Aptitude

Compound Interest When the borrower X and the lender Y agrees to fix up a certain time for example yearly, half yearly or quarterly to settle the previous money, then the difference between the amount and the money borrowed is said to be the Compound Interest and it denoted by C.I. In these calculations, principal for the second unit of time is the amount of first unit of time and so on. Some important facts and formulae Let Principal = P, Time = n years (i) If interest is compounded annually, then Amount = P (ii) If interest is compounded half- yearly, then Amount = P (iii) If interest is compounded Quarterly, then Amount = P (iv) If time is in fractions and the interest is compounded yearly, say 2 then

Amount = P

(1+

Example: What will be the Compound Interest on Rs. 5000 at 5% per annum for 3 years, compounded annually? Solution: Amount = Rs. [5,000 ] = Rs. [5,000 = Rs.5788.125 ] = Rs. [5,000 ]

Compound Interest = Rs. (5788.125 5000) = Rs.788.125.

124

IBPS PO EXAM 2013 : Quantitative Aptitude

Example: What is the compound interest on Rs. 12000 in 4 years at 20 % per annum, the interest being compounded half yearly. Solution: Given that, Principal = Rs. 12000, Rate = 20% per annum, Time = 4 years

Now, Amount = Rs.[ 12000

= Rs. (120 11 11) = Rs. 14520

Compound Interest = Rs.( 14520 12000) = Rs. 2520.

125

IBPS PO EXAM 2013 : Quantitative Aptitude

Solved Examples (Simple Interest & Compound Interest)


1. What is the difference between the simple and compound interest on Rs. 7,300/- at the rate of 6 p.c.p.a. in 2 years? (IBPS CWE PO MT 2012) (1) Rs. 29.37 (2) Rs. 26.28 (3) Rs. 31.41 (4) Rs. 23.22 (5) Rs. 21.34 Solution: SI = CI = 7300 [(1 7300 [( = = 876 )2 - 1] ) = 902.28

)2= -1] = 7300 [( )] = 7300

Difference = 902.28 876 = 26.28 Quick Method: CI = - (6 + 6)

= 12.36 -12 = 0.36% = 0.36 per cent of 7300 = 26.28 Ans: (2) 2. The simple interest accrued on an amount of Rs. 22,500 at the end of four years is Rs. 10,800. What would be the compound interest accrued on the same amount at the same rate of interest at the end of two years? (1) 16,908 (2) 5,724 (3) 28,224 (4) 8,586 (5) None of these 126 IBPS PO EXAM 2013 : Quantitative Aptitude

Solution: r = CI= 22500 = 22500 Ans: (2)


2

= 12% 22500

- 22500 = 28224 22500 = 5724

3. The simple interest accrued on a sum of a certain principal is Rs. 35,6727 in seven years at the rate of 8 pcpa. What would be the compound interest accrued on that principal at the rate of 2 pcpa in 2 years? (A) Rs. 2573.48 (B) Rs. 2564.86 (C) Rs. 2753.86 (D) Rs. 2654.48 (E) None of these Solution. Principal

CI Quicker Method:

Ans: (1)

127

IBPS PO EXAM 2013 : Quantitative Aptitude

4. What will be the simple interest on Rs. 78,000 at 10% per annum for 9 years? Solution: Here, given that Principal (P) =78,000 Rate (R) = 10% Time (T) = 9 years Now, we know that S.I. = ( S.I. = ( S.I. = Rs. 70,200 Therefore, the simple interest on Rs. 78,000 at 10% per annum for 9 years will be Rs. 70, 200 5. What will be the simple interest earned on an amount of Rs. 18,000 in 6 months at the rate of 25% p.a.? (a) Rs.2250.50 (b) Rs.2350.50 (c) Rs.2, 250 (d) Rs 2,400 Solution : P= 18,000, R = 25%, T = 6/12 year S.I. = = = Rs.2, 250.

6. What will be the simple interest on Rs. 2,400 at 4 % per annum for the period from 1st Feb, 2005 to 15th April, 2005? (a) Rs. 20 (b) Rs.20.5 (c) Rs. 22 (d) Rs.25 Solution: Principal (P) = Rs. 2,400, Rate (R) = = year 128 IBPS PO EXAM 2013 : Quantitative Aptitude , Time (T) = (27+31+15) days =73 days = year

Now, S.I. = (

S.I. = (2,400

S.I. = 20.

Therefore, the simple interest on Rs. 2,400 at 4 % per annum for the period from 1 st Feb, 2005 to 15th April, 2005 will be Rs. 20. NOTE: The day on which money is withdrawn is counted while the day on which money is deposited is not counted.

7. A sum of Rs. 600 amounts to Rs. 900 in 5 years at simple interest. What would be the amount if the interest rate is increased by 5%? (a) Rs.1, 000 (b) Rs 1,230 (c) Rs.1, 050 (d) Rs.1, 125 Solution : According to the given situation a sum of Rs. 600 amounts to Rs. 950 in 5 years, then S.I. = Rs. 900 Rs.600 = Rs.300 P = Rs. 600, T = 5 Years Therefore, R= R= % = 10%

If the interest rate is increased by 10 %, then New Rate = (10+5) % =15%, New S.I. =Rs. Then, New Amount = Rs.600 + Rs.450 = Rs.1050 8. What will be the compound interest on a sum of Rs 8000 at the rate of 15% per annum after 2 years? (a) Rs.2400 (b) Rs.2450 (c) Rs.2580 (d) Rs.2650 129 IBPS PO EXAM 2013 : Quantitative Aptitude = Rs.450

Solution : Amount = Rs. [8000 Rs. [8000 = Rs. 10,580

C.I. = Rs. [10580 8000] = Rs. 2,580 9. What is the difference between the compound interests on Rs. 10000 for 2 years at 4%pe annum yearly and half yearly? (a) Rs.8 (b) Rs.8.49 (c) Rs.7 (d) Rs. 10 Solution: C.I. , When interest is compounded yearly

Amount = Amount = = Rs.11,032.32

C.I. = 11032.32 10000 = 1032.32 C.I. When interest is compounded half yearly Amount = Rs [10000 ] = Rs. 11040.80803

C.I. = 11040.80803 10000 = 1040.808032 Diff = 8.488032 Rs. 8.9

130

IBPS PO EXAM 2013 : Quantitative Aptitude

10. A sum of money tripled itself at compound interest in 10 years. In how many years will it become 27 times. (a) (b) (c) (d) 35 years 31 years 30 years 32 years = 3P

Solution : P

=3 Suppose after n years it will become 27 times Then, P = = 27 p = n = 30.

131

IBPS PO EXAM 2013 : Quantitative Aptitude

Practice Set (Simple Interest & Compound Interest)


1. A sum of money placed at compound interest doubles itself in 4 yr. In how many years will it amount to four times itself? a) 12 yr b) 13 yr c) 8 yr d) 16 yr 2. The difference between the compound interest and simple interest on Rs. 10000 for 2 yr is Rs. 25. The rate of interest per annum is a) 5% b) 7% c) 10% d) 12% 3. The simple interest on a sum of money is of the principal and d) 4. Ratio of the principal and the amount after 1 yr is 10 : 12. Then, the rate of interest per annum is a) 12% b) 16% c) 18% d) 20% 5. At some rate of simple interest, A lent Rs. 6000 to B for 2 yr and Rs. 1500 to C for 4 yr and received Rs. 900 as interest from both of them together. The rate of interest per annum was a) 5% b) 6% c) 8% d) 10% 6. What annual payment will discharge a debt of Rs. 6450 due in 4 yr at 5% per annum simple interest? a) Rs. 1400 b) Rs. 1500 c) Rs. 1550 c) 132 d) Rs. 1600 IBPS PO EXAM 2013 : Quantitative Aptitude

the number of years is equal to the rate per cent per annum. The rate per cent per annum is equal to a) 3% b)

7. In how many years will a sum of money double itself at interest per annum? a) 24 yr b) 20 yr c) 16 yr d) 12 yr 8. In how many years will a sum on Rs. 800 at 10% per annum compound interest, compounded semi-annually becomes Rs. 926.10? a) simple

a) b) c) d) 11. In what time Rs. 8000 will amount to Rs. 9261 at 10% per annum compound interest, when the interest is compounded half-yearly? a) b)

b) c) c) d) d) 9. A certain sum amount to Rs. 5832 in 2 yr at 8% per annum compound interest, then the sum is a) Rs. 5000 b) Rs. 5200 c) Rs. 5280 d) Rs. 5400 10. In what time will Rs. 10000 amount to Rs. 13310 at 20% per annum compounded half-yearly? c) d) 12. Simple interest on Rs. 500 for 4 yr at 6.25% per annum is equal to the simple interest on Rs. 400 at 5% per annum for a certain period of time. The period of time is a) 4 yr b) 5 yr

133

IBPS PO EXAM 2013 : Quantitative Aptitude

13. If the difference between the compound and simple interests on a certain sum of money for 3 yr at 5% per annum is Rs. 15.25, then the sum is a) Rs. 2000 b) Rs. 1000 c) Rs. 1500 d) Rs. 2500 14. If the simple interest for 6 yr be equal to 30% of the principal, it will be equal to the principal after a) 20 yr b) 30 yr c) 10 yr d) 22 yr 15. The effective annual rate of interest, corresponding to a nominal rate of 6% per annum payable half-yearly is a) 6.06% b) 6.07% c) 6.08% d) 6.09% 16. What annual instalment will discharge a debt of Rs. 6450 due in 4 yr at 5% simple interest? a) Rs. 1500 b) Rs. 1835 c) Rs. 1935 134

d) Rs. 1950 17. At what rate per cent per annum will the simple interest on a sum of money be of the amount in 10 yr? a) 4% b) 6% c) d) 18. The difference between the simple and compound interest on a certain sum of money for 2 yr at 4% per annum is Rs. 4. The sum is a) Rs. 2500 b) Rs. 2400 c) Rs. 2600 d) Rs. 2000 19. The compound interest on a certain sum of money at a certain rate for 2 yr is Rs. 40.80 and the simple interest on the same sum is Rs. 40 at the same rate and for the same time. The rate of interest per annum is a) 2% b) 3% c) 4% d) 5%

IBPS PO EXAM 2013 : Quantitative Aptitude

20. A sum of money becomes eight times of itself in 3 yr at compound interest. The rate of interest per annum is a) 100% b) 80% c) 20% d) 10%

135

IBPS PO EXAM 2013 : Quantitative Aptitude

Simple Interest & Compound Interest Practice Set (Answers)


1) 2) 3) 4) 5) 6) 7) 8) 9) 10) c a d d a b c a a a 11) 12) 13) 14) 15) 16) 17) 18) 19) 20) b c a a d a d a c a

136

IBPS PO EXAM 2013 : Quantitative Aptitude

Chapter: Time and Work


Introduction: Pipes and cisterns problems are similar to time and work problem, the only difference is that pipes and cisterns problems have outlets and inlets . 1. Inlet is a pipe connected with a tank which fills it 2. Outlet is a pipe connected with a tank to empty it. Mathematical use :1. If pipe can fill (or empty) a tank in x hours, then the part filled (or emptied) in 1 hour = . 2. If a pipe P fills a tank in x hours and another pipe Q empties the full tank in y hours and if both the pipes are opened then the net part filled in 1 hour = ( ).

Example 1: How much time will it take to fill the tank if a pipe A fill it in 30 hours and another pipe B empties it in 40 hours? Sol 1: Net part filled in 1 hour = Hence the time taken to fill the tank = 12 hours. Direct method:The time taken to fill the tank = .

3. If two pipes fills the tank in x and y hours respectively then the net part filled in 1 hours , when both the pipes are opened = ( ).

137

IBPS PO EXAM 2013 : Quantitative Aptitude

Example 2: How much time will it take to fill the tank if two pipes A and B fill it in 20 hours and 30 hours respectively and both the pipes are opened? Sol 2: Net part filled by both pipes A and B together in 1 hour = Hence the time taken to fill the tank = 12 hours. Direct method:The time taken to fill the tank = .

4. If two pipes fills the tank in x and y hours respectively and a third pipe emptied the full tank in z hours , then the net part filled in 1 hours , when all the pipes are opened = ( ).

Example3: How much time will it take to fill the tank if two pipes A and B fill it in 10 hours and 20 hours respectively and a third pipe C empties it in 40 hours? Sol 3: Net part filled in 1 hour = Hence the time taken to fill the tank = 8 hours. Direct method:The time taken to fill the tank = 5. If a pipe fills the tank in x hours but due to leakage in bottom it is filled in y hours , then the time taken by leak to empty the tank if the tank is full = .

138

IBPS PO EXAM 2013 : Quantitative Aptitude

Example 4: How much time will the leak take to empty the tank if a pipe A fill it in 10 hours but due to leak in the bottom it is filled in 15 hours ? Sol 4: Tank empty due to leak in 1 hour = ( Hence the leak will empty the full tank in 30 hours Direct method:The required time =

139

IBPS PO EXAM 2013 : Quantitative Aptitude

Solved Examples (Time and Work)


1. A and B together can complete a task in 20 days. Band together can complete the same task in 30 days. A and C together can complete the same task in 40 days. What is the respective ratio of the number of days taken by A when completing the same task alone to the number of days taken by C when completing the same task alone? (IBPS CWE PO MT 2012) (1) 2: 5 (2) 2: 7 (3) 3: 7 (4) 1: 5 (5) 3: 5 Solution: A and B can finish the work in 20 days. A and Bs one days work = B and C and finish the work in 30 days. B and Cs one days work = A and C can finish the work in 40 days A and Cs one day's work = Adding we get 2(A + B + C)s one days work = + + = =

(A + B + C)s one days work (A + B + C)s one days work = = -

As one days work = = = =

A alone can finish the work in 48 days.

140

IBPS PO EXAM 2013 : Quantitative Aptitude

Cs one day work =

C alone can finish the work in 240 days. Reqd ratio = Ans: (4) 2. Seven girls can do a piece of work in 13 days, six boys can do the same piece of work in 12 days, nine men can do the same piece of work in nine days and six women can do the same piece of work in 14 days. Who are the most efficient? (Corporation Bank PO 2011) (1) (2) (3) (4) (5) Boys Girls Women Men Both men and women =1:5

Solution: One girl can complete the work in 7 13 = 91 days. One boy can complete the work in 6 12 = 72 days. One man can complete the work in 9 9 = 81 days. One woman can complete the work in 6 14 = 84 days. Hence, boys are the most efficient. Ans: (1) 3. A and B together can complete a work in 12 days. A alone can complete in 20 days. If B does the work only half a day daily, then in how many days A and B together will complete the work. (1) (2) (3) (4) 10 days 20 days 11 days 15 days

Ans: (4) Solution: (A + B)s 1 days work =

141

IBPS PO EXAM 2013 : Quantitative Aptitude

As 1 day work = Bs 1 day work = = = B does the work only half day. Bs 1 days work = = + = = =

Now, A+B)s day work = =

Hence, A and B together will complete the work in 15 days. 4. A can do a piece of work in 70 days and B is 40% more efficient than A. The number of days taken by B to do same work is (1) 3 (2) (3) 5 (4) Ans: (3) Solution: A can do a piece of work in 70 days while B us 40% more efficient than A. B do the same work in

142

IBPS PO EXAM 2013 : Quantitative Aptitude

= 70 = 70 = 50 days

5. A and B working separately can do a piece of work in 9 and 12 days, respectively. If they work for a day alternately with A beginning, the work would be completed in (1) 28 days (2) 14 days (3) 5 days (4) Ans: (3) Solution: One day work of A = One day work of B = Two days work of (A+B) = + = 10 days = 5 2 days work of (A + B) = Remaining work = 1Now, As turn. A, will complete the work in 9 = day = 5 = = days

Hence, the total time = 10 + = day

143

IBPS PO EXAM 2013 : Quantitative Aptitude

6. A and B can do a work in 72 days. B and C can do it in 120 days. The number of days needed for A to the work alone is (1) (2) (3) (4) 20 22 33 44

Ans: (3) Solution: Work done by (A+B) in 1 day = Work done by (B+C) in 1 day = Work done by (C+A) in 1 day =

Work done by 2 (A+B+C) in 1 day =


= Work done by (A+B+C) in 1 day = As 1 day work = = Hence, A do the work alone in 120 days. 7. A, B and C individually can do a work in 10 days, 12 days and 15 days, respectively. If they start working together, then the number of days required to finish the work is (1) (2) (3) (4) 16 days 8 days 4 days 2 days 144 IBPS PO EXAM 2013 : Quantitative Aptitude =

Ans: (3) Solution: Given, x = 10, y = 12 and z = 15 Required number of days =

= = 4 days 8. A cistern has two pipes. One can fill it with water in 8 h and other can empty it in 5 h. In how many hours will the cistern be emptied, if both the pipes are opened together when the cistern is already full of water? (1) h (2) 10 h (3) 6 h (4) h of

Ans: (b) Solution: Part of cistern emptied in 1 h = - = =

part is emptied in 1 h. part is emptied in = 10h

145

IBPS PO EXAM 2013 : Quantitative Aptitude

Practice Set (Time and Work)


1. Two men A and B are started a job in which A was thrice as good as B and therefore took 60 days less than B to finish the job. How many days will they take to finish the job, if they start working together? (a) 15 days (b) 20 days (c) 22 days (d) 25 days 2. A and B can separately do a piece of work in 20 and 15 days, respectively. They worked together for 6 days, after which B was replaced by C. The work as finished in next 4 days. The number of days in which C alone could do the work is (a) 30 days (b) 45 days (c) 40 days (d) 35 days 3. A can do a work in 12 days. When he had worked for 3 days, B joined him. If they complete the work in 3 more days, in how many days can B alone finish the work? (a) 6 days (b) 12 days 146 (a) 3 (d) 5 days 6. A and B can do a job alone in 12 days and 15 days, respectively. A starts the work and after 6 days B also joins to finish the work together. For how many days B actually worked on the job? (c) 4 days (d) 8 days 4. A and B can complete a piece of work in 8 days, B and C can do it in 12 days, C and A can do it in 8 days. A, Band C together can complete it in (a) 4 days (b) 5 days (c) 6 days (d) 7 days 5. X is 3 times as fast as Yand is able to complete the work in 40 days less than Y. Then, the time in which they can complete the work together is (a) 15 days (b) 10 days (c) 7 days

IBPS PO EXAM 2013 : Quantitative Aptitude

(b) 9 (c) 5 (d) 6 7. Two pipes can fill a cistern separately in 24 min and 40 min, respectively. A waste pipe can drain off 30 L/min. If all the there pipes are opened, the cistern fills in one hour. The capacity of the cistern is (a) 800 L (b) 400 L (c) 600 L (d) 500 L 8. A cistern is normally filled in 8 h but takes another 2 h longer to fill because of a leak in its bottom. If the cistern is full, the leak will empty it in (a) 16 h (b) 20 h (c) 25 h (d) 40 h 9. Two pipes, P and Q can fill a cistern in 12 and 15 min, respectively. Both are opened together but at the end of 3 min, P is turned off. In how many more minutes will Q fill the cistern? (a) 7min (b) 7 min 147

(c) 8 min (d) 8 min 10. Tapas works twice as fast as Mihir. If both of them together complete a work in 12 days. Tapas alone can complete it in (a) 15 days (b) 18 days (c) 20 days (d) 24 days 11. One man and one woman together can complete a piece of work in 8 days. A man alone can complete the work in 10 days. In how many days can one woman alone complete the work? (a) (b) 30 (c) 40 (d) 42 12. A and B together can do a work in 10 days. Band C together can do the same work in 6 days. A and C together can do the work in 12 days. Then, A, Band C together can do the work in (a) 28 days (b) 14 days (c) 5 days (d) 8 days IBPS PO EXAM 2013 : Quantitative Aptitude

13. A is thrice as good a workman as Band therefore is able to finish a job in 40 days less than B. Working together, they can do it in (a) 14 days (b) 13 days (c) 20 days (d) 15 days 14. 45 men can complete a work in 16 days. Four days after they started working, 36 more men joined them. How many days will they now take to complete the remaining work? (a) 6 days (b) 8 days (c) 6 day (d) 7 days 15. A can do a work in 5 days less than the time taken by B to do it. If both of them together take 11 days, then the time taken by B alone 9 to do the same work (in days) is (a) 15 (c) 25 (b) 20 (d) 30

16. A can complete of a work in 5 days and B, of the work in 10 days. In how many days both A and B together can complete the work? (a) 10 (b) 9 (c) 8 (d) 7 17. A boy and girl together fill a cistern with water. The boy pours 4 L of water every 3 min and the girl pours 3 L every 4 min. How much time will it take to fill 100 L of water in the cistern? (a) 36 min (b) 42 min (c) 48 min (d) 44 min 18. If 28 men complete of a piece of work in a week, then the number of men, who must be engaged to get the remaining work completed in another week, is (a) 5 (b) 6 (c) 4 (d) 3

148

IBPS PO EXAM 2013 : Quantitative Aptitude

19. While working 7 h a day, A alone can complete a piece of work in 6 days and B alone in 8 days. In what time would they complete it together, working 8 h a day? (a) 3 days (b) 4 days (c) 2.5 days (d) 3.6 days 20. A and B can complete a piece of work in 12 and 18 days, respectively. A begins to do the work and they work alternatively one at a time for one day each. The whole work will be completed in (a) 14 days

22. A pipe can empty a tank in 40 min. A second pipe with diameter twice much as that of the first is also attached with the tank to empty it. The two pipes together can empty the tank in (a) 8 min (b) 13 min (c) 30 min (d) 38 min 23. A and B can do a work in 45 days and 40 days, respectively. They began the work together but A left after sometime and B completed the remaining work in 23 days. After how many days of the start of the work did A leave? (a) 10 days

(b) 15 days (c) 16 (d) 18 days

(b) 9 days (c) 8 days (d) 5 days

days

21. A man and a boy received Rs 800 as wages for 5 days for the work they did together. The man's efficiency in the work was three times that of the boy. What are the daily wages of the boy? (a) Rs 76 (b) Rs 56 (c) Rs 44 (d) Rs 40 149

24. A and B working separately can do a piece of work in 10 days and 15 days, respectively. If they work on alternate days beginning with A, in how many days will the work be completed? (a) 18 days (b) 13 days (c) 12 days (d) 6 days

IBPS PO EXAM 2013 : Quantitative Aptitude

25. Two pipes can fill a tank separately in 20 min and 30 min, respectively. If both the pipes are opened simultaneously, then the tank will be filled in (a) 10 min (b) 12 min (c) 15 min (d) 25 min

150

IBPS PO EXAM 2013 : Quantitative Aptitude

Averages Practice Set (Time and Work)

1) 2) 3) 4) 5) 6) 7) 8) 9) 10) 11) 12) 13)

c c a c a a c d d b c c d

14) 15) 16) 17) 18) 19) 20) 21) 22) 23) 24) 25)

c c b c c a a d b b c b

151

IBPS PO EXAM 2013 : Quantitative Aptitude

Chapter: Speed Distance & Time


Speed, Distance and time is one of the most important chapter for the purpose of the maths section in aptitude exams. Time and Distance Formulae relates Time, Distance and Speed. These relationships have many practical applications. The basic concepts of Speed, Distance and time are used in solving questions based on relative motion, circular motion, problem based on trains, problem based on boats, races, etc. The questions asked in IBPS Bank PO is very diverse in nature, therefore this chapter is very important for Bank PO aspirants. For example if you know the speed of any vehicle and the distance covered by that vehicle, then we can easily calculate the time taken in whole journey by using the formula of Time and Distance. Important Formulae (i) Speed = (ii) S km/hr =(s (iii) S m/sec =(s Points to remember: 1. Read the units of time speed and distance carefully. 2. If the distance is given in km and the speed is in m/s then always convert the units. According to the demand of the question you can change the kilometer in to meter or m/s in to km /hr. Example1. A Chennai Express travelling at of its actual speed and covers 42 km in 1 hr 40 ,Distance= Speed Time, Time= )m/sec )km/hr

min 48 sec, find the actual speed of the Chennai Express. Solution: Suppose the actual speed of Chennai Express = S km/hr Then new speed = S

Time taken by Chennai Express with new speed = 1 hr 40 min 48 sec

152

IBPS PO EXAM 2013 : Quantitative Aptitude

= 1hr+40

hr+48

[Because 1hr = 60 minutes, 1 minute = 60 Second, & 1 minute = 1 Second = hr, minute

= 1hr + hr + = hrs

hr

Now, according to the formula New Speed Time taken by Chennai Express with new speed = Distance covered by Chennai Express S = 42

S= S =55km/hr Hence, the actual speed of the Chennai Express is 55 km/hr TRAINS, BOATS AND STREAMS IMPORTANT POINTS ON TRAINS: 1.) Time taken by a train x metres long to pass a single post or a pole or a standing man = Time taken by the train to cover x metres. 2.) Time taken by a train x metres long to pass a stationary object of length y metres = Time taken by the train to cover (x+y) meters. 3.) Suppose two trains or two bodies are moving in the same direction at u kmph and v kmph such that u > v, then their relative speed = (u-v) kmph.

153

IBPS PO EXAM 2013 : Quantitative Aptitude

4.) If 2 trains of lengths x km and y km are moving in the same direction at u kmph and v kmph , where u > v, then time taken by faster train to cross the slower train = hrs.

5.) Suppose two trains or two bodies are moving in opposite directions at u kmph and v kmph . Then their relative speed = (u+v) kmph 6.) If two trains of lengths x km and y km are moving in opposite directions at u kmph and v kmph , then time taken by the trains to cross each other = hrs.

7.) If two trains start at the same time from two points A and B towards each other and after crossing they take a and b hours in reaching B and A respectively. Then, As speed: Bs speed = (b: a). 8.) x kmph = 9.) y metre/sec = m/sec km/hr EXAMPLES Example 1: Find the time taken by a train 180 m long, running at 72 kmph, in crossing an electric pole. Solution: speed of the train = (72 5/18) m/sec = 20 m/sec Distance moved in passing the pole = 180m Required time taken = (180/20)sec = 9 sec. Example 2: Two trains 137m and 163m in lengths are running towards each other on parallel lines, one at the rate of 42 kmph and another at 48 kmph. In what time will they be clear of each other from the moment they meet? Solution: Relative speed of the trains = (42+48) kmph = 90 kmph = (905/18)m/sec= 25m/sec. Time taken by the trains to pass each other = Time taken to cover (137+163)m at 25m/sec = (300/25)sec = 12 sec. IMPORTANT POINT ON BOATS AND STREAMS: 1.) In water, the direction along the stream is called downstream and the direction against the stream is called upstream. 2.) If speed of a boat in still water is u km/hr and the speed of the stream is v km/hr, then Speed downstream = (u + v) km/hr Speed upstream = (u v) km/hr 3.) If the speed downstream is a km/hr and the speed upstream is b km/hr, then : 154 IBPS PO EXAM 2013 : Quantitative Aptitude

Speed in still water = (a + b)/2 km/hr Rate of stream = (a b)/2 km/hr EXAMPLES Example 1: A man can row upstream at 7 kmph and downstream at 10 kmph. Find mans rate in still water and the rate of current. Solution: Rate in still water = (10 + 7)/2 km/hr= 8.5 km/hr Rate of current = (10 7)/2 km/hr = 1.5km/hr Example 2: A man can row 8 kmph in still water and the river is running at 2 kmph. If the man takes 1 hour to row to a place and back, how far is the place? Solution: Mans rate downstream = (8+2) kmph = 10 kmph Mans rate upstream = (8-2) kmph = 6 kmph

155

IBPS PO EXAM 2013 : Quantitative Aptitude

Solved Examples (Speed Distance & Time)


1. A 476-metre-Iong moving train crosses a pole in 14 seconds. The length of a platform is equal to the distance covered by tile train in 20 seconds. A man crosses the same platform in 7 minutes and 5 seconds. What is the speed of the man in metre/second? (IBPS RRB Group A Officers Exam 2012) (1) 1.8 m/s (2) 1.4 m/s (3) 1.6 m/s (4) 2 m/s (5) 1.2 m/s Solution: Speed of train Length of platform = 34 20 = 680 metre. ( 7 minute 5 second = 7 60 + 5 = 425 second) Speed of man Ans: 3 2. Train A crosses a pole in 20 seconds and Train B crosses the same pole in one minute. The length of Train A is half the length of Train B. What is the ratio of the speed of Train A to that of Train B? (Corporation Bank PO 2011) (1) (2) (3) (4) (5) 3:2 3:4 4:3 Cannot be determined None of these

Solution: Let the length of Train B be 2x and that of Train A be x. Speed of Train A 156 IBPS PO EXAM 2013 : Quantitative Aptitude

Speed of Train B Ratio: Ans: 1 3. A 280 metre long train moving with an average speed of 108 km/hr crosses a platform in 12 seconds. A man crosses the same platform in 10 seconds. What is the speed of the man? (Allahabad Bank Probationary Officers Exam 2011) (1) (2) (3) (4) (5) 5 m/s 8 m/s 12 m/s Cannot be determined None of these m/sec

Solution: Speed of the train = 108 km/hr = If the length of the platform be x metres, then = 30

x + 280 = 360

x = (360-280 =) = 80 metres
mans speed = Ans: 2 4. A student goes to school at the rate of 2 km/h and reaches 6 min late. If he travels at the speed of 3 km/h, he is 10 min early. The distance (in km) between the school and his house is (1) (2) (3) (4) 5 4 3 1 = 8 m/sec

Ans: (2) 157 IBPS PO EXAM 2013 : Quantitative Aptitude

Solution: Let the distance between school and his house = x km According to the given condition. - =

- = = =

x = 4 km 5. The Speeds of two trains are in the ratio 6 : 7. If the second train runs 364 km in 4 h, then the speed of first train is (1) (2) (3) (4) 60 km/h 72 km/h 78 km/h 84 km/h

Ans: (3) Solution: Let the speeds of two rain be 6x km/h and 7x km/h, respectively. By given condition Speeds of second train = 7x = 91 x= = 13 km/h = 91 km/h.

Speed of first train = 13 6 = 78 km/h

158

IBPS PO EXAM 2013 : Quantitative Aptitude

6. If a man walks 3 km/h, he is late to his office by 20 min. If he increases his speed to 6 km/h, he reaches the office 30 min early. The distance of his office from the starting place is (1) (2) (3) (4) 6 km 5 km 5.5 km 4 km

Ans: (2) Solution: Let the distance of his office from the starting place is x km. By given condition, = + =

x =

= 5 km

7. In covering a distance of 30 km, Abhay takes 2 h more than Sameer. If Abhay doubles his speed, then he would take 1 h less than Sameer. Abhay's speed (in km/h) is (1) (2) (3) (4) 5 6 6.25 7.5

Ans: (1) Solution: Let the speeds of Abhay be x km/h and Sameer be y km/h respectively. Then, And = 2 ..(i) = 1 ..(i)

On adding Eqs. (i) and (ii), we get 159 IBPS PO EXAM 2013 : Quantitative Aptitude

=3

30 = 6x x = 5 km/h
8. A man completed a certain journey by a car. If he covered 30% of the distance at the speed of 20 km/h and the remaining distance at 10 km/h, his average speed for the whole journey was (1) (2) (3) (4) 25 km/h 28 km/h 30 km/h 33 km/h

Ans: (1) Solution: Let the total be 100 km. Average Speed = = =

= = 25 km/h 9. Two trains started at the time one from A to B and other from B to A. If they arrived at B and A, respectively 4 h and 9 h after they passed each other, the ratio of the speeds of the two trains was (1) (2) (3) (4) 2:1 3:2 4:3 5:4

Ans: (2) Solution: Required ratio of the speeds of two trains 160 IBPS PO EXAM 2013 : Quantitative Aptitude

10. A man takes 6 h 15 min in walking a distance and ridding to the starting place. He could walk both ways in 7h 45 min. The time taken by him to ride both ways, is (1) (2) (3) (4) 4h 4 h 30 min 4 h 45 min 5h

Ans: (3) Solution: Time taken in walking both ways = 7 h 45 min (i)

Time taken in walking one way and riding back = 6 h 15 min (ii)

On multiplying by 2 in Eq. (ii) and then subtracting Eq . (i) from Eq. (ii), we get Time taken by the man to ride both ways = 12 h 30 min 7h 45 min = 7 h 45 min

161

IBPS PO EXAM 2013 : Quantitative Aptitude

Practice Set (Speed Distance & Time)


1. A person travels 285 km in 6 h. In the first part of the journey, he travels at 40 km/h by bus. In the second part, he travels at 55 km/h by train. The distance travelled by train is (a) 165 km (b) 615 km (c) 561 km (d) 156 km 2. With average speed of 40 km/h, a train reaches its destination at time. If it goes with an average speed of 35 km/h, it is late by 15 min. The total journey is (a) 30 km (b) 40 km (c) 70 km (d) 80 km 3. If a train runs at 40 km/h, it reaches its destination late by 11 min but, if it runs at 50 km/h, it is late by 5 min only. The correct time (in min) for the train to complete the journey is (a) 13 (b) 15 (c) 19 (d) 21 (d) 38 km/h 4. Two trains, 80 m and 120 m long, are running at the speed of 25 km/h and 35 km/h, respectively in the same direction on parallel tracks. How many seconds will they take to pass each other? (a) 48 s (b) 64 s (c) 70 s (d) 72 s 5. A train, 300 m long, passed a man, walking along the line in the same direction at the rate of 3 km/h in 33 s. The speed of the train is (a) 30 km/h (b) 32 km/h (c) 32 km/h

6. By walking at of his usual speed, a man reaches his office 20 min later than his usual time. The usual time taken by him to reach his office is (a) 75 min (b) 60 min (c) 40 min

162

IBPS PO EXAM 2013 : Quantitative Aptitude

(d) 30 min 7. In a 100 m race, Kamal defeats Bimal by 5 s. If the speed of Kamal is 18 km/h, then the speed of Bimal is (a) 15.4 km/h (b) 14.5 km/h (c) 14.4 km/h (d) 14 km/h 8. A train, 240 m long, crosses a man walking along the line in opposite direction at the rate of 3 km/h in 10 s. The speed of the train is (a) 63 km/h (b) 75 km/h (c) 83.4 km/h (d) 86.4 km/h 9. A boy is late by 9 min, if he walks to school at a speed of 4 km/h. If he walks at the rate of 5 km/h he arrives 9 min early. The distance to his school is (a) 9 km (b) 5km (c) 4 km (d) 6 km 10. Two towns A and B are 500 km apart. A train starts at 8 am from A towards B at a speed of 70 km/h. At 10 am, another train starts from B towards A at a speed of 110 km/h. When will the two trains meet? (a) 1 pm 163

(b) 12 noon (c) 12: 30 pm (d) 1: 30 pm 11. A train 150 m long passes a pole in 15 s and another train of the same length travelling in the opposite direction in 12 s. The speed of the second train is (a) 45 km/h (b) 48 km/h (c) 52 km/h (d) 54 km/h 12. A, B and C start together from the same place to walk round a circular path of length 12 km. A walks at the rate of 4 km/h, B 3 km/h and C km/h. They will

meet together at the starting place at the end of (a) 10 h (b) 12 h (c) 15 h (d) 24 h 13. A train travelling at 48 km/h crosses another train, having half its length and travelling in opposite direction at 42 km/h in 12 s. It also passes a railway platform in 45 s. The length of the railway platform is (a) 200 m (b) 300 m (c) 350 m IBPS PO EXAM 2013 : Quantitative Aptitude

(d) 400 m 14. In a race of one kilometer, A gives B a start of 100 m and still wins by 20 s but, if A gives B a start of 25 s, B wins by 50 m. The time taken by A to run one kilo metre is (a) 17 s (b) (c) (d) s

(b) 27 km/h (c) 25 km/h (d) 24 km/h 17. A and B run a kilometre and A wins by 25 s. A and C run a kilometre and A wins by 275 m. When Band C run the same distance, B wins by 30 s. The time taken by A to run a kilometre is (a) 2 min 25 s

s s

(b) 2 min 50 s (c) 3 min 20 s (d) 3 min 30 s 18. A train passes a man standing on a platform in 8 s and also crosses the platform which is 264 m long in 20 s. The length of the train is (a) 188 m (b) 176 m (c) 175 m (d) 96 m

15. A student walks from his house at a speed of 2 km/h and reaches his school 6 min late. The next day he increases his speed by 1 km/h and reaches 6 min before school time. How far is the school from his house? (a) km (b) km (c) km (d) km

19. A walks at a uniform rate of 4 km/h and 4 h after his start, B bicycles after him at the uniform rate of 10 km/h. How far from the starting point will B catch A? (a) 16.7 km (b) 18.6 km

16. A train passes two persons walking in the same direction at a speed of 3 km/h and 5 km/h respectively, in 10 sand 11 s, respectively. The speed of the train is (a) 28 km/h 164

(c) 21.5 km (d) 26.7 km

IBPS PO EXAM 2013 : Quantitative Aptitude

20. A constable follows a thief who is 200 m ahead of the constable. If the constable and the thief run at speeds of 8 km/h and 7 km/h respectively, the constable would catch the thief in (a) 10 min (b) 12 min (c) 15 min (d) 20 min

165

IBPS PO EXAM 2013 : Quantitative Aptitude

Speed Distance & Time Practice Set (Answers)


1) 2) 3) 4) 5) 6) 7) 8) 9) 10) a c c d d b c c d b 11) 12) 13) 14) 15) 16) 17) 18) 19) 20) d d d b b c a b d b

166

IBPS PO EXAM 2013 : Quantitative Aptitude

Chapter: Mensuration
Mensuration is a branch of Mathematics which deals with lengths of lines, areas of surfaces and volume of solids. Mensuration may be divided into two parts: (i) (ii) Plane mensuration which deals with the sides, perimeters and areas of plane figures of different shapes. Solid mensuration which deals with the areas and volumes of solid objects.

Perimeter and Area of Triangles:

1.) If a, b, c are three sides of a triangle; then i) Perimeter = a+b+c ii) Area = {s(s-a)(s-b)(s-c)} , where s= semi-perimeter of triangle i.e., s= (a+b+c)/2 2.) If base and the corresponding altitude of triangle are known, then Area of triangle = base height 3.) If a is side of an equilateral triangle then i.) Perimeter = 3a ii.) Area = (3/4) a2 Perimeter and area of rectangles, square:

l b a a

1.) If l and b are length and breadth of rectangle, Then i.) Perimeter = 2(l + b) ii.) Area = lb iii.) Diagonal= (l2 + b2) 167 IBPS PO EXAM 2013 : Quantitative Aptitude

2.) If a is side of square , then i.) Perimeter = 4a ii.) Area = a2 iii.) Diagonal , d = a2 + a2 = a2 iv.) Side = area Area of Trapezium

Two sides of trapezium are parallel. If a and b are parallel sides and h is distance (i.e. height) between them; then Area of trapezium = (a + b) h Parallelogram

Area of parallelogram = bh where: b = base of the parallelogram h=height of the parallelogram Rhombus

Rhombus has all the sides equal and diagonals of rhombus bisect each other at right angles. If a is the side of rhombus and d1 , d2 are diagonals ; then Perimeter of rhombus = 4a Area of rhombus = d1 d2

168

IBPS PO EXAM 2013 : Quantitative Aptitude

Circle

If r is radius of circle, then diameter= 2r, circumference of circle = 2r, area of circle = r2 The Greek letter (pronounced as Pie) = 22/7 = 3.14 EXAMPLES: 1) Find the area of triangle whose sides are 10cm, 24cm and 26cm. Solution: a= 10cm b= 24cm c=26cm s= (a+ b+ c)/2= (10+ 24+ 26)/2 =30 area of triangle = {s(s-a)(s-b)(s-c)} = {30(30-10)(30-24)(30-26)} =120cm2 2) If two sides of a triangle are 6cm and 8cm and height of the triangle corresponding to 6cm side is 4cm; find the area of triangle: Solution: area of triangle= baseheight = 64 =12cm2 3.) Find the length and perimeter of the rectangle whose: Area = 120cm2 and breadth= 8cm Solution: area of rectangle =120cm2 breadth b=8cm Area = lb l 8 =120 l = 15cm Perimeter = 2 (l + b) = 2 (15 + 8) = 223 = 46cm 4.) The parallel sides of a trapezium are 8.4cm and 12.3cm. If its height is 7.2 cm, find its area. 169 IBPS PO EXAM 2013 : Quantitative Aptitude

Solution: sum of parallel sides = 8.4+ 12.3 = 20.7cm Height = 7.2cm Area of trapezium = (sum of parallel sides) height = 20.77.2 = 74.52cm2 3-D FIGURES: CUBOID: h lllllllll b l Lateral / curved surface area = 2(l+b)h Total surface area = 2(lb+bh+lh) Volume = lbh Diagonal of cuboid= (l2+b2+h2) Where: l= length, b= breadth, h = height CUBE: l l

Lateral/curved surface area = 4l2 Total surface area = 6l2 Volume = l3 Diagonal of cube = l3 Where: l = edge of cube

170

IBPS PO EXAM 2013 : Quantitative Aptitude

RIGHT CIRCULAR CYLINDER:

Lateral/curved surface area= 2rh Total surface area= 2r(r+h) Volume = r2h Where: r = radius h = height RIGHT CIRCULAR CONE:

Lateral/curved surface area = rl Total surface area = r(l+r) Volume = (1/3)r2h Where: r=radius of base h=height l=slant height = (r2+h2)

171

IBPS PO EXAM 2013 : Quantitative Aptitude

SPHERE:

Lateral/curved surface area = 4r2 Total surface area = 4r2 Volume = (4/3)r3 ,Where: r= radius of sphere HEMISPHERE:

Lateral/curved surface area = 2r2 Total surface area = 3r2 Volume = (2/3)r3 where: r= radius of hemisphere SPHERICAL SHELL:

Total surface area = 4(R2+r2) Volume = (4/3)(R3-r3) Where: R= external radius r = internal radius 172 IBPS PO EXAM 2013 : Quantitative Aptitude

EXAMPLES: 1.) Find the volume of right circular cylinder which has a height of 21cm, and the base radius 5cm. Also find the curved surface area of the cylinder. Solution: Given h=21cm, r=5cm Volume of cylinder=r2h = (22/7) 2521 = 1650cm3 Curved surface area = 2rh= 2 (22/7) 5 21= 660cm2 2.) A rectangular sheet of paper 44cm 18cm is rolled along its length and a cylinder is formed. Find the volume of cylinder.(Use =22/7) Solution: The sheet of paper is rolled along the length. Then, height of the cylinder= 18cm and the circumference of base of the cylinder= 44cm Let the radius of the base of the cylinder. 2 (22/7) r=44 r = 7cm and height = 18cm Volume of the cylinder = r2h = (22/7)7718 = 2772cm3 3.) A beam 9m long, 40cm wide and 20cm deep is made up of iron which weighs 50kg per cubic meters. Find the weight of the beam. Solution: Volume of the beam = lbh = 9 (4/100) (20/100)m3 = 72/100 m3 Since the beam is made up of iron which weighs 50kg per cubic metre. Weight of the beam = (72/100) 50 = 36kg 4.) A rectangular reservoir is 120m long and 75m wide. At what speed per hour must water flow into it through a square pipe of 20cm side so that the water rises by 24m in 18 hours? Solution: Volume of accumulated in 18 hours = (1207524)cu.m. Volume of water accumulated in one hour = (1207524)/18 m3 Area of the end of the square pipe = (20/100)2 = (1/25) m2 = 0.4 m2 Speed of the water per hour = (1207524)/(180.4) = 30,000 m = 30 km Hence water must flow at 30km/hour into the reservoir.

173

IBPS PO EXAM 2013 : Quantitative Aptitude

Solved Examples (Mensuration)


1. The sum of the ages of 4 members of a family 5 years ago was 94 years. Today, when the daughter has been married off and replaced by a daughter-in-law, the sum of their ages is 92. Assuming that there has been on other change in the family structure and all the people-are alive, what is the difference in the age of the daughter and the daughter-in-law? (IBPS CWE PO MT 2012) (1) 22 years (2) 11 years (3) 25 years (4) 19 years (5) 15 years Solution: There are four members in a family. Five years ago the sum of ages of the family members = 94 years Now, sum of present ages of family members = 94 + 5 4 = 114 years Daughter is replaced by daughter-in-law. Thus, sum of family member's ages becomes 92 years. Difference = 114 - 92 = 22 years Ans: (1) 2. The area of a square is 1444 square meters. The breadth of a rectangle is 1/4th the side of the square and the length of the rectangle is thrice the breadth. What is the difference between the area of the square and the area of the rectangle? (IBPS CWE PO MT 2012) (1) 1152.38 sq mtr (2) 1169.33 sq mtr (3) 1181.21 sq mtr (4) 1173.25 sq mtr (5) None of these Solution: Area of square = 1444 Let the side of square be a. So a2 = 1444 a= = 38 m Breadth of rectangle = 38 = 9.5 metres Length = 3 x 9.5 = 28.5 m 174 IBPS PO EXAM 2013 : Quantitative Aptitude

Area of rectangle = 28.5 x 9.5 = 270.75 m2 Difference = 1444 - 270.75 = 1173.25 sq m Ans: (4) 3. The side of a square is half the diameter of a circle. The area of the square is 1225 sq cm. What is the area of the circle? (1) 962.5 sq cm (2) 3850 sq cm (3) 15400 sq cm (4) 15600 sq cm (5) None of these Solution: Side of the square Radius of the circle = 35 cm Area of the circle Ans: (2) 4. The adjacent angles of a parallelogram are in the ratio of 5 : 7. The larger angle of the parallelogram is equal to the largest angle of a triangle. The smallest angle of the triangle is 40 less than the smaller angle of the parallelogram. What is the measure of the second largest angle of the triangle? (Corporation Bank PO 2011) (1) (2) (3) (4) (5) 30 40 55 35 None of these

Solution: Sum of adjacent angles of a parallelogram = 180 Larger angle of the parallelogram Smaller angle of the parallelogram = 180 105 = 75 Second-largest angle of the triangle = 180 105 (75 40) = 40 175 IBPS PO EXAM 2013 : Quantitative Aptitude

Ans: (2) 5. The side of a square is half the diameter of a circle. The area of the square is 1225 sq cm. What is the area of the circle? (Corporation Bank PO 2011) (1) (2) (3) (4) (5) 962.5 sq cm 3850 sq cm 15400 sq cm 15600 sq cm None of these

Solution: Side of the square Radius of the circle = 35 cm Area of the circle Ans: (2) 6. The sum of the circumference of a circle and the perimeter a square is equal to 272 cm. The diameter of the circle is 56 cm. What is the sum of the area of the circle and the area of the square? (Allahabad Bank Probationary Officers Exam 2011) (1) (2) (3) (4) (5) 2464 sq cm 2644 sq cm 3040 sq cm Cannot be determined None of these

Solution: Circumference of the circle = x diameter = 56 = 176 cm

Perimeter of the square = (272 - 176 =) 96 cm Side of the square = ( ) 24 cm

Area of the square = (24 x 24 =) 576 sq cm

176

IBPS PO EXAM 2013 : Quantitative Aptitude

Area of the circle = r2 =

x 28 x 28 = 2464 sq cm

required sum = (576 + 264) sq cm = 3040 sq cm Ans: (3) 7. The ratio of the three angles of a quadrilateral is 13: 9: 5 respectively. The value of the fourth angle of the quadrilateral is 36. What is the difference between the largest and the second smallest angles of the quadrilateral? (Allahabad Bank Probationary Officers Exam 2011) (1) (2) (3) (4) (5) 104 108 72 96 None of these

Solution: Let the three angles of the quadrilateral be13xo, 19xo and 5x o respectively. Now, according to the question; 13x + 9x + 5x = 360 - 36 = 324

27x = 324
x= = 12

required difference = 13x - 5x = 8x = 8 x 12 = 96 Ans: (4)

177

IBPS PO EXAM 2013 : Quantitative Aptitude

Practice Set (Mensuration)


1. The diameter of garden roller is 1.4 m and it is 2 m long. The area covered by the roller in 5 revolutions is a) 4.4 m2 b) 44 m2 c) 16.8 m2 d) 8.8 m2 2. In a cylindrical vessel of diameter 24 cm filled up with sufficient quantity of water, a solid spherical ball of radius 6 cm is completely immersed. Then, the increase in height of water level is a) 1.5 cm b) 2 cm c) 3 cm d) 4.2 cm 3. If the length of the diagonal of a cube is , then its surface area is a) 192 cm2 b) 512 cm2 c) 768 cm2 d) 384 cm2 4. A solid sphere of radius 1 cm is melted to convert into a wire of length 100 cm. The radius of the wire (take ) is a) 0.08 cm 178 IBPS PO EXAM 2013 : Quantitative Aptitude b) 0.09 cm c) 0.16 cm d) 0.11 cm 5. A field is in the form of a rectangle of length 18 m and width 15 m. A pit, 7.5 m long, 6 m broad and 0.8 m deep, is dug in a corner of the field and the earth taken out is evenly spread over the remaining area of the field. The level of the field raised is a) 12 cm b) 14 cm c) 16 cm d) 18 cm 6. A right circular cylinder, a hemisphere and a right circular cone stand on the same base and have the same height. The ratio of the volumes is a) 3 : 6 : 1 b) 3 : 4 : 1 c) 3 : 2 : 1 d) 4 : 3 : 1 7. The base of a right pyramid is an equilateral triangle of side 4 cm. The height of the pyramid is half of its slant height. Its volume is a) b)

c) d) 8. A tent is of the shape of a right circular cylinder upto a height of 3 m and then becomes a right circular cone with maximum height of 13.5 m above the ground. If the radius of the base is 14 m, the cost of painting the inner side of the tent at the rate Rs. 2 per sq m is a) Rs. 2050 b) Rs. 2060 c) Rs. 2068 d) Rs. 2080 9. A solid cone of height 9 cm with diameter of its base 18 cm is cut out from a wooden solid sphere of radius 9 cm. The percentage of wood wasted is a) 25% b) 30% c) 50% d) 75% 10. The number of spherical bullets that can be made out of a solid cube of lead whose edge measures 44 cm, each bullet being of 4 cm diameter, is a) 2541 b) 2451 c) 2514 d) 2415 11. From a solid cylinder whose height is 12 cm and diameter 10 cm, a 179

conical cavity of same height and same diameter of the base is hollowed out. The volume of the remaining solid is approximately

a) 942.86 cm3 b) 314.29 cm3 c) 628.57 cm3 d) 450.76 cm3 12. The base of a right pyramid is a square of side 16 cm long. If its height be 15 cm, then the area of the lateral surface (in cm 2) is a) 136 b) 544 c) 800 d) 1280 13. The curved surface area of a cylindrical pillar is 264 sq m and its volume is 924 cu m. The ratio of its diameter to height is a) 3 : 7 b) 7 : 3 c) 6 : 7 d) 7 : 6 14. The ratio of the volume of a cube and of a solid sphere is 363 : 49. The ratio of an edge of the cube and the radius of the sphere is

a) b) c) d)

7 : 11 22 : 7 11 : 7 7 : 22

IBPS PO EXAM 2013 : Quantitative Aptitude

15. The base of a solid right prism is a triangle whose sides are 9 cm, 12 cm and 15 cm. The height of the prism is 5 cm. Then, the total surface area of the prism is a) 180 cm2 b) 234 cm2 c) 288 cm2 d) 270 cm2 16. Volume of two cones are in the ratio 1 : 4 and their diameters are in the ratio 4 : 5. The ratio of their heights is a) 1 : 5 b) 5 : 4 c) 5 : 16 d) 25 : 64 17. Water is being pumped out through a circular pipe whose internal diameter is 7 cm, if the flow of water is 12 cm/s then how many litres of waters being pumped out in one hour? a) 1663.2 L b) 1500 L c) 1747.6 L d) 2000 L 18. A right circular cylinder of height 16 cm is covered by a rectangular tin foil of size 16 cm 22 cm. The volume of the cylinder is

a) 352 cm3 b) 308 cm3 c) 176 cm3 d) 616 cm3 19. The volume of a cube (in cm3), whose diagonal measures cm is a) 16 b) 27 c) 64 d) 8 20. Some solid metallic right circular cones, each with radius of the base 3 cm and height 4 cm, are melted to form a solid sphere of radius 6 cm. The number of right circular cones is a) 12 b) 24 c) 48 d) 6 21. A cuboidal water tank contains 216 L of water. Its depth is of its length and breadth is of of the difference between length and depth. The length of the tank is a) 72 dm b) 18dm c) 6 dm d) 2 dm

180

IBPS PO EXAM 2013 : Quantitative Aptitude

Mensuration Practice Set (Answers)

1) 2) 3) 4) 5) 6) 7) 8) 9) 10) 11)

b b d d c c c c d a c

12) 13) 14) 15) 16) 17) 18) 19) 20) 21)

b b b c d a d c b b

181

IBPS PO EXAM 2013 : Quantitative Aptitude

Chapter: Permutations, Combinations & Probability


Permutation The arrangement of a number of things taking some or all of them at a time is called permutation. If there are n number of things and we have to select r things at a time then the total number of permutation is denoted by n =

For example if there are 3 candidates A ,B and C for the post of president and vice president of a college , since we have to select only 2 candidates , it can be done in 3! Ways. i.e. (A, B) (B, C) (A, C) (B, A) (C, B) and (C, A). Here order of arrangement matters. Restricted Permutation: Sometimes we have to find out the number of permutation keeping few specific objects at specific places. In this case, we find out the number of permutation of filling remaining vacant places by the remaining objects. If r objects are taken out of n dissimilar objects (i) A specific object is taken each time: if there are n objects Suppose that is taken each time. If .

takes first place then the remaining (n-1) can take any place so number of

objects can be arranged in n-1

ways. Since

permutation is r n-1 . (ii) Specific object never taken: then r objects are taken out of (n-1) objects, so number of permutation is n-1 . {Note: n = n-1 + r n-1 }

Permutation of things when some are identical: If we have n things in which p are exactly of one kind , q of second kind , r of third kind and the rest are different then the number of permutation of n things taken all at a time n 182 IBPS PO EXAM 2013 : Quantitative Aptitude =

Example: In how many ways can the letters of the word LEADER be arranged? Solution: The word LEADER contains total 6 letters namely 1L, 2E, 1A, 1D, 1R Therefore, the number of ways to arrange the letters of the word LEADER = = 360.

Repetition of things: The number of permutation formed by taking r things at a time out of n things in any object arrangement such that each object can be taken any number of time is Circular permutation: If we fix one of the objects around the circumference of a circle then number of permutation of n different thing taken all at a time is (n-1)! Ways. It will be same as by putting (n-1) objects at (n-1) places. But if we do not consider the direction i.e. clockwise and anticlockwise then the number of permutation is Combination From a given group of object each of the number of groups which are formed by taking some objects or all objects at a time without caring about the sequence of the objects is called combination. The number of combination formed by taking r objects at a time out of n object is denoted by n n = where C expresses combination. . .

For example if we have 3 objects A , B and C , 2 objects are taken out at a time then 3 combination are formed AB , BC and CA. Note: If r= 0 , then n = =1

183

IBPS PO EXAM 2013 : Quantitative Aptitude

If r= 1 , then n If r= n , then n n =n

= =

=n =1

Example: Find the value of Solution: We have, Restricted combination: =

. = = 100 {Because 1! =1}

The combination of r object out of n objects on which p specific objects: 1. Are always included is n-p remaining (n-p). 2. Are never included is n-p . Since p specific object are never included we have to form the combination taking r obects out of (n-p) objects. The number of ways to select some or all thing out of any number of given thing: There are 2 ways to select anything i.e. either it will be selected or not. Therefore number of ways to select n things is 2 For non- empty selection is Note: n +n 2 -1. = -1. n times = . In these empty selection is also include. . We have to keep aside p specific objects and to select

+. + n

184

IBPS PO EXAM 2013 : Quantitative Aptitude

Difference between permutation and combination: Suppose there are 5 objects out of which 2 have to be chosen. Permutation Number of required way = = = = = 5 4 = 20 = = 10 Combination

So it is clear that in permutation order matters while in combination order does not matter. Probability The mathematical measure of the uncertainty is called probability. For example, consider the following questions: (a) (b) (c) (d) Will it rain today? Which of the three candidates will win? On throwing a dice, the number obtained will be even or odd? On tossing a coin, head will occur or tail will occur?

The answer to all these question is not sure i.e. there is uncertainty .We study the uncertainty of the result of such question in the theory of probability , which may not have one result but more than one result are possible . Random experiment: The experiments in which the outcomes cannot be predicted before hand is called random experiments. When these kind of experiment are repeated under identical condition, they do not produce the same outcome every time and there may be many possible outcome which depends upon chance and cannot be predicted. For example, on tossing a coin either the head will come up or the tail will come up, we cannot predict it. This is an example of random event. Sample Space: The set of all possible outcomes of experiments is called the sample space and it is denoted by S. And the subset of a sample space is called an event. That is, every subset A of the sample 185 IBPS PO EXAM 2013 : Quantitative Aptitude

space S is an event of that random experiment. For example, in an experiment of tossing a coin, if h is obtained then it is a random event, since here S = {H, T} and {H} S Now, the probability of any event A can be defined as the ratio between the number of favourable outcomes to the event A and the number of total equiprobable outcomes, that is P(A) = Here it should be noted that the probability of a certain or sure event is 1 and that of impossible event is 0. Now, since the probability of an event to occur is =

So the probability of an event A not to occur is = 1 Mutually Exclusive events: Two events A and B are said to be mutually exclusive if they cannot occur together, that is simultaneously. For example , on throwing a dice , the events A = { 2,4,6 } and B = { 1,3,5 } are mutually exclusive events , i.e. A B = . In term of probability if A and B are mutually exclusive events, then P (A B) = P (A) + P (B) and, P (S) = P (A) + P (A) = 1 where A is Complement of A. Example: If a dice is thrown once then the probability of the number appearing on dice is more than 2? (a) (b) (c) (d) 1/3 1/2 2/3 4.3/4

Solution: As there are 6 faces on a dice, So the total number of possible events are 1, 2 , 3 . 6 , that is = 6 Now the number more than are 3, 4 , 5 and 6 So total number of favourable events =

186

IBPS PO EXAM 2013 : Quantitative Aptitude

Probability of an event =

Required probability = = Example : An urn contains 3 green, 6 red, and 4 black balls. 3 balls are drawn. Find the probability that all 3 balls are of same colour? (a) 3/44 (b) 25/286 (c) 15/286 (d) 5/286

Solution: Total number of balls in an urn is 13. Number of ways 3 balls can be drawn out of 13 balls = Numbers of ways 3 green balls are drawn = Numbers of ways 6 red balls are drawn = Numbers of ways 4 black balls are drawn = Now, The required probability = + + = = = = =1 = 20 =4 = = 286

187

IBPS PO EXAM 2013 : Quantitative Aptitude

Solved Examples (Permutations, Combinations & Probability)


1. In how many different ways can the letters of the word 'THERAPY' be arranged so that the vowels never come together? (IBPS CWE PO MT 2012) (1) 720 (2) 1440 (3) 5040 (4) 3600 (5) 4800 Solution: Total number of letters is 7, and these letters can be arranged in 7! ways . = 1 x 2 x 3 x 4 x 5 x 6 x 7 = 5040 ways There are seven letters in the word THERAPY including 2 vowels. (E, A) and five consonants. Consider two vowels as one letter. We have 6 letters which can be arranged in 6P6 = 6 ways. But vowels can be arranged in 2! ways. Hence, the number of ways, all vowels will come together = 6! x 2! = 1 x 2 x 3 x 4 x 5 x 6 x 2 = 1440 Total number of ways in which vowels will never come together = 5040 - 1440 = 3600 Ans: (4) 2. A bag contains 13 white and 7 black balls. Two balls are drawn at random. What is the probability that they are of the same colour? (IBPS CWE PO MT 2012) (1) (2) (3) (4)

188

IBPS PO EXAM 2013 : Quantitative Aptitude

(5) Solution: Total number of balls = 13 + 7 = 20 Number of sample space = n(S) = 20C2 = 190 Number of events = n(E) = 13C2 + 7C2 = 78 + 21 = 99 P(E) = Ans: (4) Directions (1-5): Study the given information carefully to answer the questions that follow. An urn contains 4 green, 5 blue, 2 red and 3 yellow marbles. 3. If two marbles are drawn at random, what is the probability that both are red or at least one is red? (1) (2) (3) (4) (5) None of these Solution: Total number of marbles in the urn = 4 + 5 + 2 + 3 = 14 Total number of possible outcomes = Selection of 2 marbles out of 14 marbles = 14C2= 91 =

Total number of favourble cases = 2C2 + 2C1 + 12C1 = 1 + 2 x 12 = 25 189 IBPS PO EXAM 2013 : Quantitative Aptitude

required probability = Ans: (5) 4. If three marbles are drawn at random, what is the probability that at least one is yellow? (1)

(2)

(3)

(4)

(5) None of these Solution: Total number of possible outcomes = 14C3 = = 364

When no marbles is yellow, favourable number of cases = 14C3 = = 165

Probability that no marble is yellow = required probability = (Probability that at least one is yellow) = (1 - Probability that no marble is yellow) 1Ans: (3) = =

190

IBPS PO EXAM 2013 : Quantitative Aptitude

5. If eight marbles are drawn at random, what is the probability that there are equal numbers of marbles of each colour? (1) (2) (3) (4) (5) None of these Solution: Total possible outcomes = 14C8 = 14C6 [ = 3003 Total number of favourable cases = 4C2 x 5C2 x 2C2 X 3C2 = 6 x 10 x 1 x 3 = 180 = 3003 required probability = Ans: (3) 6. If three marbles are drawn at random, what is the probability that none is green? (1) =
n

Cr = nCn-r]

(2)

(3)

(4)

191

IBPS PO EXAM 2013 : Quantitative Aptitude

(5)

Solution: Total number of possible outcomes = 14C2 = = 364

Now, according tot the question, no marble should be green. Total number of favourable outcomes = Selection of 3 marbles out of 5 blue, 2 red and 3 yellow marbles = 10C3 = = 120 =

required probability = Ans: (5)

7. If three marbles are drawn at random, what is the probability that two are blue and two are red? (1)

(2) (3)

(4) (5) None of these Solution: Total number of possible outcomes = 14C4 = = 1001

Total number of favourable cases 192 IBPS PO EXAM 2013 : Quantitative Aptitude

= 5C2 x 22C = 10 x 1 = 10 required probability = Ans: (1) Directions (Q. 8-10): Study the given information carefully and answer the questions that follow: A basket contains 4 red, 5 blue and 3 green marbles. 8. If three marbles are picked at random, what is the probability that either all are green or all are red? (1) (2) (3) (4) (5) None of these Solution: P(All Green) + P(All Red) = 3C3 / 12C3 + 4C3 / 12C3 = 1/44 Ans : (4) 9. If two marbles are drawn at random, what is the probability that both are red? (1) (2) (3) (4) (5) None of these 193 IBPS PO EXAM 2013 : Quantitative Aptitude

Solution: 4C2 / 12C2 = 1/11 Ans: (5) 10. If three marbles are picked at random, what is the probability that at least one is blue? (1) (2) (3) (4) (5) None of these Solution: 1 P (None Blue) = 1- (7C3 / 12C3) = 37/44 Ans: (2)

194

IBPS PO EXAM 2013 : Quantitative Aptitude

Practice Set-1 (Permutations, Combinations & Probability)


1. In how many ways can six different rings be worn on four fingers of one hand? a) 10 b) 12 c) 15 d) 16 2. There are three prizes to he distributed among five students. If no student gets more than one prize, then this can be done in a) 10 ways b) 30 ways c) 60 ways d) 80 ways 3. In a hockey championship, there were 153 matches played. Every two teams played one match with each other. The number of teams participating in the championship is a) 18 b) 19 c) 17 d) 16 4. In an examination paper, there are two groups each containing 4 questions. A candidate is required to attempt 5 questions but not more than 3 questions from any group. In how many ways can 5 questions be selected? 195 a) 24 b) 48 c) 96 d) None of these 5. After a get together every person present shakes the hand of every other person. If there were 105 hands shakes in all, how many persons were present in the party? a) 14 b) 13 c) 15 d) 16 6. There are 4 candidates for the post of a lecturer in Mathematics and one is to be selected by votes of 5 men. The number of ways in which the votes can be given is a) 1048 b) 1072 c) 1024 d) none of these 7. A student is to answer 10 out of 13 questions in an examination such that he must choose at least 4 from the first five questions. The number of choices available to him is a) 140 b) 280 c) 196 d) 346

IBPS PO EXAM 2013 : Quantitative Aptitude

8. After a get-together every person present shakes the hand of every other person. If there were 105 hands shakes in all, how many persons were present in the party? a) 14 b) 13 c) 15 d) 16 9. Out of eight crew members three particular members can sit only on the left side. Another two particular members can sit only on the right side. Find the number of ways in which the crew can be arranged so that four men can sit on each side. a) 864 b) 865 c) 863 d) 1728

196

IBPS PO EXAM 2013 : Quantitative Aptitude

Practice Set-2 (Permutations, Combinations & Probability)


1. Four different objects 1, 2, 3, 4 are distributed at random in four places marked 1, 2, 3, 4. What is the probability that none of the objects occupy the place corresponding to its number? a) b) c) a) d) 2. Two dice are tossed. The probability that the total score is a prime number is a) b) c) d) 3. A bag contains 3 white balls and 2 black balls. Another bag contains 2 white balls and 4 black balls. A bag and a ball are picked at random. The probability that the ball will be white is 197 a) b) c) d) b) c) d) 5. A bag contains 2 red, 3 green and 2 blue balls. 2 balls are to be drawn randomly. What is probability that the balls drawn contain no blue ball? a) b) c) d) 4. Suppose six coins are flipped. Then the probability of getting at least one tail is

IBPS PO EXAM 2013 : Quantitative Aptitude

6. I forgot the last digit of a 7-digit telephone number. If I randomly dial the final 3 digits after correctly dialling the first four, then what is the chance of dialling the correct number? a) b) c) d) 7. A box contains 6 white balls and 7 black balls. Two balls are drawn at random. What is the probability that both are of the same colour? a) b) c) d) 8. A brother and sister appear for an interview against two vacant posts in an office. The probability of the brother's selection is and that of

the sister's selection is . What is the probability that one of them is selected? a) b) c) d) 9. A room has 3 lamps. From a collection of 10 light bulbs of which 6 are not good, a person selects 3 at random and puts them in a socket. The probability that he will have light, is a) 5/6 b) 1/2 c) 1/6 d) none of the above 10. Four boys and three girls stand in queue for an interview. The probability that they will stand in alternate positions is a) 1/34 b) 1/35 c) 1/17 d) 1/68

198

IBPS PO EXAM 2013 : Quantitative Aptitude

Permutations, Combinations & Probability Practice Set-1 (Answers)


1) 2) 3) 4) 5) 6) 7) 8) 9) c a a b c d a c d

199

IBPS PO EXAM 2013 : Quantitative Aptitude

Permutations, Combinations & Probability Practice Set-2 (Answers)

1) 2) 3) 4) 5) 6) 7) 8) 9) 10)

c b d c b d b b d a

200

IBPS PO EXAM 2013 : Quantitative Aptitude

Chapter: Data Interpretation


Introduction: Analyzing data is the major part of our daily routine. Financial data as in P & L sheet, marketing and sales data, data on productivity, data on performance appraisal, data on each and every thing that we can imagine. In some companies entire department generates and manages every conceivable data that we can imagine. To collect information from all the amount of data it needs to be presented in a lucid and concise manner. Therefore we use data representation as it immediately provides the overall scenario and it is also sufficient to compute any detailed information. Strategies: While studying the DI section one should follow few strategies given below: 1. Solve the graph with which you are most comfortable for example some are more comfortable with line graph while the other may be with pie chart. 2. The problems with numbers with 2 or 3 digits like 82, 114 etc are easier to solve then the problem with number of 4 or 5 digits like 3457, 52468. 3. In pie charts if sectors are 15% 20% etc then it will be easier calculation then the sectors like 17.5%, 23.6 % etc. 4. We must also look at the number in answer option, if the answers are 12.5 5 6.33% etc then it will be easier to arrive at. 5. Wider are the choices in answer, easier will be the elimination process and lesser will be the calculation. 6. The answer choice Cannot be determined makes the question much easier as it becomes data sufficiency question and if the question can be answered then only four options are left.

201

IBPS PO EXAM 2013 : Quantitative Aptitude

7. Pick the alternative which is the middle one and check if your answer is less than, equal to or greater than this value. In this way only one iteration will give you the correct answer. 8. Te answer choice None of these on the other hand make it more difficult as we have to calculate the question to the exact value. 9. It is better to solve a line chart with two lines and 5 points than to solve a table with 5 rows and 6 columns. But this may not be the case always looking at the other factors. Growth rate and Growth: Growth and Growth rate are two different cases. Growth refers to just increase in the underlying value, while the Growth rate refers to the percentage increase. For example following table shows the sales and profit of a company A in Rs. Lakhs

1999 Sales Profit Here, 240 50

2000 290 55

2001 320 70

2002 350 50

2003 380 40

Growth during the period from 2000 to 2002 is simple the difference of sales = 350-290 = 60 But the Growth rate during this period is the percentage of growth = Profit Percentages: The profit percentage is not but it is . For example in above .

table the profit percentage of year 1999 is not =

202

IBPS PO EXAM 2013 : Quantitative Aptitude

But the profit percentage of year 1999 is =

as cost price of this year is 190

Also the percentage change in profit percentage is percent of profit percent of two years with base as previous year profit percent. For example the percentage change in profit percentage in the year 2002 over that in year 2001 is:

= =

But in line graph growth rate is related to slope of line. There are few points to understand: 1. The slope of each segment is same across all years of company B but still the growth rate is not the same in all years. The same slope simply means the sales grow by a constant amount each year. But the growth rate depends on the base value so in company B its decreasing from 2000 to 2004. 2. The line representing sales of A in 2002-03 is steeper but 2003 is not the year with highest growth rate of sales of A . The highest growth rate of sale of A is in year 2001.

203

IBPS PO EXAM 2013 : Quantitative Aptitude

Chapter: Data Interpretation-Table Chart


In studying problems on statistics, the data collected by the investigator are arranged in systematic form, called the tabular form. In order to avoid some heads again and again, we make tables, consisting of horizontal lines called rows and vertical lines called columns with distinctive heads, known as captions. Units of measurements are given along with the captions. Example: The table given below shows the population, literates and illiterates (in thousands) and the percentage of literacy in 3 states, in a year: State Population Literates Illiterates Percentage of literacy .. 16.1

Madras Bombay Bengal

49342 . 60314

6421 4068

.. 16790

After reading the table, mark a tick () against the correct answer in each question given below and hence complete the table. 1) Percentage of literacy in Madras is (a) 14.9% (b) 13.01% (c) 12.61% (d) 15.04% 2) Percentage of literacy in Bombay is (a) 19.5% (b) 16.7% (c) 18.3% (d) 14.6%

204

IBPS PO EXAM 2013 : Quantitative Aptitude

3) Number of literates in Bengal (in thousands) is: (a) 50599 (b) 9715 (c) 76865 (d) 9475 Solution: 1) (b) percentage of literacy in Madras = (6421/49342)100% = 13.01% 2) (a) Population of Bombay = (4068+16790) thousands = 20858 thousands. Therefore, percentage of literacy in Bombay = (4068/20858)100% = 19.5% 3) (b) Number of literates in Bengal =(16.1/100)60314 = 9715 thousands

205

IBPS PO EXAM 2013 : Quantitative Aptitude

Solved Examples (Data Interpretation-Table Chart)


Direction (1-5): Study the table carefully to answer the questions that follow :- (IBPS PO Exam 2011) Number of people visiting six different Super-markets and the percentage of Men, Women and Children visiting those Super-markets Names of the Total Number of Supermarkets Percentage of

People

Men

Women

Children

34560

35

55

10

65900

37

43

20

45640

35

45

20

55500

41

26

33

42350

06

70

24

59650

24

62

14

206

IBPS PO EXAM 2013 : Quantitative Aptitude

1. The Number of men visiting Super-market D forms approximately what percent of the total number of people visiting all the Super-markets together? (1) 11 (2) 5.5 (3) 13 (4) 9 (5) 7.5 Solution: Number of men visiting supermarket D = 41% of 55500 = = 4155500 = 22755

Total number of people visiting all the supermarkets together = 34560 + 65900 + 45640 + 55500 + 42350 + 59650 = 303600 required probability = Ans: (5) 2. The Number of children visiting Super-market C forms what percent of number of children visiting Super- market F? (rounded off to two digits after decimal) (1) 91.49 (2) 49.85 (3) 121.71 (4) 109.30 (5) None of these Solution: Number of children visiting supermarkets C = 20% of 45640 = 20 x 45640 = = 9128 100 = 7.5% (Aprox)

Number of children visiting supermarket F = 14% of 59650 = 207 = 8351 IBPS PO EXAM 2013 : Quantitative Aptitude

required percentage = Ans: (4)

x 100 = 109.30%

3. What-is the total number of children visiting Super-markets B and D together? (1) 18515 (2) 28479 (3) 31495 (4) 22308 (5) None of these Solution: Total number of children visiting supermarket B and D together = 20% of 65900 + 33% of 55500 = +

= 13180 + 18315 = 31495 Ans: (3) 4. What is the average number of women visiting all the Super-markets together? (1) 24823.5 (2) 22388.5 (3) 26432.5 (4) 20988.5 (5) None of these Solution: Total number of women = 55% of 34560 + 43% of 65900 + 45% of 45640 + 26% of 55500+ 70% of 42350 + 62% of 59650 = 19008 +'28337 + 20538 + 14430 + 29645 + 36983 = 148941 required average = Ans: (1) = 24823.5

208

IBPS PO EXAM 2013 : Quantitative Aptitude

5. What is the ratio of number of women visiting Super-markets A to that visiting Supermarket C? (1) 35: 37 (2) 245: 316 (3) 352: 377 (4) 1041: 1156 (5) None of these Solution: Required ratio = 19008: 20538 = 1056: 1141 Ans : (3) (Directions Q.6-10): Study the table carefully to answer the questions that follow :- (IBPS PO Exam 2011)
Percentage SUBJECTS (Maximum Marks) of Marks Obtained by Students Strategic Brand Compensation Consumer Service Training & Different Students in Management Management Management Behaviour Marketing Development Different (150) (100) (150) (125) (75) (50) Subject of MBA Anushka 66 75 88 56 56 90 ,

Archit Arpan Garvita Gunit Pranita

82 76 90 64 48

76 66 88 70 56

84 78 96 68 50

96 -88 76 7.2 64

92 72 84 68 64 ,

88 70 86 74 58 1;;' ~ ,'_ ,-

6. How many marks did Anushka get in all the Subjects together? (1) 369 (2) 463 (3) 558 (4) 496 (5) None of these Solution: Total marks of Anuska = 209 + 75 + + + + 45

IBPS PO EXAM 2013 : Quantitative Aptitude

= = 99 + 75 + 132 + 70 + 42 + 45 = 463 Ans: (2) 7. The Marks obtained by Garvita in Brand Management are what percent of marks obtained by Archit in the same Subject? (rounded off to two digits after decimal) (1) 86.36 (2) 101.71 (3) 115.79 (4) 133.33 (5) None of these Solution: Marks obtained by Garvita in Brand Management = 88% of 100 = 88 Marks obtained by Archita in Brand Management = 76% of 100 = 76 required percentage = Ans: (3) 8. What is the average marks obtained by all students together in Compensation Management? (1) 116 (2) 120 (3) 123 (4) 131 (5) None of these Solution: Average marks obtained by all students together in Compensation Management x 100 115.79%

210

IBPS PO EXAM 2013 : Quantitative Aptitude

150=116

Ans: (1) 9. Who has scored the highest total marks in all the subjects together? (1) Archit (2) Gunit (3) Pranita (4) Garvita (5) Arpan Solution: Total obtained in all the subjects together by Arapn: 76% of 150 + 66% of 100 + 78% of 150 + 88% of 125 + 72% of 75 + 70% of 50 = + + + + +

= 114 + 66 + 117 + 110 + 54 + 35 = 496 Archit: 82% of 150 + 76% of 100 + 84% of 150 + 96% of 125 + 92% of 75 + 88% of 50 = + + + + +

= 123 + 76 + 126 + 120 + 69 + 44 = 558 Garvita: 90% of 150 + 88% of 100 + 96% of 150 + 76% of 125 + 84% of 75 + 86% of 50 . = 135 + 88 + 144 + 95 + 63 + 43 = 568 = + + + + +

= 135 + 88 + 144 + 95 + 63 + 43 = 568 Gunit: 64% of 150 + 70% of 100 + 68% of 150 + 72% of 125 + 68% of 75 + 74% of 50 = + + + + +

= 96 + 70 + 102 + 90 + 51 + 37 = 446 Pranita: 48% of 150 + 56% of 100 + 50% of 150 + 64% of 125 + 64% of 75 + 58% of 50 211 IBPS PO EXAM 2013 : Quantitative Aptitude

= 72 + 56 + 75 + 80 + 48 + 29 = 360 Clearly, Garvita scored the highest total marks in all the subjects together. Ans: (4) 10. How many Students have scored the highest marks in more than one Subject? (1) three (2) two (3) one (4) none (5) Now of these Solution: Archit (consumer behaviour and service marketing) and Garvita (strategic management, brand management and compensation management). Ans: (2)

212

IBPS PO EXAM 2013 : Quantitative Aptitude

Practice Set (Data Interpretation-Table Chart)


Directions (Q. 1-5): Study the table carefully to answer the questions that follow Number of cars (in thousand) of two models (Basic and Premium) produced by five different companies in five different years (IBPS RRB Grade Officer Exam 2012) Compan y Year 2006 2007 2008 2009 2010 Basi c Premiu m Basi c Premiu m Basi c Premiu m Basi c Premiu m Basi c Premiu m A B C D E

4.4 4.9 13.6 6.6 5.8

2.5 7.2 15.5 13.9 14.9

5.6 9.4 14.8 11.8 12.2

2.4 7.2 9.5 11.4 7.2

5.4 7.5 12.8 16.6 19.9

6.1 8.3 9.9 18.2 22.3

7.6 8.4 9.2 10.6 14.6

7.5 4.9 8.2 10.4 12.2

2.7 4.2 7.7 7.2 13.2

5.1 5.5 11.5 12.8 12.2

1. The number of cars of premium model produced by Company D in the year 2009 was approximately what per cent of the total number of cars (both models) produced by Company C in the year 2007? (1) (2) (3) (4) (5) 70 51 56 61 66 213 IBPS PO EXAM 2013 : Quantitative Aptitude

2. What was the approximate percentage decrease in the number of cars of basic model produced by Company B in the year 2009 as compared to the previous year? (1) 15 (2) 20 (3) 10 (4) 80 (5) 85 3. What was the average number of cars of premium model produced by Company A over all the years together? (1) (2) (3) (4) (5) 9000 8000 6000 48000 None of these

4. In which year was the difference between the basic model and the premium model of cars produced by Company E the second highest? (1) (2) (3) (4) (5) 2010 2006 2007 2008 2009

5. In which company did the production of cars of premium model consistently increase from the year 2006 to the year 2010? (1) (2) (3) (4) (5) Both C and E Both C and d C only D only E only

214

IBPS PO EXAM 2013 : Quantitative Aptitude

Directions (Q. 6-10): Study the table carefully to answer the questions that follow. Number of animals in grasslands of four different countries in five different years (RBI GradeB Officers Exam 2011) Country Year South Africa Tiger 1990 1995 2000 2005 2010 145 134 120 110 160 Lion 156 165 135 184 224 Bear 250 354 324 285 264 Tiger 320 445 583 466 411 China Lion 346 256 325 475 535 Bear 436 542 454 322 534 Sri Lanka Tiger 280 354 433 343 535 Lion 468 354 345 324 532 Bear 255 343 545 546 453 Tiger 423 368 354 562 349 England Lion 342 136 267 235 345 Bear 234 345 456 567 324

6. What is the average of the number of tigers in the grassland of Sri Lanka over all the years together? (1) (2) (3) (4) (5) 386 389 369 276 None of these

7. What is the difference between the total number of lions and bears in the grassland of England in the year 2005 and the number of tigers in the grassland of South Africa in the year 1995? (1) (2) (3) (4) (5) 597 558 677 668 None of these 215 IBPS PO EXAM 2013 : Quantitative Aptitude

8. The total number of animals together in the grassland of China in the year 1990 is approximately what per cent of the total number of bears in the grassland of Sri Lanka over all the years together? (1) (2) (3) (4) (5) 44% 56% 41% 47% 51%

9. If 35 per cent of the total number of animals in the grassland of China in the year 2010 died due to an epidemic, how many animals remained in the grassland of China in the year2010? (1) (2) (3) (4) (5) 976 952 986 962 None of these

10. What is three-fourths of the total number of lions in the grasslands of all the four countries in the year 2000? (1) (2) (3) (4) (5) 848 868 804 824 None of these

216

IBPS PO EXAM 2013 : Quantitative Aptitude

Directions (Q. 11-5): Study the table carefully to answer the questions that follow: Number of girls and boys (in hundreds) in six different years in five different schools School Years 2005 2006 2007 2008 2009 2010 A B C D E

Boys Girls Boys Girls Boys Girls Boys Girls Boys Girls 3.3 6.6 9.3 5.4 3.6 4.2 6.9 9.6 5.2 4.9 4.7 6.3 7.5 9.8 3.1 2.2 4.2 5.4 5.9 4.4 5.5 6.9 5.8 6.6 8.7 4.5 3.3 4.9 5.2 2.4 4.4 6.4 5.3 1.4 6.5 2.3 5.5 3.3 2.7 5.4 5.4 6.6 3.6 2.4 5.7 6.5

8.4 12.9 12.3 14.4

6.6 12.1 5.2 6.8

11.7 4.2 12.2 9.4 10.8 12.7

11. What is the approximate percentage decrease in the number of boys in School D in the year 2008 as compared to that in the previous year? (1) (2) (3) (4) (5) 17 12 9 5 23

12. The number of girls in School B in the year 2009 is approximately what per cent of the total number of students (both boys and girls) in School E in the year 2006? (1) (2) (3) (4) (5) 46 52 70 58 65

13. What is the average number of girls in School A in all the years taken together? (1) 760 (2) 800 (3) 860 217 IBPS PO EXAM 2013 : Quantitative Aptitude

(4) 600 (5) None of these 14. What is the ratio of the number of boys in School C in the year 2009 to the number of girls in School A in the year 2009? (1) (2) (3) (4) (5) 29 : 41 36 : 11 29 : 43 36: 13 None of these

15. In which year is the total number of students (both girls and boys together) the third highest in School E? (1) (2) (3) (4) (5) 2006 2007 2008 2005 2010

218

IBPS PO EXAM 2013 : Quantitative Aptitude

Directions-(Q. 16-20) Study the table carefully to answer the questions that follow: Number of Athletes (in hundreds) who participated in a Sports Event from Five Different Countries over the years (Allahabad Bank Probationary Officers Exam 2011) Countries Year 2005 2006 2007 2008 2009 2010 A B C D E

Male Fem Male Fem Male Fem Male Fem Male Fem 4.4 66 46 9.6 11.8 82 3.3 42 18 49 64 52 6.3 84 74 11.4 106 64 4.2 62 48 84 52 72 4.5 69 48 66 79 108 3.1 33 28 42 63 69 5.6 84 93 126 14.4 156 4.1 63 73 94 4.7 78 87 89 2.1 52 65 58 92 98

102 118 121 136

16. In which of the following years was the total number of participants (athletes) second highest from Country C? (1) (2) (3) (4) (5) 2005 2006 2007 2008 None of these

17. What was the average number of female athletes who participated from Country B over all the years together? (1) (2) (3) (4) (5) 1200 400 600 1800 3600

219

IBPS PO EXAM 2013 : Quantitative Aptitude

18. What was the approximate percentage decrease in the number of male athletes who participated from Country C in the year 2007 as compared to the previous year? (1) (2) (3) (4) (5) 21 30 35 39 25

19. The Number of female athletes who participated from Country E in the year 2009 was approximately what percentage of the 42 total number of athletes who participated from Country-B in the year 2008? (1) (2) (3) (4) (5) 40 46 50 56 60

20. In which of the following country is the difference between the number of male and female participants second highest in the year 2006? (1) (2) (3) (4) (5) A B C D E

220

IBPS PO EXAM 2013 : Quantitative Aptitude

Data Interpretation-Table Chart Practice Set (Answers)


1) 2) 3) 4) 5) 6) 7) 8) 9) 10) 5 2 5 5 3 2 4 5 4 3 11) 12) 13) 14) 15) 16) 17) 18) 19) 20) 1 5 3 3 5 5 3 2 2 5

221

IBPS PO EXAM 2013 : Quantitative Aptitude

Chapter: Data Interpretation-Line Graphs


Line graphs of a frequency distribution is obtained from the histogram of the frequency distribution by joining the mid points of respective tops of the rectangles in a histogram. To complete the line graphs, the mid-points at each end are joined to the immediately lower or higher mid-points (as the case may be) at zero frequency. Example: Study the following graph and answer the following questions:

1.) The total expenditure of which of the following pairs of years was equal to the income in 1992? (a) 1987 and 1988 (b) 1987 and 1989 (c) 1988 and 1989 (d) 1988 and 1990 (e) none of these 2.) What was the percentage decrease in expenditure from 1988 and 1989? 222 IBPS PO EXAM 2013 : Quantitative Aptitude

(a) 80 (b) 50 (c) 40 (d) 10 (e) none of these 3.) In how many of the given years was the expenditure more than the average expenditure of the given years? (a) 4 (b) 3 (c) 1 (d) 5 (e) none of these 4.) In which of the following years was the percentage of expenditure to income, the highest? (a) 1987 (b) 1988 (c) 1989 (d) 1991 (e) none of these 5.) What was the approximate percentage increase in income from 1991 to 1992? (a) 35 (b) 40 (c) 20 (d) 15 (e) 25 Solution: 1.) (c) : income in 1992= 475 crores Total expenditure in 1988 and 1989 = Rs.( 250+225) crores =Rs. 475 crores. 2.) (d) : expenditure in 1988 = Rs. 250 crores Expenditure in 1989 = Rs. 225 crores.

Decrease % =

= 10%

3.) (b): average expenditure =Rs. =Rs. 287.5 The expenditure is greater than the average expenditure during the years 1987, 1990 and 1992. Required no. of years =3 4.) (e): the required percentage : In 1987 is (300100/450)% = 66.66% In 1988 is (250100/400)% = 62.5% In 1989 is (225100/350)% = 64.29% 223 IBPS PO EXAM 2013 : Quantitative Aptitude

In 1990 is (375100/425)% = 88.24% In 1991 is (175100/375)% = 46.6% In 1992 is (400 100/475)% = 84.21% Clearly the percentage is highest in 1990. 5.) (e): income in 1991 = 375 crores Income in 1992 = 475 crores Therefore increase % = (100100/375)% = 26.6% = 25% nearly

224

IBPS PO EXAM 2013 : Quantitative Aptitude

Solved Examples (Data Interpretation-Line Graphs)


Directions-(Q.1-5): Study the following graph carefully to answer the questions that follow: (Allahabad Bank Probationary Officers Exam 2011)

1. In which state was the total number of trees planted by NGO A and NGO B together second lowest? (1) (2) (3) (4) (5) Bihar Punjab Haryana Assam Tamil Nadu

Solution: Number of tree planted by NGO-A and NGO-B together in Bihar: 100 + 60 = 160 Punjab: 120 + 80 = 200 Haryana: 80 + 140 = 220 Assam: 150 + 160 = 310 225 IBPS PO EXAM 2013 : Quantitative Aptitude

Tamil Nadu: 140 + 180 = 320 Ans: (2) 2. What was the difference between the trees planted by NGO A in Haryana and the number of trees planted by NGO C in Tamil Nadu? (1) (2) (3) (4) (5) 90 60 120 100 None of these

Solution: Required difference = 160 80 = 80 Ans: (5) 3. What was the average number of trees planted in Haryana by all the NGOs together? (1) (2) (3) (4) (5) 420 140 120 390 None of these = 129

Solution: Required average = Ans: (5)

4. The total number of trees planted by NGO A and NGO B together in Bihar was approximately what per cent of the total number of trees planted by NGO-B and NGO-C together in Punjab? (1) (2) (3) (4) (5) 85 90 105 110 95 100 = 100 95%

Solution: Required percentage = Ans: (5)

226

IBPS PO EXAM 2013 : Quantitative Aptitude

5. What was the ratio of the number of trees planted by NGO B in Tamil Nadu, number of trees planted by NGO C in Assam and the number of trees planted by NGO A in Assam? (1) (2) (3) (4) (5) 5: 3 : 6 5: 6 : 3 6: 4 : 5 6: 5 : 3 None of these

Solution: Required ratio: 180: 120 + 150 = 6: 4: 5 Ans: (3) Direction (Q. 6 10): Study the graph carefully and answer the questions that follow: Per cent profit made by two companies over the years

6. If in the year 2006 the expenditures incurred by company P and Q were same, what was the ratio of the income of company Q to that of company P in that year? (a) 26 : 27 (b) 27 : 26 (c) 24 : 25 (d) 25 : 24 (e) None of these Answer: (c) 227 IBPS PO EXAM 2013 : Quantitative Aptitude

7. If the amount of profit earned by company Q in the year 2007 was Rs. 2.4 lakhs, what was its expenditure in that year? (a) Rs. 13 lakhs (b) Rs. 15 lakhs (c) Rs. 24 lakhs (d) Rs. 16 lakhs (e) Rs. 20 lakhs Answer: (d) 8. What is the average per cent profit earned by company P over all the years together? (a) 30 (b) 25 (c) 40 (d) 33 (e) None of these Answer: (b) 9. If in the year 2009, the incomes of both the companies P and Q were same, what was the ratio of the expenditure of company P to the expenditure of company Q in the same year? (a) 26 : 23 (b) 23 : 26 (c) 24 : 25 (d) 25 : 24 (e) None of these Answer: (a) 10. What is the ratio of the amount of profit earned by company A to that by company B in the year 2010? (a) 27 : 24 (b) 24 : 27 (c) 23 : 24 (d) 24 : 23 (e) None of these Cannot be determined Answer: (e)

228

IBPS PO EXAM 2013 : Quantitative Aptitude

Practice Set (Data Interpretation-Line Graphs)


Directions (1-5): Study the following graph carefully and answer the questions given below: (Allahabad Bank PO Exam: 2010) Profit earned by Three Companies over the years (Rs. in crores)

1. What was the average profit earned by all the three companies in the year 2008? (a) Rs. 300 crore (b) Rs. 400 crore (c) Rs. 350 crore (d) Rs. 520 crore (e) None of these 2. In which of the following years was the difference between the profits earned by company B and company A the minimum? (a) 2003 (b) 2004 229 IBPS PO EXAM 2013 : Quantitative Aptitude

(c) 2005 (d) 2008 (e) None of these 3. In which of the following years was the total profit earned by all three comapnies together the highest? (a) 2004 (b) 2007 (c) 2008 (d) 2009 (e) None of these 4. What was the approximate percentage increase in the profit earned by Company A from 2006 to 2007? (a) 36 (b) 24 (c) 40 (d) 20 (e) 54 5. What was the difference between the profit earned by company A in 2004 and the profit earned by company C in 2009? (a) Rs. 50 crore (b) Rs. 1 crore (c) Rs. 100 crore (d) Rs. 200 crore (e) None of these

230

IBPS PO EXAM 2013 : Quantitative Aptitude

Direction (Q. 6 10): Study the given graph carefully and answer the questions that follow: The line diagram shows the cost of production and profit of six companies for the year 201112. (The figures are in 'Lakhs'). Revenue = Cost of Production + Profit.

6. The ratio of profits of company B and D to the profits of A and E is: (a) (b) (c) (d) (e) 2:3 10 : 9 3:2 10 : 7 None of these

7. The profit of company C is what percentage of the revenue of company F? (a) (b) (c) (d) (e) 20% 25% 30% 35% None of these

8. The revenue of company C is how many times of company E's profit? (a) (b) (c) (d) (e) 5.5 5.25 5.75 5 None of these

231

IBPS PO EXAM 2013 : Quantitative Aptitude

9. Which company has the maximum percentage of profit? (a) (b) (c) (d) (e) C D E F None of these

10. What is the average profit of the last five companies (B, C, D, E and F)? (a) (b) (c) (d) (e) Rs. 500 Rs. 5,000 Rs. 50,000 Rs. 4,66,667 None of these

232

IBPS PO EXAM 2013 : Quantitative Aptitude

Data Interpretation-Line Graphs Practice Set (Answers)


1. 2. 3. 4. 5. 6. 7. 8. 9. (b) (e) (d) (a) (c) (d) (a) (c) (b)

10. (e)

233

IBPS PO EXAM 2013 : Quantitative Aptitude

Chapter: Data Interpretation-Bar Graphs


In a bar diagram, information is presented by means of rectangles, whose lengths indicate the quantity of the variable which the bar is representing. The following points are important: 1) All bars are in the form of rectangles and the width of the bars is uniform throughout the diagram. 2) The height of each bar is proportional to the frequency of the variable. 3) The gap between various bars is uniform. 4) The base line of all the bars is the same. 5) The bars can be either horizontal or vertical depending on the space available. Example: The expenditure of a company under different heads (in thousands of rupees) is given below: Head Expenditure(in thousands of rupees) 400 100 150 200 300

Salary of employees Travelling allowance (TA) Rent Equipment Miscellaneous

Draw a bar chart to depict the above data.

234

IBPS PO EXAM 2013 : Quantitative Aptitude

Example: The following bar diagram represents the percentages of total expenditure incurred by a state during the years 1981- 90 for different items. In each bar the blue portion stands for the expenditure during the first five years and the red portion stands for the next five years. Study the graph and answer questions 1-5.

1) Which of the items listed below accounts for the maximum expenditure during the year 1981 to 1985? 235 IBPS PO EXAM 2013 : Quantitative Aptitude

(a) Communication (b) education (c) health (d) housing 2) Which of the items listed below accounts for the maximum expenditure during 1986 to 1990? (a) Agriculture (b) communication (c) education (d) health 3) The amount of expenditure on Agriculture is approximately what proportion of that on industry during the year 1986-90? (a) 1/5 (b) (c) 1/3 (d) data inadequate 4) If the total expenditure on housing is Rs. 610 crores during 1981-85, the total expenditure on industry during the same period would (approximately) (a) Rs 2440 crores (b) Rs 1220 crores (c) Rs 4620 crores (d) none of these 5) Out of every 10,000 rupees spent during 1981-90 approximately, how much was spent during the years 1981-85 on housing? (a) Rs 1400 (b) Rs 700 (c) Rs 1000 (d) Rs 2800 Answer: 1) (d) out of the items listed in the question, clearly maximum expenditure during 1981-85 is on housing. 2) (a) out of the items listed in the question, clearly the maximum expenditure during 1986-90 is on agriculture. 3) (a) expenditure on agriculture during 1986-90 a. = (15-10)% of total expenditure = 5x/100 = x/20 b. Expenditure on industry during 1986-90 c. = (52.5-27.5)% of total expenditure = 25x/100 =x/4. d. Required ratio = x/20 : x/4 = 1:5 4) (d) expenditure on housing during 1981-85 = 10% of total expenditure. a. Let the total expenditure be Rs. x. b. Then , 105 of x = 610 crores or 10x/100 =610 crores c. x=6100 crores d. total expenditure on industry during 1981-85 = 25% of 6100 crores e. = Rs 1525 crores. 5) (c) 20% of total expenditure during 1981-90 was spent on housing. a. Expenditure on housing during 1981-90 for a total expenditure of Rs 10000 = (2010000/100) = Rs 2000 b. Ratio of expenditure on housing during 1981-85 and that during 1986-90= 10%/(20-10)% = 1/1 c. Expenditure on housing during 1981-85 = Rs 1000

236

IBPS PO EXAM 2013 : Quantitative Aptitude

Solved Examples (Data Interpretation-Bar Graphs)


Directions (Q. 1-5): Study the following graph and answer the questions given below:

1. Out of the total number of students who opted for the given three subjects, in the year 2009, 38% were girls. How many boys opted for Mathematics in the same year? (1) 1322 (2) 1332 (3) 1312 (4) Cannot be determined (5) None of these Solution: Number of students who opted for all three subjects in 2009 = (20 + 20 + 5) thousand = 45000 Number of boys = = 27900

Since, we do not know the number of girls in Mathematics, number of boys opted for Mathematics cannot be determined. Ans : (4)

237

IBPS PO EXAM 2013 : Quantitative Aptitude

2. If the total number of students in the University in the year 2007 was 455030, then, the total number of students who opted for the given three subjects were approximately what percent of the total students? (1) 19 (2) 9 (3) 12 (4) 5 (5) 23 Solution: Required percentage = = Ans: (2) 3. What is the total number of students who opted for Hindi and who opted for Mathematics in the years 2006, 2007 and 2009 together? (1) 97000 (2) 93000 (3) 85000 (4) 96000 (5) None of these Solution: Required number of students = (5 + 35 + 15 + 15 + 20 + 5) x 1000 = 95 x 1000 = 95000 Ans: (5) 4. The total number of students who opted for Mathematics in the years 2005 and 2008 together are approximately what percent of the total number of students who opted for all three subjects in the same years? (1) 38 (2) 28 (3) 42 (4) 32 (5) 48 100 9 100

238

IBPS PO EXAM 2013 : Quantitative Aptitude

Solution: Required percentage = 100

= Ans: (4)

100 =

100

32

5. What is the respective ratio between the number of students who opted for English in the years 2006 and 2008 together and the number of students who opted for Hindi in the year 2005 and 2009 together? (1) 11: 5 (2) 12: 7 (3) 11: 7 (4) 12:5 (5) None of these Solution: Required ratio = (25 + 30): (5+20) = 55:25 = 11:15 Ans: (1) Directions (Q. 6-10): Study the following graph carefully to answer the questions that follow: Monthly income (Rs in thousand) of three different persons in six different years (IBPS RRB Grade A Officers Exam 2012)

239

IBPS PO EXAM 2013 : Quantitative Aptitude

6. What was the difference between the total monthly salary of Arun in all the years together and Suman's monthly income in the year 2007? (1) (2) (3) (4) (5) Rs. 1.24 Iakh Rs. 1.14 Iakh Rs. 11.4 lakh Rs. 12.4 lakh None of these

Solution. Arun monthly income in all year together

Suman's monthly income in the year 2007 = 15 thousand Difference = 129 15 = 114 = 114 1000 = 114000 lakh Ans: 2 7. What is the ratio of Arun's monthly income in the year 2006, Suman's monthly income in the year 2007 and Jyoti's monthly income in the year 2005? (1) (2) (3) (4) (5) 6:3:5 6:4:5 5:6:4 5:4:7 None of these

Solution. Ratio = Arun : Suman : Jyoti 18 : 15 : 9 6:5:3 Ans: 5

240

IBPS PO EXAM 2013 : Quantitative Aptitude

8. In which year was the difference between Jyoti's and Arun's monthly income the second highest? (1) (2) (3) (4) (5) 2005 2006 2007 2009 2010

Solution. Difference in 2005 14 9 = 5 2006 18 10 = 8 2007 23 18 = 5 2008 27 21 = 6 2009 27 26 = 1 2010 Ans: 2 9. The monthly income of Suman in the year 2009 was approximately what percentage of the monthly income of Jyoti in the year 2010? (1) (2) (3) (4) (5) 72 89 83 67 95 35 26 = 9

Solution. Monthly income of Suman in 2009 = 29000 Monthly income of Jyoti in 2010 = 35000

Ans: 3

241

IBPS PO EXAM 2013 : Quantitative Aptitude

10. What was the percentage increase in the monthly income of Jyoti in the year 2008 as compared to the previous year? (1) (2) (3) (4) (5) 50 150 160 60 None of these

Solution. Ans: 1

242

IBPS PO EXAM 2013 : Quantitative Aptitude

Practice Set (Data Interpretation-Bar Graphs)


Directions (Q. 1-5): Study the following graph carefully to answer the questions that follow: (Corporation Bank PO 2011)

1. What is the percentage increase in the number of runs scored by Team B in Match 4 as compared to that in the previous match (Match 3)? (1) (2) (3) (4) (5) 40 30 20 25 None of these 243 IBPS PO EXAM 2013 : Quantitative Aptitude

2. What is the ratio of the number of runs scored by Team A in Match 2 to the number of runs scored by Team C in Match 6? (1) (2) (3) (4) (5) 5:4 2:5 2:3 3:4 None of these

3. What is the average number of runs scored by Team B in all the matches together? (1) (2) (3) (4) (5) 250 275 200 300 225

4. The number of runs scored by all the teams together in Match 3 is approximately what percentage of the total runs scored by Team C in all the matches together? (1) (2) (3) (4) (5) 37 57 52 47 42

5. In which match is the total runs scored by all the teams together the second highest? (1) (2) (3) (4) (5) Match 2 only Match 6 only Match 4 only Both Match 2 and Match 6 Both Match 2 and Match 4

244

IBPS PO EXAM 2013 : Quantitative Aptitude

Directions (Q. 6-10): Study the given graph carefully to answer the questions that follow:

6. What is the average number of people using mobile service M for all the years together? 1) 16

2) 14444

3) 16666

4) 14 5) None of these 7. The total number of people using all the three mobile services in the year 2007 is what per cent of the total number of people using all the three mobile services in the year 2008? (rounded off to two digits after decimal) 1) 89.72 2) 93.46 3) 88.18 245 IBPS PO EXAM 2013 : Quantitative Aptitude

4) 91.67 5) None of these 8. The number of people using mobile service N in the year 2006 forms a proximately what per cent of the total number of people using all the three mobile services in that year? 1) 2) 3) 4) 5) 18 26 11 23 29

9. What is the ratio of the number of people using mobile service L in the year 2005 to that of those using the same service in the year 2004? 1) 2) 3) 4) 5) 8: 7 3:2 19: 13 15: 11 None of these

10. What is the total number of people using mobile service M in the years 2008 and 2009 together? 1) 2) 3) 4) 5) 35,000 30,000 45,000 25,000 None of these

246

IBPS PO EXAM 2013 : Quantitative Aptitude

Data Interpretation-Bar Graphs Practice Set (Answers)

1) 2) 3) 4) 5) 6) 7) 8) 9) 10)

3 3 1 5 4 5 4 1 2 3

247

IBPS PO EXAM 2013 : Quantitative Aptitude

Chapter: Data Interpretation-Pie Diagram


In a pie chart, the values of different components of a frequency distribution are represented by the sectors of a circle. These sectors are so constructed that the area of each sector is proportional to the corresponding value of the component. Since the sum of all the central angles is 360 degrees, we have Central angle of a component = degrees

Example: The following pie diagram shows the expenditure incurred on the preparation of a book by a publisher, under various heads. A: Paper 20, B: Printing 25%, C: Binding, Canvassing, Designing etc 30% D: Miscellaneous 10% E: Royalty 15% Study the diagram carefully and answer the questions 1-5:

1) What is the angle of pie diagram showing the expenditure incurred on paying the royalty? (a) 15 degrees (b) 24 degrees (c) 48 degrees (d) 54 degrees

248

IBPS PO EXAM 2013 : Quantitative Aptitude

2) The marked price of a book is 20% more than the C.P. If the marked price of the book be Rs 30, what is the cost of paper used in a single copy of the book? (a) Rs. 6 (b) Rs. 5 (c) Rs 4.50 (d) Rs 6.50 3) Which two expenditures together will form an angle of 108 degrees at the centre of the pie diagram: (a) A and E (b) B and E (c) A and D (d) D and E 4) If the difference between two expenditures be represented by 18 degrees in the piediagram, these expenditures are : (a) B and E (b) A and C (c) B and D (d) none of these

Answers:
1) (d) Angle representing royalty D = (15360/100)degrees = 54 degrees 2) (b) C.P. of a book = Rs ( 10030/120) = Rs 25 Cost of paper = Rs (2025/100) = Rs 5 3) (c) angle A = (20360/100)degrees =72 degrees Angle B = (25360/100) degrees = 90 degrees Angle C = (30360/100)degrees = 108 degrees Angle D = (10360/100) degrees = 36 degrees Angle E = (15360/100) degrees = 54 degrees Thus, A and D together will form an angle of 108 degrees. 4) (d) These expenditures are A and B; Band C; D and E; and A and E.

249

IBPS PO EXAM 2013 : Quantitative Aptitude

Solved Examples (Data Interpretation-Pie Diagram)


Direction (Q. 1-5): Study the following pie-chart and answer the questions given below: Preferences of students among six beverages in terms of degree of angle in the pie-chart Total No. of students = 6800

1. What is the difference between the total number of students who prefer Beverage A and C together and the total number of students who prefer beverage D and F together? (1) 959 (2) 955 (3) 952 (4) 954 (5) None of these Solution: Difference of corresponding angles = (122.4 + 21.6)0 - (79.2 + 14.4)0 = 50.40 required difference = Ans: 3 x 6800 = 952

250

IBPS PO EXAM 2013 : Quantitative Aptitude

2. What is the ratio of the number of students who prefer beverage F and the number of students who prefer beverage A? (1) 3: 11 (2) 3: 13 (3) 6: 11 (4) 5: 11 (5) None of these Solution: Required Ratio = 21.6: 79.2 = 3: 11 Ans: 1 3. The number of student who prefer beverage E and F together is what per cent of the total of student? (1) 18 (2) 14 (3) 26 (4) 24 (5) None of these Solution: Required percentage = ( Ans: 4 4. The number of students who prefer beverage C are approximately what percent of the number of students who prefer Beverage D? (1) 7 (2) 12 (3) 18 (4) 22 (5) 29 Solution: Required percentage = Ans: 2 5. How many students prefer beverage B and beverage E together? (1) 2312 (2) 2313 (3) 2315 (4) 2318 251 IBPS PO EXAM 2013 : Quantitative Aptitude 100 = 11.76 12% ) 100 =24%

(5) None of these Solution: Number of students who prefer beverages B and E together = Ans: 1 Directions (Q. 6-10): Study the following pie-chart and answer the following questions. (IBPS RRB Group A Officers Exam 2012) Percentagewise distribution of teachers in six different universities. Total number of teachers = 6400 68000 = = 2312

6. The number of teachers in University B is approximately what per cent of the total number of teachers in University D and University E together? (1) (2) (3) (4) (5) 55 59 49 45 65

Solution: Number of teachers in University B

252

IBPS PO EXAM 2013 : Quantitative Aptitude

Number of teachers in University D Number of teachers in University E Required percentage

Ans: 3 7. If twenty five per cent of the teachers in University C are females, what is the number of male teachers in University C? (1) (2) (3) (4) (5) 922 911 924 912 None of these

Solution: Number of teachers in University C Number of female teachers in University C

Number of male teachers in University C = 1216 304 = 912 Ans: 4 8. The difference between the total number of teachers in University A, University B and University C together and the total number of teachers in University D, University E and University F together is exactly equal to the number of teachers of which University? (1) (2) (3) (4) University A University B University C University D 253 IBPS PO EXAM 2013 : Quantitative Aptitude

(5) University F Solution: Number of teachers in University A Number of teachers in University B Number of teachers in University C Number of teachers in University D Number of teachers in University E Number of teachers in University F Difference = 3392 3008 = 384 Quicker method: Difference = (D + E + F)% (A + B + C)% = (53 47) = 6% 6% of 6400 = 384 Hence, University of D is equal to 6%. Ans: 4 9. If one-thirty sixth of the teachers from University F are professors and the salary of each professor is Rs. 96000, what will be the total salary of all the professors together from University F? (1) (2) (3) (4) (5) Rs. 307.2 lakh Rs. 32.64 lakh Rs. 3.072 lakh Rs. 3.264 lakh None of these

Solution: Number of teachers in University F 254 IBPS PO EXAM 2013 : Quantitative Aptitude

Number of professors in University F Total Salary of professors in University F = 32 96000 = 30.72 lakh Ans: 5 10. What is the average number of teachers in University A, University C, University D and University F together? (1) (2) (3) (4) (5) 854 3546 3456 874 None of these

Solution: Average Ans: 5

255

IBPS PO EXAM 2013 : Quantitative Aptitude

Practice Set (Data Interpretation-Pie Diagram)


Directions (1-5): Study the following pie-charts carefully and answer the questions given below: Discipline-wise Breakup of the Number of candidates appeared in Interview and Disciplinewise Break up of the Number of candidates selected by an organisation Discipline-wise Breakup of Number of candidates appeared in Interview Total Number of Candidates Appeared In the Interview = 25780 Percentage Distribution

Discipline-wise Break-up of Number of candidates selected after Interview by the organization Total Number of Candidates selected After Interview = 7390 Percentage Distribution

256

IBPS PO EXAM 2013 : Quantitative Aptitude

1. What was the ratio of the number of candidates appeared . in interview from other disciplines and the number of candidates selected from Engineering discipline respectively (rounded off to the nearest integer)? 1) 2) 3) 4) 5) 3609: 813 3094: 813 3094: 1035 4125: 1035 3981: 767

2. The total number of candidates appeared in interview from Management and other discipline was what percentage of number of candidates appeared from Engineering discipline? 1) 2) 3) 4) 5) 50 150 200 Cannot be determined None of these

3. Approximately what was the difference between the number of candidates selected from Agriculture discipline and number of candidates selected from Engineering discipline? 1) 2) 3) 4) 5) 517 665 346 813 296

4. For which discipline was the difference in number of candidates selected to number of candidates appeared in interview the maximum? 1) 2) 3) 4) 5) Management Engineering Science Agriculture None of these

5. Approximately what was the total number of candidates selected from Commerce and Agricultural discipline together? 1) 1700 257 IBPS PO EXAM 2013 : Quantitative Aptitude

2) 3) 4) 5)

1800 2217 1996 1550

Directions (Q.6-10): Study the following pie-chart carefully to answer these questions. (Central Bank of India (PO) 2010) Percentagewise Distribution of teachers who teach six different subjects Total number of Teachers = 1800 Percentage of teachers

6. If two-ninths of the teachers who teach Physics are female, then the number of male Physics teachers is approximately what percentage of the total number of teachers who teach Chemistry? 1) 57 2) 42 3) 63 4) 69 5) 51 7. What is the total number of teachers teaching Chemistry, English and Biology? 1) 1, 226 258 IBPS PO EXAM 2013 : Quantitative Aptitude

2) 1, 116 3) 1, 176 4) 998 5) None of these 8. What is the difference between the total number of teachers who teach English and Physics together and the total number of teachers who teach Mathematics and Biology together? 1) 352 2) 342 3) 643 4) 653 5) None of these 9. What is the ratio of the number of teachers who teach Mathematics to the number of teachers who teach Hindi? 1) 13:7 2) 7:13 3) 7: 26 4) 8: 15 5) None of these 10. If the percentage of Mathematics teachers is increased by 50 per cent and the percentage of Hindi teachers decreased by 25 per cent then what will be the total number of Mathematics and Hindi teachers together? 1) 390 2) 379 3) 459 4) 480 5) None of these 259 IBPS PO EXAM 2013 : Quantitative Aptitude

Direction (Q. 11 15): Study the given pie-charts carefully and answer the questions that follow: Discipline-wise breakup of the number of candidates appeared in Interview and Discipline-wise breakup of the candidates selected by and organisation. Total number of candidates appeared in the interview = 25,600 and total number of candidates selected after interview = 7,500.

11. What was the ratio of the number of candidates appeared in interview from other disciplines and the number of candidates selected from art disciplines? (a) 256 : 125 (b) 125 : 256 (c) 125 : 216 (d) Cannot be determined (e) None of these 12. The total number of candidates appeared in interview from Management and Art disciplines was what per cent of the number of candidates from Engineering discipline? (a) 66.67 260 IBPS PO EXAM 2013 : Quantitative Aptitude

(b) 75 (c) 80 (d) 120 (e) 150 13. What was the difference between the number of candidates selected from Science discipline and the number of candidates selected from Commerce discipline? (a) 1,000 (b) 1,100 (c) 1,200 (d) 1,250 (e) None of these 14. From which discipline was the difference in number of candidates selected to number of candidates appeared in interview the maximum ? (a) Management (b) Engineering (c) Commerce (d) Science (e) Art 15. What was the total number of candidates selected from Commerce and Art discipline together? (a) 1,800 (b) 1,950 (c) 2,100 (d) 2,250 (e) 2,400 261 IBPS PO EXAM 2013 : Quantitative Aptitude

Data Interpretation- Pie Diagram Practice Set (Answers)


1. 2. 3. 4. 5. 6. 7. 8. 2 2 5 3 1 1 2 2 9. 10. 11. 12. 13. 14. 15. 5 3 a e c d b

262

IBPS PO EXAM 2013 : Quantitative Aptitude

Chapter: Data Interpretation-Case lets


Direction for questions 1 to 5: Answer these questions on the basis of the following information. Shekhar bought 10 acres of land for Rs.250000 in 2011. That year he cultivated Sugarcane and Soya bean in the 10 acres with the ratio of area under Sugarcane and Soya bean being 5:4. The profit obtained from Sugarcane and Soya bean was in the ratio 3:2 with the total profit being Rs.58500. This was 15% of the amount he invested in cultivation that year. The next year he again cultivated Sugarcane and Soya bean, with the areas being same as before and reaped a profit of Rs.66000 in total with that from Sugarcane and Soya bean being in the ratio 8:7 but his return on his investment that year was only 14%. 1. What is the amount invested by Shekhar for cultivation in 2011? (a) Rs.356000 (b) Rs.374800 (c) Rs.380000 (d) Rs. 390000 Solution: (d) Shekhar had a profit of Rs.58500 and this profit was 15% of the money he invested, his investment was

2. What is the profit obtained by Shekhar by cultivating Sugarcane in 2011? (a) Rs.43800 (b) Rs.35100 (c) Rs.36200 (d) None of these Solution: (b) The profit obtained by Shekhar by cultivating Sugarcane in 2012 =

263

IBPS PO EXAM 2013 : Quantitative Aptitude

3. What is the profit obtained by cultivating Soya bean in 2012? (a) Rs.30800 (b) Rs.36100 (c) Rs.24200 (d) None of these Solution: (a) The profit obtained by cultivating Soya bean in 2012 = 4. What is the ratio of the profit obtained from Sugarcane and Soya bean in the two years together? (a) 89 : 79 (b) 167 : 211 (c) 703 : 542 (d) None of these Solution: (c) The profit obtained in 2011 from Sugarcane = Rs.35100 The profit obtained in 2012 from Sugarcane = 66000 - 30800 = Rs.35200 Total profit from Sugarcane = Rs.70300 Total profit in the two years = Rs.58500 + Rs.66000 = Rs.124500 Therefore, Profit from Soya bean Rs.54200. Now, the required ratio = 70300: 54200 = 703 : 542.

264

IBPS PO EXAM 2013 : Quantitative Aptitude

5. What is the approximate amount invested by Shekhar for cultivation in 2012? (a) Rs.428500 (b) Rs.471400 (c) Rs.495300 (d) Rs.518650 Solution: (b) Here the profit of 66,000 is 14% of the amount invested, therefore, the invested =

265

IBPS PO EXAM 2013 : Quantitative Aptitude

Solved Examples (Data Interpretation-Caselets)


Directions: (1-5) Study the following information and answer the questions that follow: (IBPS CWE PO MT 2012) The premises of a bank are to be renovated. The renovation is in terms of flooring. Certain areas are to be floored either with marble or wood. All rooms/halls and pantry are rectangular. The area to be renovated comprises of a hall for customer transaction measuring 23 m by 29 m, branch managers room measuring 13 m by 17 m, a pantry measuring 14 m by 13 m, a record keeping cum server room measuring 21 m by 13 m and locker area measuring 29 m by 21 m. The total area of the bank is 2000 square meters. The cost of wooden flooring is f 170/- per square meter and the cost of marble flooring is Rs. 190/- per square meter. The locker area, record keeping cum server room and pantry are to be floored with marble. The branch manager's room and the hall for customer transaction are to be floored with wood. No other area is to be renovated in terms of flooring. 1. What is the respective ratio of the total cost of wood en flooring to the total cost of marble flooring? (1) 1879: 2527 (2) 1887: 2386 (3) 1887: 2527 (4) 1829: 2527 (5) 1887: 2351 Solution: Total flooring area with marble = locker area + record keeping + pantry = 182+273 +609 = 1064 sqm Cost of flooring = 1064 190 Total flooring area with wooden = Branch Manager room + Hall = 221 + 667 = 888 sqm Cost of flooring = 888 170 Ratio= 888 170: 1064 190 = 888 17: 1064 19 = 15096 : 20216 = 1887: 2527 Ans: 3 266 IBPS PO EXAM 2013 : Quantitative Aptitude

2. If the four walls and ceiling of the branch managers room (The height of the room is 12 meters) are to be painted at the cost off 190/- per square meter, how much will be the total cost of renovation of the branch manager's room including the cost of flooring? (1) Rs. 1, 36,800/(2) Rs. 2, 16,660/(3) Rs. 1, 78,790/(4) Rs. 2, 11,940/(5) None of these Solution: Cost of flooring of branch manager room =221 170= Rs. 37570 Cost of painting = [2(17 12+ 13 12)+ 13 x 17] 190 = [2( 204 + 156) + 221] 190= (2 360 + 221) 190 = (720 + 221) 190 = 941 190 = Rs. 178790 Total cost = 178790 + 31570 = Rs.216360 Ans: (5) 3. If the remaining area of the bank is to be carpeted at the rate of Rs. 110/- per square meter, how much will be the increment in the total cost of renovation of bank premises? (1) Rs. 5,820/(2) Rs. 4,848/(3) Rs. 3,689/(4) Rs. 6,890/(5) None of these Solution: Total area of bank = 2000 sqm Total flooring area = 1952 sqm Remaining area = 2000 - 1952 = 48 sqm Cost of carpeting = 48 110 = Rs.5280 Ans: (5) 4. What is the percentage area of the bank that is not to be renovated? (1) 2.2 % (2) 2.4 % (3) 4.2 % (4) 4.4 % 267 IBPS PO EXAM 2013 : Quantitative Aptitude

(5) None of these Solution: Area not to be renovated = 48 sq m Reqd % = l00 = 2.4% Ans: 2 5. What is the total cost of renovation of the hall for customer transaction and the locker area? (1) Rs. 2, 29,100 (2) Rs. 2, 30,206 (3) Rs. 2, 16,920 (4) Rs. 2, 42,440 (5) None of these Solution: Cost of renovation of hall + locker area = 667 170 + 609 190 = 113390 + 115710= Rs. 229100 Ans: (1) Directions (Q. 6 10): Study the given information carefully to answer the questions that follow: An organization consists of 3500 employees working in different departments, viz HR, Marketing, IT, Production and Accounts. The ratio of male to female employees in the organisation is 3 : 2. 8% of the males work in the HR department. 22% of the female work in the account department. The ratio of males to females working in the HR department is 3 : 5. Oneseventh of the females work in the IT department. 46% of the males work in the Production department. The number of females is one-sixth of the males working in the same. The remaining females work in the Marketing department. The total number of employees working in the IT department is 375. 22% of the males work in the Marketing department and remaining work in the Account department. 6. The number of males working in the Account department forms approximately what per cent of the total number of males in the organisation? (a) 6 (b) 8 (c) 10 268 IBPS PO EXAM 2013 : Quantitative Aptitude

(d) 11 (e) 12 Answer: (a) 7. How many females work in Production department? (a) 140 (b) 200 (c) 180 (d) 160 (e) None of these Answer: (e) 8. The total number of employees working in the Account department forms approximately what per cent of the total number of female employees in the organisation? (a) 28 (b) 32 (c) 29 (d) 31 (e) 30 Answer: (d) 9. The ratio of the numbers of females working in IT department to the numbers of males working in the same department is (a) 15 : 8 (b) 1 : 2 (c) 8 : 15 (d) 2 : 1 (e) 7 : 11 Answer: (c) 10. What is the total number of employees working in the Marketing and Production departments together? (a) 1900 (b) 2040 (c) 2020 (d) 2031 (e) 2042 Answer:(b)

269

IBPS PO EXAM 2013 : Quantitative Aptitude

PracticeSet-(Data Interpretation- Caselets)


Directions (Q. 1-5): Study the information carefully to answer the questions that follow. A company produced five different products, viz mobile phone, pen drive, calculator, television and washing machine. The total number of all the five products is 1650.24% of the total number of products is mobile phones. One-sixth of the total number of products is pen drives. 14% of the total number of products is calculators. Remaining products are either television or washing machine. The number of washing machines is 50 more than the number of televisions produced. (IBPS RRB Grade Officer Exam 2012) 1. What is the ratio of the number of washing machines to the number of calculators produced by the company? (1) (2) (3) (4) (5) 17:11 19:11 11:17 19: 13 None of these 3. The number of televisions produced is approximately what per cent of the total number of calculators and washing machines produced together? (1) (2) (3) (4) (5) 63 55 59 51 67

4. What is the difference between the total number of televisions and mobile phones together and the number of calculators produced? (1) 534 (2) 524 (3) 511 (4) 523 (5) None of these 5. What is the total number of pen drives, calculators and washing machines produced by the company? (1) 907 (2) 917 (3) 925 (4) 905 (5) None of these

2. If 24 per cent of the pen drives are defective, what is the number of pen drives which are not defective? (1) (2) (3) (4) (5) 209 215 219 225 None of these 270

IBPS PO EXAM 2013 : Quantitative Aptitude

Directions (Q. 6-10): Study the following information carefully to answer the questions that follow: (RBI GradeB Officers Exam 2011) There are two trains, Train A and Train B. Both trains have four different types of coaches, viz General, Sleeper, First Class and AC. In Train A, there are total 700 passengers. Train B has thirty per cent more passengers than Train A. Twenty per cent of the passengers of Train A are in General Coach. One-fourth of the total number of passengers of Train A are in AC coach. Twenty three per cent of the passengers of Train A are in Sleeper Coach. Remaining passengers of Train A are in First Class Coach. The total number of passengers in AC Coach in both the trains together is 480. Thirty per cent of the number of passengers of Train B are in Sleeper Coach. Ten per cent of the total passengers of Train B are in First Class Coach. The remaining passengers of Train B are in General Coach. 6. What is the ratio of the number of passengers in First Class Coach of Train A to the number of passengers in Sleeper Coach of Train B? (1) (2) (3) (4) (5) 13 : 7 7 : 13 32 : 39 Data Inadequate None of these

(3) 435 (4) 445 (5) None of these 8. What is the difference between the number of passengers in the AC Coach of Train A and the total number of passengers in Sleeper and First Class Coach together of Train B? (1) (2) (3) (4) (5) 199 178 187 179 None of these

9. The total number of passengers in General Coaches of both the trains together is approximately what percentage of the total number of passengers in Train B? (1) (2) (3) (4) (5) 35 42 46 38 31

10. If the cost per ticket of First Class coach is Rs.450, what will be the total amount generated from First Class Coach of Train A? (1) (2) (3) (4) (5) Rs.1, 00, 080 Rs.1, 08, 000 Rs.1, 00, 800 Rs.10, 800 None of these

7. What is the total number of passengers in the General Coach of Train A and the AC Coach of Train B together? (1) 449 (2) 459 271

IBPS PO EXAM 2013 : Quantitative Aptitude

Directions (Q. 11-15): Study the information carefully to answer the questions that follow: A company produces five different products, viz Television, Refrigerator, Mobile Phone, Oven and Water Heater. The total number of all the five products manufactured is 1200. 15 per cent of the total number of products are Televisions. Three-tenths of the total number of products are Refrigerators. The number of Mobile Phones manufactured is 40 more than the number of Televisions. 22 per cent of the total number of products is Oven and the remaining number of products is Water Heaters. (Corporation Bank PO 2011) 11. What is the total number of Refrigerators and Water Heaters? (1) (2) (3) (4) (5) 436 476 576 536 None of these

(2) (3) (4) (5)

Rs. 21,68,400 Rs. 2,16,48,000 Rs. 2,16,840 None of these

14. What is the difference between the number of Refrigerators and the number of Mobile Phones manufactured? (1) (2) (3) (4) (5) 160 140 120 130 None of these

15. If 25 per cent of the number of Ovens are defective, what is the number of nondefective Ovens? (1) 176 (2) 188 (3) 198 (4) 186 (5) None of these

12. The number of Mobile Phones is approximately what per cent of the number of Televisions? (1) 115 (2) 140 (3) 135 (4) 130 (5) 120 13. If the cost of one Oven is Rs. 8,200, what is the cost of all the Ovens manufactured by the company? (1) Rs. 2,16,480 272 IBPS PO EXAM 2013 : Quantitative Aptitude

Directions-(Q. 16-20): Study the information carefully to answer the questions that follows: In a ship there are 1200 passengers. 18 per cent of the total number of passengers is from Britain. Two- fifth of the total number of passengers is from South Africa 6 per cent of the total number of passengers is from Madagascar. Remaining number of passengers is from India. 25 per cent of the number of passengers from Britain is females. Half the numbers of passengers from South Africa are male. There is no female passenger from Madagascar. Twothird of the number of passengers from India are females. (Allahabad Bank Probationary Officers Exam 2011) 16. What is the ratio of the number of passengers from Madagascar, number of female passengers from South Africa and the total number of passengers from India? (1) (2) (3) (4) (5) 2: 5 : 18 3: 10 : 18 3: 11 : 18 2: 18 : 5 None of these

(1) (2) (3) (4) (5)

111 115 120 125 131

18. What is the average number of male passengers from all the four countries? (1) (2) (3) (4) (5) 154.5 164.5 145 164 None of these

19. What is the difference between the number of male passengers from Madagascar and the number of male passengers from India? (1) (2) (3) (4) (5) 64 82 74 72 None of these

20. What is the total number of male from Britain passengers female passengers from India together? (1) (2) (3) (4) (5) 340 420 350 460 None of these

17. The number of male passengers from South Africa is approximately what percentage of the total number of passengers from Britain?

273

IBPS PO EXAM 2013 : Quantitative Aptitude

Data Interpretation-Caselets Practice Set (Answers)


1) 2) 3) 4) 5) 6) 7) 8) 9) 10) 2 1 2 5 4 3 4 5 2 3 11) 12) 13) 14) 15) 16) 17) 18) 19) 20) 4 5 5 2 3 2 1 1 4 5

274

IBPS PO EXAM 2013 : Quantitative Aptitude

Solved Examples (Data Interpretation-Miscellaneous)


Directions: (1-5) Study the following graph and table carefully and answer the questions given below: Time Taken To Travel (In Hours) By Six Vehicles On Two Different Days

Distance covered (in kilometers) by six vehicles on each day

(IBPS CWE PO MT 2012) 1. Which of the following vehicles travelled at the same speed on both the days? (1) Vehicle A (2) Vehicle C (3) Vehicle F 275 IBPS PO EXAM 2013 : Quantitative Aptitude

(4) Vehicle B (5) None of these Solution: The speed of Vehicle B on both the days is 43 km/hr. Ans: (4) 2. What was the difference between the speed of vehicle A on day 1 and the speed of vehicle C on the same day? (1) 7 km/hr (2) 12 km/hr (3) 11 km/hr (4) 8 km/hr (5) None of these Solution: Speed of A on 1st day = 52 km/hr Speed of C on 1st day = 63 km/hr Difference = 65 - 52 = 11 km/hr Ans: 2 3. What was the speed of vehicle C on day 2 in terms of meters per second? (1) 15.3 (2) 12.8 (3) 11.5 (4) 13.8 (5) None of these Solution: Speed of Vehicle C on 2nd day = 45 km/hr = 45 = 2.5 5 = 12.5 m/sec Ans: (5) 4. The distance travelled by vehicle F on day 2 was approximately what percent of the distance travelled by it on day 1? (1) 80 (2) 65 (3) 85 276 IBPS PO EXAM 2013 : Quantitative Aptitude

(4) 95 (5) 90 Solution: Reqd % = Ans: (5) 5. What is the respective ratio between the speeds of vehicle 0 and vehicle E on day 2? (1) 15: 13 (2) 17: 13 (3) 13: 11 (4) 17: 14 (5) None of these Solution: Reqd Ratio = = Ans: (2) = = 17:13 100 = 90.46 90%

277

IBPS PO EXAM 2013 : Quantitative Aptitude

Directions (6-10) Study the following pie-chart and table carefully and answer the questions given below: Percentagewise distribution of the number of mobile phones sold a shopkeeper during six months Total number of mobile phones sold = 45000

The respective ratio between the numbers of mobile phones sold of company A and company B during six months

6. What is the respective ratio between the number of mobile phones sold of company B during July and those sold during December of the same company? (1) 119: 145 (2) 116: 135 (3) 119: 135 (4) 119: 130 (5) None of these Solution: Total number of mobiles sold in the month of July 278 IBPS PO EXAM 2013 : Quantitative Aptitude

= 45000

= 7650

Mobile phones sold by Company B in the month of July = 7650 = 3570 Total number of mobile phones sold in the month of December = 45000 = 7200 Mobile phones sold by Company B in the month of December = 7200 = 4050 Reqd ratio = Ans: (3) 7. If 35% of the mobile phones sold by company A during November were sold at a discount, how many mobile phones of company A during that month were sold without a discount? (1) 882 (2) 1635 (3) 1638 (4) 885 (5) None of these Solution: Number of mobile phones sold in the month of November = 45000 = 5400 Number of mobile phones sold by Company A in the month of November = 5400 = 2520 Number of mobile phones without discount in the month of November by Company A = 2520 = 2520 0.65 = 1638 Ans: 3 8. If the shopkeeper earned a profit of Rs. 433/- on each mobile phone sold of company B during October, what was his total profit earned on the mobile phones of that company during the same month? (1) Rs. 6,49,900 (2) Rs. 6,45,900 (3) Rs. 6,49,400 (4) Rs 6,49,500 (5) None of these 279 IBPS PO EXAM 2013 : Quantitative Aptitude = = = 119: 135

Solution: Number of mobile phones sold in the month of October = 45000 = 3600 Number of mobile phones sold by Company B in the month of October = 3600 = 1500 Total profit earned by Company B in the month of October = 1500 433 = 649500 Ans: (4) 9. The number of mobile phones sold of company A during July is approximately what percent of the number of mobile phones sold of company A during December? (1) 110 (2) 140 (3) 150 (4) 105 (5) 130 Solution: Number of mobile phones sold in the month of July = 45000 = 7650 Number of mobile phones sold by Company A in the month of July = 7650 = 4080 Number of mobile phones sold in the month of December = 45000 = 7200 Number of mobile phones sold by Company A in the month of December = 7200 = 3150 Reqd % = Ans: (5) 10. What is the total number of mobile phones sold of company B during August and September together? (1) 10000 (2) 15000 (3) 10500 (4) 9500 (5) None of these = 129.52 130

280

IBPS PO EXAM 2013 : Quantitative Aptitude

Solution: Number of mobile phones sold in the month of August = 45000=9900 Number of mobile phones sold in the month of September = 45000 = 45000 = 11250 Number of mobile phones sold by Company B in the month of August = 9900 = 5500 Number of mobile phones sold by Company B in September= 11250 = 4500 Total number of mobile phones sold in August and September by Company B = 5500+4500= 10000 Quicker Method: Total number of mobile phones sold by Company B in August and September = 10000 Ans: (1) .

281

IBPS PO EXAM 2013 : Quantitative Aptitude

Directions: (11-15) Study the following information and answer the questions that follow: The graph given below represents the production (in tonnes) and sales (in tonnes) of company a from 2006-2011.

The table given below represents the respective ratio of the production (in tonnes) of Company A to the production (in tonnes) of Company B, and the respective ratio of the sales (in tonnes) of Company A to the sales (in tonnes) of Company B.

11. What is the approximate percentage increase in the production of Company A (in tonnes) from the year 2009 to the production of Company A (in tonnes) in the year 2010? (1) 18% (2) 38% (3) 23% (4) 27% (5) 32% Solution: Production of Company A in year 2009 = 550 Production of Company A in year 2010 = 700

282

IBPS PO EXAM 2013 : Quantitative Aptitude

Reqd % = = = 27.27

100 = 27%

100

Ans: (4) 12. The sale of Company A in the year 2009 was approximately what percent of the production of Company A in the same year? (1) 65% (2) 73% (3) 79% (4) 83% (5) 69% Solution: Sales of Company A in year 2009 = 400 Production of Company A in year 2009 = 550 Reqd % = Ans: (2) 13. What is the average production of Company B (in tonnes) from the year 2006 to the year 2011? (1) 574 (2) 649 (3) 675 (4) 593 (5) 618 Solution: Average production of Company B = = = 675 100 = = 72.72 73%

Ans: (3)

283

IBPS PO EXAM 2013 : Quantitative Aptitude

14. What is the respective ratio of the total production (in tonnes) of C ompany A to the total sales (in tonnes) of Company A? (1) 81: 64 (2) 64: 55 (3) 71: 81 (4) 71: 55 (5) 81: 55 Solution: = = = 81: 55

Ans: (5) 15. What is the respective ratio of production of Company B (in tonnes) in the year 2006 to production of Company B (in tonnes) in the year 2008? (1) 2: 5 (2) 4: 5 (3) 3: 4 (4) 3: 5 (5) 1: 4 Solution: Production of Company B in the year 2006 = 150 = 600 Production of Company B in the year 2008 = 200 Ratio = = 3: 4 Ans: (3)

284

IBPS PO EXAM 2013 : Quantitative Aptitude

Practice Set (Data Interpretation-Miscellaneous)


Directions (Q. 1-5): Study the radar-graph carefully and answer the questions that follow. Monthly salary (in thousands) of five different persons in three different years (RBI GradeB Officers Exam 2011)

1. What is the average of the monthly salary of Sumit in the year 2008, Anil in the year 2009 and Jyoti in the year 2010? (1) (2) (3) (4) (5) Rs.20, 000 Rs.5, 000 Rs.45, 000 Rs.15, 000 None of these

2. The total monthly salary of Arvind in all years together is what per cent of the total monthly salary of all the five persons together in the year 2008? (1) (2) (3) (4) (5) 55% 60% 75% 70% None of these 285 IBPS PO EXAM 2013 : Quantitative Aptitude

3. Among the five persons, whose earning per month over all the years together is the second lowest? (2) (3) (4) (5) (6) Sumit Anil Jyoti Arvind Poonam

4. What is the per cent decrease in the monthly salary of Poonam in the year 2009 as compared to her monthly salary in the previous year? (1) (2) (3) (4) (5) 40% 10% 20% 80% None of these

5. If Jyoti's monthly salary in the year 2010 was increased by 30 per cent what would her monthly salary be in that year? (1) (2) (3) (4) (5) Rs.36, 000 Rs.39, 000 Rs.45, 000 Rs.42, 000 None of these

286

IBPS PO EXAM 2013 : Quantitative Aptitude

Directions (Q. 6-10): Study the following pie-chart and bar-graph and answer the following questions. (Corporation Bank PO 2011) Percentage wise distribution of teachers in six different districts Total number of Teachers = 4500 Percentage of Teachers

287

IBPS PO EXAM 2013 : Quantitative Aptitude

6. What is the total number of male teachers in District F, female teachers in District C and female teachers in District B together? (1) (2) (3) (4) (5) 1080 1120 1180 1020 None of these

7. The number of female teachers in District D is approximately what per cent of the total number of teachers (both male and female) in District A? (1) (2) (3) (4) (5) 70 75 80 95 90

8. In which district is the number of male teachers more than the number of female teachers? (1) (2) (3) (4) (5) B only D only Both B and E Both E and F None

9. What is the difference between the number of female teachers in District F and the total number of teachers (both male and female) in District E? (1) (2) (3) (4) (5) 625 775 675 725 None of these

10. What is the ratio of the number of male teachers in District C to the number of female teachers in District B? (1) (2) (3) (4) (5) 11 : 15 15 : 11 15 : 8 30 : 13 None of these

288

IBPS PO EXAM 2013 : Quantitative Aptitude

Directions-(Q. 11-15) Study the following pie-chart and bar chart and answer the following questions. (Allahabad Bank Probationary Officers Exam 2011) Percentagewise Distribution of students in six different Schools Total number of Students = 6000 Percentage of Students

Number of Boys out of the 6000 Students in each School Separately.

289

IBPS PO EXAM 2013 : Quantitative Aptitude

11. What is the sum of the number of girls in School C. Number of girls in School-E and the number of boys in school-D together? (1) (2) (3) (4) (5) 1700 1900 1600 1800 None of these

12. What is the respective ratio of the number of boys in School-C, number of girls in School B and total number of students in School E? (1) (2) (3) (4) (5) 45: 7: 97 43: 9: 97 45: 7: 87 43: 9: 87 None of these

13. What is the difference between the total number of students in School-P and the number of boys in School E? (1) (2) (3) (4) (5) 820 860 880 900 None of these

14. In which school the total numbers of students (both boys and girls) together are equal to the number of girls in School E? (1) (2) (3) (4) (5) A B C D F

15. The Number of girls in School A is approximately what per cent of total number of students in School B? (1) 55 (2) 50 (3) 35 (4) 45 (5) 4 290 IBPS PO EXAM 2013 : Quantitative Aptitude

Data Interpretation-Miscellaneous Practice Set (Answers)


1 2 3 4 5 6 7 8 (1) (2) (4) (4) (2) 13 (1) 14 (5) 15 (3) (5) (2) (5) 9 10 11 12 (2) (3) (4) (3)

291

IBPS PO EXAM 2013 : Quantitative Aptitude

FEEDBACK

Disclaimer Readers are requested to verify/cross-check up to their satisfaction themselves about the advertisements, advertorials, and external contents. If any miss-happening, ill result, mass depletion or any similar incident occurs due to any information cited or referenced in this e-book, Editor, Director/s, employees of Jagranjosh.com cant be held liable/responsible in any matter whatsoever. No responsibilities lie as well in case of the advertisements, advertorials, and external contents.

292

IBPS PO EXAM 2013 : Quantitative Aptitude

You might also like